REENTRY PART 2 NUTRITION FUNDAMENTAL GERI

¡Supera tus tareas y exámenes ahora con Quizwiz!

The home health nurse observes that there is still about half a month's supply of Glucophage in the medicine bottle at the end of the month. What question would be the most appropriate for the nurse to ask?

"Are you trying to make the medication stretch for 2 months?"

A 70-year-old woman asks, "How in the world can my bones be brittle when I eat all the right foods?" Which response by the nurse would be the most informative? a. "Calcium loss is expected in the older adult." b. "Calcium is continuously withdrawn from bone for nerve and muscle function." c. "Smoking and alcohol consumption speed calcium loss from the bones." d. "Walking and standing increase calcium loss from the bone."

"Calcium is continuously withdrawn from bone for nerve and muscle function."

When the nurse answers the call light after a delay of 5 minutes, the angry patient says, "You made me wait an hour. I am in pain and no one is willing to help me." The nurse's best response would be:

"I know it must have seemed like an hour. I'll bring your medication."

What factors increase the risk of vehicular accidents for the older adult? (Select all that apply.) a. Increased reflex time b. Cognitive disorders c. Altered depth perception d. Changes in night vision e. Reduced flexibility

- Cognitive disorders -Altered depth perception -Changes in night vision -Reduced flexibility

Which long-term care residents would benefit from a referral to a podiatrist? (Select all that apply.) a. 90-year-old poststroke patient with right hemiparesis b. 85-year-old diabetic patient who is 100 lb overweight c. 80-year-old resident with phlebitis and a stasis ulcer on the left ankle d. 75-year-old resident with congestive heart failure (CHF) e. 70-year-old resident with chronic obstructive pulmonary disease (COPD)

-85 year old diabetic patient who is 100lb overweight -80 year old resident with phlebitis and a stasis ulcer on the left ankle

What is the purpose of the hydrocolloid dressing applied to a clean stage II pressure ulcer? (Select all that apply.) a. Débride the ulcer b. Prevent shear force trauma c. Absorb the exudate d. Harden eschar e. Make an air-occlusive seal

-Debride the ulcer -Prevent shear force trauma -Absorb the exudate -Make an air-occulusive seal

What would be appropriate sources of support for the grieving patient? (Select all that apply.) a. Friends b. Family c. Self-help magazines d. Support groups e. Therapist

-Friends -Family -Support groups -Therapist

What factors make the older adult more susceptible to pressure ulcers? (Select all that apply.) a. The epidermal layer has thickened b. Subcutaneous fat has diminished c. Bruising is prevalent d. Skin receptor cells have reduced in sensitivity e. The skin is dry and scaly

-Subcutaneous fat has diminished -Skin receptor cells have reduced in sensitivity

The nurse conducting a patient assessment records a seated blood pressure of 135/86. What standing blood pressure would be indicative of orthostatic hypotension? a. 145/85 b. 134/76 c. 130/72 d. 124/62

124/62

What is the recommended amount of sodium that should be consumed in a day?

1500 MG

The nurse is explaining the activity recommendations from the USDA's new MyPlate plan. What is the minimum amount of moderate weekly exercise needed to balance nutritional intake?

2 hours and 30 minutes

To avoid interfering with sleep, when should activity be avoided during the day? a. 30 minutes before bedtime b. 1 hour before bedtime c. 2 hours before bedtime d. 3 hours before bedtime

2 hours before bedtime

Flick chest

2 or more runs fractured in 2 or more places resulting in instability in part of the chest wall with associated hemothorax, pneumothorax, and pulmonary contusion

What percentage of body fluid in an adult can be deadly ?

20%

Normal kidneys filter blood at what rate?

225 ml/min

The nurse encourages a patient who has been vomiting to drink fluids because the body fluid lost daily must match the amount of fluid taken in to maintain homeostasis. What is the recommended daily amount of fluid for an adult? a. 1000 mL b. 1500 mL c. 2050 mL d. 2500 mL

2500 ml

To demonstrate the energy-producing potential of different foods, the nurse explains that 3 grams of lean meat produces 12 kcal/g, whereas 3 grams of fish oil would produce:

27 kcal/g

During pregnancy and lactation females should increase their water intake up to

3 liters/day

At least how many minutes of daily activity is recommended for a 75-year-old woman? a. 15 b. 90 c. 30 d. 45

30

When doing exercises such as walking or swimming, how many minutes must the older adult perform the activity in order for it to be considered beneficial?

30 MINUTES

What is the minimum amount of urine the kidneys can excrete

30 mL

The nurse is aware that an adult has the urge to urinate when the bladder has approximately _____ mL of urine in it.

300

Magnesium

4th most abundant mineral in the body and the second most abundant cation in intracellular fluid of the body

What percentage ofthe adult body is water

50 to 60%

What percentage of an adult's body weight consists of water? a. 10% to 20% b. 30% to 40% c. 50% to 60% d. 70% to 80%

50% to 60%

What is the kcal/g for fat?

9 kcal/g

Bicarbonate

A main anion of the extracellular fluid

assessing respiratory status

A patient with multiple serious injuries sustained in a motorcycle accident is lying beside his wrecked motorcycle unconscious and bleeding when the rescuer arrives at the scene. What will be the rescuer's priority action? a. Assessing blood loss b. Assessing respiratory status c. Obtaining vital signs d. Organizing laypeople at the scene

Yellow

A physician documents that a patient has a scleral icterus. How does the nurse describe the color of the patient's sclera? a. Red b. Blue c. Green d. Yellow

Lithotomy

A physician needs to insert a vaginal speculum into a patient for a vaginal examination. In what position should the nurse place the patient? a. Sims b. Prone c. Lithotomy d. Dorsal recumbent

Tilted with the head of the bed down

A physician orders a patient to be placed in the Trendelenburg position. How will the nurse position the bed? a. On the floor b. Parallel with the floor c. Tilted with the head of the bed down d. Tilted with the foot of the bed down

Which senior citizen political action group uses volunteers and lobbyists to advance the interests of older adults? a. American Association of Retired Persons (AARP) b. National Council of Senior Citizens (NCSC) c. National Alliance of Senior Citizens (NASC) d. Gray Panthers

American Association of Retired Persons (AARP)

Which of the following are building blocks of protein? a. Monosaccharides b. Glucose, fructose, and galactose c. Amino acids d. Cellulose and glycogen

Amino acids

Which of the following is not a lipid or lipoid substance? a. Steroids b. Vitamins A, D, E, and K c. Prostaglandins d. Ammonia

Ammonia

place the egg in a saucer under the bed

An American Indian patient requests that an egg yolk be placed in a saucer and put under his bed to absorb the pain. What should the nurse do? a. Explain that medication will relieve the pain better b. Place the egg in a saucer under the bed c. Ask the physician for permission d. Warn that housekeeping staff will remove the egg

What are some medications that have restrictions to certain foods ?

Anticoagulants Antibiotics Aspirin Diuretics Antihypertensives Laxatives

What is the function of vitamin C ?

Antioxidant, wound healing, tissue growth and maintenance, proper immune function absorption of iron

What is the function of Vitamin E ?

Antioxidant; protection of cell membranes

Disease

Any disturbance of a structure or function of the body is a pathologic condition. What is the term for this condition? a. Injury b. Condition c. Disease d. Pathology

Endorphins

Anyone of the neuropeptifes composed of many amino acids elaborated by the putuitary gland and actinic on the central and peripheral nervous systems to reduce pain

Gerontology encompasses application to __________. (Select all that apply.) a. appropriate housing b. health care c. public education d. business ventures e. government-sponsored pensions

Appropriate housing Health care Public education Buisness ventures

The nurse is educating a patient regarding the need to avoid foods high in potassium. What food choices led the nurse to conclude that teaching was not effective? a. Apples and green beans b. Kiwis and onions c. Apricots and asparagus d. Grapes and lima beans

Apricots and asparagus

water

As a safety precaution against breakage of dentures, the nurse should place __________ in the emesis basin before cleaning the dentures.

Symptoms

As part of an assessment, the nurse asks the patient for subjective information related to the present illness. What are the subjective findings perceived by the patient? a. Assessments b. Symptoms c. Signs d. Observations

Cardiopulmonary resuscitation CPR

Basic emergency procedure for life support consisting of artificial respiration and manual external cardiac massage

The nurse explains that a diet low in fiber results in a small stool that a. moves rapidly through the intestines. b. becomes excessively dry. c. overstimulates the defecation reflex. d. contributes to frequent bowel movements.

Becomes excessively dry

Tip of the nose to the earlobe to the xiphoid process

Before inserting a nasogastric tube, what measurement should the nurse take? a. Tip of the nose to the earlobe to the xiphoid process b. Bridge of the nose to the xiphoid process c. Nose to the top of the ear to the stomach d. Clavicular notch to the stomach

What is the deficiency of Vitamin B1?

Beriberi Mental confusion Anorexia Muscle weakness and wasting Chronic alcoholism

What is the name for Vitamin A ?

Beta Carotene or Retinol

What organ in the body makes most of the cholesterol in the body?

By the liver From saturated fat

Which of the following is a waste product of the aerobic glucose metabolism? a. ATP b. CO2 c. Urea d. Glycogen

CO2

paramedics arrive

CPR has been initiated at an accident site. When can CPR be terminated? a. Victim is clinically dead b. Victim is brain dead c. Paramedics arrive d. Rescuer perceives CPR is futile

What is the name for Vitamin D?

Calciferol

How is ammonia toxic to the brain?

Causes disorientation and a diminished level of consciousness

Hypertonic

Cell to shrink ; higher osmotic pressure

Hypotonic

Cells to swell and bust; lower osmotic pressure

What would be the most effective intervention to prevent pressure ulcers in the bedridden patient? a. Perform skin assessment every day. b. Use a draw sheet to move the patient. c. Change the patient's position every 2 hours. d. Remove wet bed linens promptly.

Change the patients position every 2 hours

What is the most important steroid in the body?

Cholesterol

What is the food sources of vitamin c?

Citrus fruits and juices Strawberries Kiwi fruit Melon Broccoli Peppers Tomatoes Potatoes Fortified beverages

From the inner toward the outer canthus

Clear water is used to cleanse the eyes. It is important to use proper technique when cleansing the eyes to prevent infection. What direction will the water flow when cleansing a patient's eyes? a. Upward toward the forehead b. Downward toward the chin c. From the outer toward the inner canthus d. From the inner toward the outer canthus

combines the somatic mutation free radical and crosslink theories to suggest that chemicals produced by metabolism accumulate in normal cells and cause damage to body organs, such as the muscles heart, nerves, and brain.

Clinker theory

A 70-year-old woman complains, "I weigh exactly the same as I did when I wore a size 10 and now I can barely squeeze into a size 16." Which statement by the nurse would most correctly explain the size change to the woman? a. "Metabolism in the older adult creates increased adipose tissue." b. "Postmenopausal women gain adipose tissue related to loss of calcium." c. "Decrease in muscle mass is replaced with adipose tissue." d. "Kyphosis causes a redistribution of weight."

Decrease in muscle mass is replaced with adipose tissue."

A nurse is caring for a client with riboflavin deficiency. What signs of riboflavin deficiency should the nurse monitor for in the client? (Select all that apply) A. dermatitis B. Glossitus C. Photophobia D. Peripheral Paralysis

Dermatitis Glossitus Photophobia

What is the deficiency of Vitamin B7? (Biotin)

Dermatitis, conjunctivitis, alopecia, and depression

What is the toxicity of magnesium ?

Diarrhea

What are the three types of passive transport? (Select all that apply.) a. Diffusion b. Titration c. Osmosis d. Distillation e. Filtration

Diffusion Osmosis Filtration

What three processes represent passive transport diffusion?

Diffusion filteration and osmosis

The determination of daily weights is used to monitor a. serum K+. b. diuretic therapy. c. concentration of plasma protein. d. pH.

Diuretic therapy

A mother brings in her 2 year old child that ate 10 gummy adult vitamins because he thought it was candy. Which question is the most important to assess for potential toxicity?

Does the product contain Vitamin C?

Which assessment finding in a 75-year-old man on a chlorpromazine (Thorazine) protocol should be immediately reported to the physician?

Drool

Legs and feet

During a head-to-toe assessment, the nurse assesses the patient's perineal area. Which area should the nurse assess next? a. Chest b. Arms c. Abdomen d. Legs and feet

III

During a neurologic assessment, the nurse notes a patient has a unilateral, dilated, and nonreactive pupil. This is a sign that the patient is experiencing pressure on which cranial nerve? a. I b. II c. III d. IV

During inspiration

During a physical assessment, the nurse listens for adventitious lung sounds. Crackles are classified as fine, medium, or coarse. When are these sounds most often auscultated? a. During expiration b. Following expiration c. During inspiration d. Following inspiration

Which of the following is NOT a way someone can lose sodium?

Eating 5 pounds of boiled crawfish

What can the nurse implement to reduce episodes of bowel incontinence? a. Encourage intake of foods that cause mild constipation. b. Use appropriate disposable garments, pads, and bed covering. c. Establish a toileting schedule. d. Coach the patient in Kegel exercises.

Establish a toileting schedule

What data would be collected during the identifying information portion of the health interview? (Select all that apply.) a. Ethnicity b. Previous or current occupation c. Educational background d. Perception of general health e. Completion of advanced directives

Ethnicity Previous or current occupation Educational background

How often should a 75 year old have a visual screening?

Every 12 months

In most cases, the more expensive the vitamin supplement the better quality it is

False

What is the function of sodium?

Fluid and acid base balance, nerve conduction muscle contraction

The nurse explains that the term that describes the ability to make judgments quickly on the basis of unfamiliar stimuli is __________ __________.

Fluid intelligence

Extracellular

Fluid outside the cells of the body

Epistaxis

Hemorrhage of the nose; nosebleed

What does filtration require to occur?

Hydrostatic pressure

Where is code stored?

In the nucleus in the DNA

The nurse cautions a patient with a pancreatic disorder that will interfere with the digestion of fats and may lead to a clotting disorder. What is the cause of these potential problems?

Inability to use vitamin K

The patient has begun taking psyllium (Metamucil). What advice should be given to the patient? a. Eat several servings of fresh fruit and vegetables a day. b. Avoid citrus fruit juices. c. Reduce intake of carbonated drinks. d. Increase his fluid intake to 3000 mL a day.

Increase his fluid intake to 3000 ml a day

A 13 y/o child is considered overweight. Which of the following would be MOST helpful in treating this condition?

Increasing physical activity

NG tube

Inserted through the nose into the stomach

____________________softens stools, speeds transit of foods through the digestive tract, and reduces pressure in the colon.

Insoluble fiber

What would be included in a list of the five techniques of health assessment? (Select all that apply.) a. Inspection b. Palpation c. Interview d. Auscultation e. Percussion

Inspection Palpation Auscultation Percussion

According to Erikson, what is the primary development task of the older adult population?

Integrity versus despair

Acute pain

Intense unpleasant sensation of short duration lasting less than 6 months

Which fluid compartment is located between the cells and is also called tissue fluid? a. Lymph b. Plasma c. Interstitial d. Transcellular

Interstitial

Which of the following forms the greatest extracellular fluid compartment? a. Lymph b. Interstitial c. Plasma d. Transcellular

Interstitial

When a patient takes substances into the body, they first enter the extracellular compartment. What must the substances enter to carry out their function? a. Horizontal compartment b. Intracellular compartment c. Compartmental d. Vertical compartment

Intracellular compartment

Which of the two compartments found inside body fluids is the largest?

Intracellular fluid and extracellular

What is name of the plasma within the vessels known as?

Intravascular fluid

What is iodine food sources?

Iodized salt, seafood, plants grown in iodine rich soils

The most common deficiency in the United States and in the world is A. magnesium B. calcium C. zinc D. iron

Iron

A school nurse is teaching a group of adolescents about adequate nutrition. What increased intake should the nurse encourage?

Iron and calcium

A nurse caring for a patient who is prescribed a full-liquid diet recognizes that this diet lacks some nutrients. What nutrients are lacking?

Iron and fiber

Brain death

Irreversible form of unconsciousness characterized by a complete loss of brain function while the heart continues to beat

What is the goal of empathetic listening?

It allows the nurse to interpret what the patient has said

Which of the following is true of ageism? a. It is discrimination against persons solely on the basis of age. b. It causes a person to fear aging. c. It involves the use of cultural sensitivity to address concerns of aging. d. It focuses on resources for the older adult.

It involves the use of cultural sensitvity to address concerns of aging

A friend tells you she thinks her father is experiencing a "midlife crisis" because he purchased a new red sports car, started wearing trendy clothing, and is considering a career change. Whose theory explains this behavior?

Jung's

What should the home health nurse suggest in the case of a fire in the home of the older adult? (Select all that apply.) a. Keep a flashlight at the bedside b. Use an appropriate fire extinguisher to control fire c. Keep the doors open for an easy escape route d. Call 911 before exiting the home e. Open the windows to decrease smoke

Keep a flashlight at the bedside

When asked about the severity of pain, the 93 year old patient does not answer right away. What is the best response by the nurse?

Keep eye contact and wait for the answer

Homeostasis

Keeping body fluids in balance a relatively constancy in the internal environment of the body naturally maintained by adaptive responses that promote healthy survival

Which of the following is an example of metabolic acidosis with a respiratory compensation? a. Morphine overdose and diuresis b. Persistent vomiting and hypoventilation c. Ketoacidosis and Kussmaul respirations d. Hypocalcemic tetany and hyperventilation

Ketoacidosis and Kussmaul respirations

Which of the following is most likely to happen when fatty acids are broken down rapidly and incompletely? a. The plasma pH decreases, causing metabolic alkalosis. b. The patient hypoventilates in an attempt to correct the pH disturbance. c. Ketoacids are produced, causing metabolic acidosis. d. The plasma [H+] decreases. ANS: C

Ketoacids are produced, causing metabolic acidosis

Which organ of the body helps filter and recycle water everyday?

Kidney

Prostaglandins, Lipoproteins and fat-soluble vitamins are examples of what?

Lipoid substances

Clear liquid diets includes

Liquids that are easily digested and absorbed and leave little residue or waste in the GI tract

A diet for a patient with anemia should include:

Liver

Patients with anemia should include what ?

Liver

Which of the following is a a good source of iron? A. liver B. dairy products C. walnuts D. Tea

Liver

What is the food source for folic acid ? (vitamin b9)

Liver Green leafy vegetables Legumes Fruits Enriched grain products

The male patient who has been on long-term antibiotic therapy inquires what may have caused his thrush. What is the most informative response of the nurse? a. A vitamin A deficiency b. Long-term antibiotic therapy has destroyed the normal flora of his mouth c. An allergy to the antibiotic d. Oral hygiene has been inadequate

Long term antibiotic therapy has destroyed the normal flora of his mouth

What are the effects of decreased speed impulses in the older adult? a. A "scrambled" message in the brain b. Longer time required to complete an activity c. Increased confusion d. Forget how to complete an activity

Longer time required to complete an activity

Which lipoprotein account for 60% to 70% of all the cholesterol in the blood?

Low density lipoprotein (LDLs)

The 75-year-old man who has been hospitalized following a severe case of pneumonia is concerned about his mounting hospital bill and asks if his Medicare coverage will pay for his care. What would be the most helpful response by the nurse? a. Medicare Part C pays 50% of all medical costs for persons older than 65. b. Medicare Part B pays hospital costs and physician fees. c. Medicare Part A pays for inpatient hospital costs. d. Medicare Part D pays 80% of the charges made by physicians.

Medicare Part A pays for inpatient hospital costs

What is the most beneficial legislation that has influenced health care for the older adult? a. Medicare and Medicaid b. Elimination of the mandatory retirement age c. The Americans with Disabilities Act d. The Drug Benefit Program

Medicare and Medicaid

Which of the following patients usually do NOT require extra iron?

Men

The nurse explains that the tool that allows the evaluation of core function in a resident in a long-term facility is the __________ __________ __________.

Minimum Data Sheet

Which of the following are indicators of self-neglect in the aging person? (Select all that apply.) a. Misbalanced checkbook b. Reduced personal hygiene c. Increased alcohol consumption d. Irritability e. Loss of weight

Misbalanced checkbook Reduced personal hygiene Increased alcohol consumption Loss of weight

Rapid eye movement

One of the two highly individualize sleeping states that follows NREM state. May last from a few minutes to a half an hour and alternate with NREM periods dreaming occurs at this time

Non-rapid eye movement

One of the two highly individualized sleeping states divided into 4 through which a sleeper progresses during a typical sleeping cycle; represents 3/4 of a period of typical sleep

The nurse explains to a patient that the drug Lasix reduces edema by drawing water from the interstitial space into the intravascular space. What is this process called? a. Diffusion b. Filtration c. Osmosis d. Homeostasis

Osmosis

What is the definition When patient is given a diuretic, it draws interstitial space into the intravascular space. What is this process called?

Osmosis

What is the term for softening of the bones?

Osteomalacia

What is the name for Vitamin b2?

Riboflavin

The 80 year old woman tells the home health nurse, "I can only eat cereal because I just can't make those dentures work!" What would be the most appropriate suggestion by the nurse?

Take only tiny bites of food Chew slowly Use a dental adhesive Select soft, nonsticky foods (e.g. ground meat, boiled carrots)

poison control center

The emergency department nurse admits a victim of poisoning. Who should the nurse call to receive the best assistance for dealing with this victim? a. American Red Cross b. Fire department paramedics c. Poison control center d. Civil defense office

A TENS unit

The home health nurse is caring for a patient with an implanted pacemaker. What type of pain management would be contraindicated? a. Peripheral analgesics b. A TENS unit c. Opioid analgesics d. Adjuvant analgesics

administering diuretics in the morning

The home health nurse is instructing the family of an older adult patient with arthritis about sleep promotion. What intervention can best promote sleep for the older adult patient? a. Giving nonsteroidal anti-inflammatory drugs (NSAIDs) in the mornings b. Administering diuretics in the mornings c. Encouraging daytime sleeping d. Avoiding the stimulation of backrubs or warm drinks before bedtime

Proper measurement techniques are necessary Cuff fits over clothing Stethoscope is not required

The home health nurse is preparing to educate a patient regarding electronic self-blood pressure measurement. What information should the nurse provide regarding this procedure? (Select all that apply.) a. Expect precise values b. Proper measurement techniques are necessary c. Cuff fits over clothing d. Stethoscope is not required e. Recalibration is not necessary

What is the name of vitamin B1?

Thiamin

What is the role of LDLs?

To deliver cholesterol to nonhepatic tissue

120 F

To what temperature should water for eye compress be heated? a. 95° F b. 110° F c. 115° F d. 120° F

the nursing care and the service provided to older adults

gerontics

systolic

number or reading that represents ventricles contracting

umbilicus

point of abdomen at which the umbilical cord joined the fetus

alignment

relationship of various body parts to one another

data obtained orally

subjective data

organic disease

What type of disease results in a structural change in an organ that interferes with its functioning? a. Functional disease b. Organic disease c. Acute disease d. Chronic disease

To protect the nasolacrimal ducts

Why should the nurse instill eye irrigation from the inner to the outer canthus? a. To avoid harming the sclera b. To include the conjunctiva in the irrigation c. To keep the pupil constricted d. To protect the nasolacrimal ducts

When assessing for ineffective health maintenance, what should be assessed?

Willingness to follow a health maintenance plan possible conflict between a health maintenance plan and culture. The presence of prohibited items such as sweets, alcohol, and cigarettes The level of cognitive impairement

The patient with terminal liver cancer says to the nurse, "I'm going to take a long time to die, aren't I? I'm going to get sicker and weaker every day." What would be the best response by the nurse? a. "You will die in 4 to 6 months." b. "I don't want to hear this kind of negative talk. Make use of the time you have." c. "We have many medications that can make you feel better." d. "What concerns you the most about dying?"

"What concerns you most about dying?"

The nurse is engaging in the patient in social conversation. What is the benefit of social conversation in the health care setting?

It lets the patient know that he or she is considered to be a person, not just a patient

What is the end products of aerobic catabolism?

Lactic acid

Which of the following best indicates the role of albumin in water balance? a. Blocks the renal reabsorption of Na+ b. Enhances the renal excretion of K+ c. Maintains plasma oncotic pressure d. "Plugs up" capillary pores, keeping water in the plasma ANS: C

Maintains plasma oncotic pressure

What is the function of Vitamin D?

Maintenance of blood calcium Phosphorus balance

quicker

The nurse is caring for a patient using patient-controlled analgesia (PCA). What is a major advantage to this method? a. Less expensive b. More effective c. Less addictive d. Quicker

old drainage

The nurse is collecting a specimen for a wound culture. What should be avoided when collecting this specimen?

the point of pain

The nurse is performing passive range of motion (ROM) for the patient. How will the nurse move the joint through ROM? a. The fullest extent b. Place the joint in normal position c. The point of pain d. Relax the patient

Five

The nurse is performing passive range-of-motion exercises on a patient following a traumatic injury. What is the number of times the nurse should move each joint when performing passive range-of-motion (ROM) exercises? a. Three b. Four c. Five d. Six

fatigue

The nurse is planning interventions for a patient experiencing pain. For what type of synergistic relationship should the nurse assess? a. Inflammatory process b. Circulatory disorder c. Food allergy d. Fatigue

white

The nurse is preparing a patient for a barium enema. What color will the nurse inform the patient his stools will be following this procedure?

Answer questions for clarification

The nurse is preparing a patient for a diagnostic examination. What can the nurse implement to assist with reducing anxiety?

Center of Gravity

The nurse points to the X in the illustration below and describes this point as the ________ of _________.

Remission

What is the term used to describe a disease where there has been a partial or complete disappearance of clinical and subjective characteristics of the disease? a. Acute b. Functional c. Chronic d. Remission

get a physicians order

What must the nurse do before applying a safety reminder device (SRD)? a. Get permission from the family b. Assess patient's skin condition c. Get a physician's order d. Explain the SRD to the patient

Bending knees and hips

What should the nurse do to protect his or her back when lifting or moving a patient? a. Lowering the height of the bed b. Holding the back straight with locked knees c. Bending knees and hips d. Getting the patient to the side of the bed

encourage water intake

What should the nurse do when preparing the patient for a bone scan?

assess patient for allergies to dye or shellfish

What should the nurse do when preparing the patient for an abdominal scan?

Core

What type of body temperature remains relatively constant? a. Surface b. Rectal c. Oral d. Core

febrile

body temp above normal

febrile

body temperature above normal

How does infection present itself in older adults?

body temperature may not become significantly elevated until the infection is severe and pain may not be present to indicate infection

a communication technique used when there are inconsistencies in information or when verbal and nonverbal messages appear contradictory

confrontation

The body uses 22 common amino acids, but 9 of them be obtained from protein in the diet and are considered

essential

What are the elements that are found in food and necessary for good health but that the body cannot make?

essential nutrients

questions or problems related to moral values or principles

ethical dilemmas

assessment

evaluation or appraisal of a condition

flatulence

excess formation of gases in the stomach or intestine

a simple one in which all members share a common historical and cultural experience

homogeneous

lack of patient compliance with providers recommendations about the daily timing, dosaging, and frequency of medication use or other recommended health practices

nonaderhence

failure of a patient to follow through with recommended health practices

noncompliance

information obtained through the senses or measured by instruments

objective data

hypotension

occurs when the blood pressure is below normal

Hypertension

occurs when the elevated BP is above normal

Which of the following is true of sucrose, maltase, and lactase? a. All are double sugars. b. All are proteolytic. c. All are disaccharidases. d. All are storage forms of monosaccharides.

all are proteolytic

Amino acids are a. joined together by peptide bonds. b. the building blocks of protein. c. characterized as essential and nonessential. d. all of the above

all of the above

pronation

palm of hand turned down

Which hormone stimulates the kidneys to reabsorb calcium and to excrete phosphate? a. ADH b. Aldosterone c. Parathyroid hormone d. Erythropoietin

parathyroid hormone

compartment syndrome

pathologic condition caused by progressive development of arterial compression and reduced blood supply to an extremity. Increased pressure from external devices (casts, bulky dressing) causes decreased blood flow, resulting in ischemic tissue necrosis; most often occurs in the extremities

How are amino acids joined together?

peptide bonds

Cyanosis

slightly bluish, grayish, slatelike, or dark discoloration of the skin due to presence of abnormal amounts of reduced hemoglobin in the blood.

bradycardia

slow respiratory rate fewer than 12 bpm.

symptoms

subjective indication of a disease or a change in condition as perceived by the patient

Electrolyte

substances that is sometimes called a mineral or salt; develops tiny electrical charges when dissolved in water and breaks up into particles known as ions

Ingredients are listed in descending order by weight on Nutrition Facts label

False

Vitamin A is probably the most unstable of all vitamins

False

Synthesis of DNA, and RNA, and the formation of red and white blood cells; can help prevent neural tube defects in fetus if taken during pregnancy

Folate/Folic acid (B9)

Which of the following would least improve home security? a. Peephole in the door at a convenient height b. Brightly lit porch c. Large dog with a loud bark d. Hook and eye latch on the screen door

Hook and eye latch on the screen door

By allowing the patient to keep leaves in her room

How can the nurse demonstrate cultural sensitivity to a Haitian American patient?

Most body water is located within which space? a. Interstitial b. Intravascular c. Intracellular d. Transcellular

Intracellular

When entering the room of an older adult to insert a catheter, what would be the most effective approach by the nurse?

Introduce yourself at the door and ask, "May I insert this catheter for a urine specimen?

Common nutritional problems during childhood are

Iron deficiency, Anemia and Obesity

What evaluations should be conducted during an older adult female's routine physical examination?

Measurement of height and weight blood pressure electrocardiography rectal examination mammography

What causes the greatest damage to self-worth? a. Measuring self against internal ideals b. Measuring self against individual values c. Measuring self against external standards d. Measuring self against expressions of positive feedback

Measuring self against external standards

The nurse reminds the male patient with lactose intolerance that he can avoid the unpleasant symptoms of nausea, bloating, flatulence, and diarrhea, if he will avoid certain foods. What product should the patient be instructed to avoid?

Milk

What is the food source for Vitamin B2?

Milk Meats Poultry Fish Enriched and fortified grains and cereals

What is the food source for calcium

Milk, cheese, milk products, green leafy vegetables, broccoli, legumes, fish with bones, fortified cereals

110 F

Proper hair care is important for the patient's self-image. What is the proper water temperature when shampooing a patient's hair? a. 101° F b. 105° F c. 110° F d. 120° F

What is the most abundant organic matter in the body?

Protein

Chronic renal failure diet

Protein controlled; low sodium; potassium; fluid and phosphorus restricted. (must monitor patient electrolyte balance closely )

The nurse is counseling a patient about the difference between type 1 and type 2 diabetes. What should the nurse stress that patients with type 2 diabetes are required to receive on a daily basis?

Regular carbohydrate controlled meals

What can suppress the defecation reflex? a. Frequent episodes of diarrhea b. Long-term use of vitamin A and vitamin B complex c. Repeatedly ignoring the urge d. Excessive fiber and bulk in the diet

Repeatedly ignoring the urge

What condition occurs with the build up of carbon dioxide and increase carbonic acid

Respiratory Acidosis

Diamond position

The nurse is preparing to perform perineal care for the female patient. What is the best method for using a bath blanket to drape the patient? a. Square position b. Long position c. Diamond position d. Rectangular position

Podiatrist

The nurse is providing hand and foot care to a patient and notices the patient has extremely hard nails. Who is the person best prepared to provide nail care for patients with extremely hard nails? a. Physician b. RN c. CNA d. Podiatrist

Give a daily bath

The nurse is providing personal hygiene for a Hindu patient from India. What intervention should the nurse implement? a. Not serve meat b. Shampoo the patient's hair weekly c. Give a daily bath d. Cut nails monthly

less than 1%

The nurse is trying to reassure a patient who is concerned about receiving addictive drugs. What percentage of patients become addicted to analgesics? a. Less than 0.1% b. Less than 1% c. Less than 5% d. Less than 6%

injury

The nurse manager is providing an in-service regarding a "safe hospital environment." What will this education mainly focus on preventing? a. Falls b. Exposure to contaminants c. Injury d. Electrical hazard

ausculation

To listen for sounds within the body to evaluate the condition of the heart, lungs, pleura, intestines, or other organs or to detect fetal heart sounds

1.5

To maintain a wide base of support, the nurse should stand with the feet separated by the distance of _______ times the length of the nurse's shoe.

15

To prevent skin breakdown in a wheelchair-bound patient, the nurse teaches the patient to shift the patient's weight every _______ minutes.

The chest

While on break in the hospital cafeteria a nurse witnesses her pregnant co-worker start to choke. The co-worker is conscious, but unable to breathe. Where should the nurse administer thrusts? a. Below the navel b. The chest c. At the xiphoid process d. The upper back

Warm, edematous, and red

While teaching a patient about the signs of IV therapy-associated phlebitis, how does the nurse describe an area with phlebitis? a. Warm, edematous, and red b. Painful and cyanotic c. Painless and numb d. Edematous and cool

The area between 4 and 12 feet of a person is commonly referred to as which of the following?

social space

Water and chloride usually follow the movement of which cation? a. Bicarbonate b. Sodium c. Potassium d. Albumin

sodium

Flatness

soft, high pitched, flat sound produced by performed percussion over tissue.

is similar but proposes that aging results from deoxyribonuclei acid (DNA) damage caused by exposure to chemicals or radiation and that this damage causes chromosomal abnormalities that lead to disease or loss of function later in life.

somatic mutation theory

How can you tell if pressure ulcer is stage I or stage II

stage I-lesions are commonly dismissed as minor abrasions b/c their primary attribute is nonblanchable erythema. Stage II- characterized by skin loss

What are complex carbohydrates?

starch, glycogen, fiber

things that excite or incite an organism or part to function became active or respond

stimuli

Which of the following terms is most descriptive of ribose and deoxyribose? a. Steroids b. Lipids c. Sugars d. Disaccharides

sugars

codes

system of notification that allows information to be transmitted rapidly

The home health nurse is discussing oral hygiene with the older adult. Which of the following practices would be appropriate to add to the current maintenance plan?

using fluoride toothpaste

What site would be the best choice for an intramuscular injection to an emaciated 82-year-old patient?

ventrogluteal

What would the nurse be conveying when using touch as a form of communication? (Select all that apply.) a. Affection b. Understanding c. Concern d. Apathy e. Acknowledgment

affection understanding concern acknowledgment

What symptom could be an indicator of depression in an 80-year-old man? a. Repetitive routines b. Agitation and irritability c. Constant talking d. Seeking the company of family members

agitation and irritability

signs

an objective finding as perceived by the provider/examiner a sign can be seen heard, measured, or felt by the examiner

What are the factors that increase the risk for osteoporosis?

female gender caucasian or asian race small body frame family history osteoporosis poor nutrition (diet low in calcium and vitamin d) malabsorption disorders, such as celiac disease menopause (low estrogen) chemotherapy

thrill

fine vibration sensation all the artery, which is palpated by the examiner

What are the signs and symptoms of myocardial infarction in an older adult?

flu like symptoms, sudden onset dyspnea, or chest discomfort, confusion, and syncope

interstitial fluid

fluid between the cells or in tissues of the body

enlargement of the heart, often related to heart failure

cardiomegaly

perineal care

care given to the genitalia

oral hygiene

care of the oral cavity

Bicarbonate (HCO3-) is considered an anion because it a. contains hydrogen. b. contains carbon. c. contains oxygen. d. carries a negative charge.

carries a negative charge

What is the role of tRNA?

carries amino acids to the ribosome and matches them to the coded mRNA message

clouding of the lens of the eye, developing over time and resulting in progressive, painless loss of vision

cataract

Which of the following is least related to urea? a. Hepatic synthesis b. Nitrogenous waste c. Chemical formula is NH4+ d. Excreted in the urine

chemical formula is NH4+

What are the symptoms of ischemia in an older adult?

chest pain, pain radiating down the left arm, vague gastronintestinal(GI) discomfort or shortness of breath

How is TB diagnosed?

chest x-ray, sputum cultures, and blood test

grief that is complicated by adjustment disorders such as depression alcohol abuse or post traumatic stress disorder

complicated grief

knowledge that comes from past learning and prior experiences

crystalized intelligence

What should the nurse base her communication approach on for the most effective communication? (Select all that apply.) a. Culture b. Ethnicity c. Income level d. Perspective e. Level of education

culture ethnicity perspective level of education

the statistical study of human populations

demographics

When the 80-year-old female patient refuses to take a medication because it burns her stomach, what action should be taken by the nurse?

document the reason for refusal and report the refusal to the charge nurse

The nurse clarifies that the point at which a person on Medicaid Part D must assume full cost of medication is called the __________ __________.

donut hole

orthostatic hypotension

drop of 25 mmHg in systolic pressure and a drop of 10mmHg in diastolic pressure when moving from a lying or sitting position

What factors increase the risk of medication-related problems in the older adult? (Select all that apply.) a. Drug-testing methodology b. Age-related changes c. Polypharmacy d. Cognitive and sensory changes e. Lack of adequate medical follow-up

drug-testing methodology age related hcanges polypharmacy cognitive and sensory changes

What are some risk of the risk of hormone therapy?

heart attack, blood clots, stroke and breast cancer

Apical pulse

heartebeat as measured by the bell or disk of the stethoscope placed over the apex of the heart

What should an older adult be encouraged to implement in order to prevent lithium toxicity?

increase fluid intake to 3500 ml daily

Gluconeogenesis a. increases blood cholesterol. b. increases urea synthesis. c. increases blood glucose. d. causes hypoglycemia

increases blood glucose

A drug that blocks the renal reabsorption of sodium a. decreases the renal excretion of Na+. b. increases the amount of urine. c. decreases the excretion of bicarbonate. d. always increases the reabsorption of K+.

increases the amount of urine

K+ a. is a cation. b. is an acid. c. neutralizes H+. d. is the chief extracellular cation.

is a cation

Which of the following is characteristic of glycolysis? a. Occurs within the cytoplasm b. Functions aerobically c. Converts protein to glucose d. Completely metabolizes glucose to CO2, H2O, and ATP

occurs within the cytoplasm

objective data

of or pertaining to a clincial finding that is observed palpated or ausculated laboratory finds as well as radiologic and other studies are included observable and measurable signs

What can the long-term care nurse do to best relieve anxiety in a female resident? a. Stimulate her with group activity. b. Sedate her to allow her to sleep. c. Provide her with coffee for relaxation. d. Offer her a diversion of a quiet activity.

offer her a diversion of a quiet activity

Which older adult would be at the highest risk for suicide? a. One with a lack of activity b. One with an altered nutritional intake c. One who abuses alcohol d. One who has a chronic disease

one who abuses alcohol

Between meal snacks or supplements are given to clients

only with a physicians order

hearing loss associated with aging particularly of higher pitched sounds

presbycusis

found in a older people farsightedness resulting from a loss of elasticity of the lens of the eye and resulting in a decrease in power of accomadation

presbyopia

infiltration

process whereby a fluid passes into the tissues

purulent

producing or containing pus

The nurse is aware that children with __________ have the treatment and care needs of persons of advanced age.

progeria

proposes that everyone has a biological clock that starts ticking at conception.

programmed theory

preventing the spread of disease

prophalytic

inflammation

protective response of body tissues or irritation, injurt, or invasion, by disease producing organism

Glucose is usually broken down to a. make carbon dioxide. b. provide energy (ATP) for the working cells. c. use up urea. d. make glycogen.

provide energy (ATP) for the working cells

study of the use of personal space in communication

proxemics

anorexia

psychoneurotic disorder characterized by prolonged refusal to eat

pertaining to a combination of psychological and social factors. the psychosocial theories of aging attempt to explain why older adults have different responses to the aging process

psychosocial

PASS

pull the pin,aim low, squeeze handle, sweep the unit

atmosphere of mutual respect and understanding

rapport

The nurse uses a diagram to depict __________, which is a term for interaction between different roles.

relationship

connections formed by the dynamics interaction of individuals who play interrelated roles

relationships

temperature

relative measure of sensible heat or cold.

What does catabolism releases?

release energy that is converted into ATP and used to "run" the body

also called life review, involves allowing the older adult to think back and reflect on past experiences

reminisence

crackles

short, discretem interrupted crackling or bubbling adventitious breath sounds heard on auscultation of the chest most commonly on inspiration.

labia minora

smaller fold of tissue covered by labia majora

Which statement by the nurse would be the most appropriate at the beginning of the health interview? a. "Hello, Mrs. Smith. My name is Alice. We'll start with a few questions before the physical exam." b. "Welcome, Sara. I'm Alice. Let's get down to some questions about your health." c. "I'm Alice Jones. I'm here to do an interview about your health." d. "Hey, Mrs. Smith! Are you ready for some questions about your health?"

"Hello, Mrs. Smith. My name is Alice. We'll start with a few questions before the physical exam."

What statement by a patient's wife would indicate that the patient is experiencing myoclonus? a. "His loud snoring and jerking awake wakes me up, too." b. "I am black and blue from his kicking me every night." c. "He wakes up at 2 AM every morning and walks around the house." d. "His constant leg movements tear up the covers and keep me awake."

"I am black and blue from his kicking me every night."

Which of the following is descriptive of normal saline? a. Distilled water b. 0.9% sodium chloride c. Highly acidic d. Contains albumin

0.9% sodium chloride

What is the treatment and lifestyle changes for rosacea?

avoiding triggers such as stressful situations extreme heat, such exposure, spicy foods, and alcoholic beverages. Light and laser treatment

Excessive secretion of ADH, as occurs with head injury, is most likely to cause a. tenting. b. blood volume expansion. c. diuresis. d. hypernatremia.

blood volume expansion

The nurse clarifies that the term __________ refers to the study of how persons respond to medicines.

pharmacodynamics

Semi Fowlers

posture assumed by patient when head of bed is raised approximately 30 degrees

Hyperkalemia refers to an increase in the blood levels of which ion? a. Hydrogen b. Bicarbonate c. Calcium d. Potassium

potassium

Kaliuresis refers to the urinary loss of a. water. b. albumin. c. potassium. d. sodium.

potassium

The patient is taking a diuretic medication everyday to prevent edema and lower his blood pressure. The nurse observes the patient for signs of a decrease in

potassium

Which of the following minerals has the symptom of cardiac dysrhythmias and cardiac arrest?

potassium

a perceived lack of control over a current situation or problem and the person's perception that any action he or she takes will not affect the outcome of the particular situation.

powerlessness

A 70-year-old woman with painful arthritic joints asks the nurse what would be the most beneficial exercise for her based upon her disability. What would be the most appropriate suggestion of the nurse? a. Train with hand weights. b. Walk on a treadmill. c. Do low-impact aerobics. d. Swim.

swim

Which of the following is true of hyperkalemia? a. Also causes blood volume expansion and hypertension b. Refers to an elevation in serum potassium c. Is most often due to impaired respiratory function d. Is due to an accumulation of urea in the blood

refers to an elevation in serum potassium

A change of pH from 7.40 to 7.25 a. is indicative of expanded blood volume. b. is indicative of alkalosis. c. reflects an increase in the amount of H+ in the blood. d. is within normal limits.

reflects an increase in the amount of H+ in the blood`

The older man complains of a hard white patch that has developed on the side of his tongue. What would be the most appropriate action of the nurse? a. Request a dental consult to evaluate his dentures for adequate fit. b. Examine his teeth to assess for a lost filling, which has left sharp edges on his teeth. c. Request a medical consult for evaluation of a precancerous lesion. d. Provide frequent, warm, salt water rinses for his mouth.

request a medical consult for evaluation of a precancerous lesion

as in respite care; allows the primary caregivers to have time away from the constant demands of caregiving, thereby decreasing caregiver stress and the risk for abuse

respite

In G0 the cell is a. resting. b. undergoing mitosis. c. duplicating DNA. d. making mRNA.

resting

What happens to the vision with age related macular degeneration?

restricted but not totally lost, the macula is damaged and central vision is significantly diminished

organic disease

results in a structural change in an organ that interferes with its functioning

To increase safe mobility for a patient recovering from a fractured left hip, the nurse would instruct the patient to place the cane on the __________ side

right

What are patients who have had Parkinson disease for 10 years likely to exhibit? (Select all that apply)

rigidity and tremors when at rest dementia

The nurse admits a 70-year-old female to the long-term care facility. While assisting with the assessment, the nurse notes that the patient's husband died 2 months ago and that she has pain daily in her deformed hands from rheumatoid arthritis, needs assistance to dress herself, and has become incontinent of urine. What is the most appropriate nursing diagnosis for this patient? a. Risk for disturbed self-perception b. Powerlessness c. Hopelessness d. Impaired social reaction

risk for disturbed self-perception

personal hygiene

self care measures that people use to maintain their health

an object, action, or other stimulus that represents something else by conscious association, convention, or another relationship

symbols

as in respirations, an abnormal pattern of respirations characterized by alternating periods of apnea and deep rapid breathing

cheyne stokes

Phosphorus

chiefly an intracellular anion in fluid of the body

subjective data

symptoms, verbal statements provided by the patient. That which arises from within or is perceived by the individual and related to the examiner

What are some neurologic changes associated with aging?

decreased number of brain cells decreased number of nerve fibers decreased number of neuroreceptors decreased peripheral nerve function

How does age affects the older person's taste?

decreased number of papillae tongue decreased number of nasal sensory

What is the cause of progressively graying hair? a. Decreased production of melanin b. Altered blood circulation to the scalp c. Decreased density of hair d. Environmental factors

decreased production of melanin

Which of the following is NOT a sign of iron deficiency anemia?

decreased sensitivity to cold

What should the nurse focus on when creating a nursing care plan for a patient with metabolic acidosis? a. Frequent periods of ambulation b. Increasing fluid intake c. Decreasing fluid intake d. Deep-breathing exercises

deep breathing exercises

Poor skin turgor is most indicative of a. cystitis. b. acidosis. c. edema. d. dehydration.

dehydration

Which of the following is most likely to occur when output exceeds intake? a. Diuresis b. Polyuria c. Hematuria d. Dehydration

dehydration

What is xerosis and how would you implement as a nurse for your patient with xerosis?

dry skin caused by a decrease in the function of sebaceous and sweat gland secretion. Refer to dermatologist

difficult poorly articulated speech resulting from interference in the control and execution over the muscles of speech, usually caused by neourologic damage

dysarthria

What are the signs and symptoms of heart failure?

dyspnea (shortness of breath) with exertion orthopnea (dyspnea at rest when recumbent) coughing or wheezing with exertion or at rest fatigue, weakness, or generalized muscle weakness with minimal exertion peripheral edema weight gain without an increase in food intake (fluid retention) nausea, vomiting, and anorexia paroxysmal noctural dyspnea( extreme orthopnea during sleep)

proposes that errors in ribonuclei acid protein synthesis cause errors to occur in cells in the body, resulting a progressive decline in biolgic function.

error theory

related to the immune system. the immunologic theory of aging process that aging is function of changes in the immune system, which weakens over time to make an aging person more susceptible to disease.

immunologic

What is the focus of aerobic exercises such as walking and biking? a. Improve cardiovascular function b. Build muscle mass c. Improve dexterity d. Enhance balance

improve cardiovascular function

Translation a. occurs within the nucleus. b. involves DNA and tRNA. c. involves tRNA and mRNA. d. refers to the production of a complementary strand of DNA.

involves tRNA and mRNA

An enlarges thyroid gland is related to a deficiency in?

iodine

Which of the following is not true of glycogen? Glycogen a. is a storage form of glucose. b. is an alcohol to which three fatty acids attach, thereby forming a triglyceride. c. helps in the regulation of blood glucose. d. Is stored within the liver and skeletal muscle.

is an alcohol to which three fatty acids attach, thereby forming a triglyceride

Which of the following is true of dehydration? a. Is characterized by poor skin turgor and "tenting" b. Is accompanied by an decrease in urine specific gravity c. Causes a shift of water from the vasculature into the interstitium d. Is generally treated with a diuretic

is characterized by poor skin turgor and "tenting"

DNA a. is double stranded and stores the genetic code. b. contains ribose as its sugar. c. is located within the cytoplasm. d. contains the following bases: cytosine, guanine, adenine, and uracil.

is double stranded and stores the genetic code

Aldosterone a. is released in response to hypovolemia. b. causes Na+ excretion. c. causes K+ reabsorption. d. determines the permeability of the collecting duct to water.

is released in response to hypovolemia

related to decreased supply of oxygenated blood, as in ischemic heart disease, a pathologic condition cause by lack of oxygen in cells of the myocardium

ischemic

When a 75-year-old male resident in an extended-care facility tells the nurse he wants to build up the muscles in his arms, the nurse recommends a(n) __________ exercise program.

isotonic

Why is it important for the nurse to be cautious when using medical jargon with an older adult patient

it could become a communication barrier

Which of the following is true of glycogen? a. It is an end product of fatty acid catabolism. b. It is stored in adipose tissue. c. It is the storage form of glucose. d. It is an essential amino acid.

it is storage form of glucose

What would be the most important suggestion for the nurse to give her 75 year old patient in an attempt to prevent a potential drug interaction?

keep a list of drugs that he is currently taking

What is an expected assessment finding in an older adult suffering from hyperthermia? a. Excessive perspiration b. Bradycardia c. Temperature of 100° F d. Leg cramps

leg cramps

Which of the follwing are signs of low potassium? (select all that apply) A. leg cramps B. palpitations C. high blood pressure D. fluorosis

leg cramps palpitations

vital signs

measurement of temperature,pulse,respiration,and Blood pressure

An 80-year-old extended-care resident comes to the nurse asking for a bandage for a bleeding, dark pigmented mole with irregular shape and border. What does the nurse suspect? a. Melanoma b. Basal cell carcinoma c. Cutaneous papilloma d. Senile lentigo

melanoma

Which of the following accurately reflects a homogenous society? a. Members share a common cultural history b. Members are diverse in role expectation c. Members have conflicted role status d. Members may choose or change role performance

members share a common cultural history

the mental facility or power that enables one to retain and to recall, through unconscious associative processes previously experienced sensations impressions ideas and concepts

memory

What is the role of mRNA?

messenger Copies codes from DNA in the nucleus and them carries the code or message to the ribosomes in the cytoplasm

Where does most of the body's acid come from? a. It is reabsorbed from the urine by the kidneys. b. Metabolic reactions within the body c. Ingestion of fruit juices d. Ingestion of a variety of sources of bicarbonate

metabolic reactions within the body

as in drug metabolism, the transformation of a drug by the body tissues into a metabolite as the boy readies the agent to elimination

metabolism

Which of the following is true of urea? Urea is a. nitrogen containing. b. produced by hepatic cells. c. eliminated in the urine. d. All of the above are true.

all of the above are true

What does anabolism require to occur?

an input of energy in the form of adenosine triphosphate(ATP)

How would you describe seborrheic dermatitis? what is the cause?

an unsightly skin condition characterized by yellow, waxy crusts that can be either dry or moist. The cause is excessive sebum production, seborrheic dermatitis can occur on the scalp, eyebrow,s eyelids, ears, axilla, breast, groin, and gluteal folds

A fit, young woman was at zero nitrogen balance. The nurse discovers that this patient is now pregnant with her first child. For what is this patient at risk?

anabolism

What is glycolysis?

anaerobic process that breaks down glucose into lactic acid

canthus

angle at the medial and lateral margins of the eyelid

fowlers

when the head of the bed is raised 45 to 60 degrees

vasoconstriction

when the lumen of blood vessels narrows thus hindering blood flow and resulting in less edema

vasodilation

when the lumen of blood vessels widens thus increasing the blood flow.

wheezing

adventitous breath sounds that have a whistling or signing sound resulting from narrowing of the lumen of a respiratory passageway

a harmful or unpleasant reaction to medication

adverse drug reaction

the degree of worth and competence one attributes to oneself.

self-esteem

vertigo

sensation that the outer world is revolving about one self or that one is moving space

Cow's milk should not be given to a child until:

12 months of age

What is the best method of communication for a patient with aphasia?

Gestures

;-0

;-0

Hypervitaminosis occurs almost exclusively from supplement use

True

jaundice

`A yellow tinge to the skin that may indicate obstruction in the flow of bile from the liver is known as ___________________.

What is the name for Vitamin C?

ascorbic acid

professionals who are required by law to report reasonable suspicions of abuse

mandated reporter

pulse

rhythmic beating or vibrating movement

When the medication nurse offers a pill to the older adult patient, the patient asks, "What is this and what is it for?" What is the best response by the nurse?

"It's an antibiotic for the infection in your urine"

When the 70 year old female patients says, "Keeping up with when to take the flu vaccine is a big hassle. Im not going to add trying to keep up with a pneumonia vaccine as well. It's too expensive." Which response by the nurse would be the most helpful?

"The pneumonia vaccine needs to be repeated every 10 ."

What factors can contribute to sleep apnea? (Select all that apply.) a. Obesity b. Diabetes c. Hypotension d. African American heritage e. Use of alcohol

-Obesity -Diabetes -African American heritage -Use of alcohol

What changes in sleep patterns are often seen in older adults? (Select all that apply.) a. Inability to sleep throughout the night b. Sleeping soundly all night c. Increase in the number of hours asleep at night d. Difficulty in arousing from deep sleep e. Waking up early

-inability to sleep throughout the night -Waking up early

Which of the following foods are high in potassium? A. bananas B. whole grain cereals C. Milk D. all of the above

All of the above

How would a nurse treat a pt that is hyperventilating or anxious

Breath inhale and exhale through paper bag Rasing carbon dioxide levels in the blood

Electrolytes are not measured by weight; their chemical activity is expressed in milliequivalents. What does 1 milliequivalent of potassium have the same combining power as? a. 1 mEq of nitrogen b. 1 mEq of oxygen c. 1 mEq of hydrogen d. 1 mEq of magnesium

1 mEq of hydrogen

What are the three ways proteins are used?

1.Synthesis of hormones, enzymes, antibodies, plasma and muscle proteins, hemoglobins , and most cell membranes 2. If needed can be broken down as source of energy for ATP production 3. If needed can be broken down and converted to glucose (gluconeogenesis)

The same friend asks how long humans can live. What is the nurse's best reply ?

110 YEARS

Which patient would have the highest risk of suicide? a. 85-year-old male with a history of an affective disorder b. 50-year-old female with a recent divorce c. 70-year-old female with high blood pressure d. 65-year-old male with chronic asthma

85-year-old male with a history of an affective disorder

1. The nurse is aware that sexuality: a. becomes absent with age. b. remains part of life until death. c. as expressed through intercourse is not possible after the age of 65 years. d. must be expressed in sexual intercourse. ANS: B Sexuality is an integral part of the makeup of each person. Although sexual expression may change, the need for sexual affection does not diminish. Intercourse is only one method of sexual expression. DIF: Cognitive Level: Knowledge REF: 259 OBJ: 1 TOP: Sexuality KEY: Nursing Process Step: Assessment MSC: NCLEX: Physiological Integrity: Physiological Adaptation 2. The nurse counsels the 70-year-old female who has remained on hormone replacement therapy (HRT) that she needs to have a: a. semiweekly douche to wash out cervical debris. b. liver function assessment annually. c. mammogram biannually. d. Pap smear annually. ANS: D Persons on HRT should have an annual Pap smear and mammogram. DIF: Cognitive Level: Application REF: 259 OBJ: 1 TOP: Hormone Replacement Therapy KEY: Nursing Process Step: Implementation MSC: NCLEX: Health Promotion and Maintenance: Prevention and Early Detection of Disease 3. The nurse evaluates a need for further instruction to reduce the symptoms of vaginal dryness when the 70-year-old patient says: a. "Vaseline was good enough for my mother. It's good enough for me." b. "I use a water-soluble lubricant to aid intercourse." c. "I'm trying an estrogen cream to see if it works." d. "I'll let you know how wild yams work for vaginal dryness." ANS: A Vaseline and petroleum products should be avoided because they do nothing for long-term dryness. DIF: Cognitive Level: Application REF: 260 OBJ: 4 TOP: Vaginal Dryness KEY: Nursing Process Step: Evaluation MSC: NCLEX: Physiological Integrity: Pharmacological Therapies 4. The nurse identifies the person most likely to experience erectile dysfunction as the 65-year-old who has _____ sexually active in earlier years. a. diabetes and was very b. irritable bowel syndrome and was minimally c. chronic pancreatitis and was very d. osteoarthritis and was moderately ANS: A Diabetic men are at risk for erectile dysfunction even at young ages. DIF: Cognitive Level: Analysis REF: 260 OBJ: 4 TOP: Erectile Dysfunction KEY: Nursing Process Step: Assessment MSC: NCLEX: Physiological Integrity: Physiological Adaptation 5. The 65-year-old male who, although he is having painful symptoms related to a benign enlargement of the prostate, refuses to consider a prostatectomy because he fears that the surgery will make him impotent. The nurse reassures him that: a. a prostatectomy will enhance sexual function. b. new techniques for a prostatectomy do not damage nerves. c. the prostate has nothing to do with erection or seminal fluid production. d. impotence following a prostatectomy is entirely psychological. ANS: B Newer techniques do not damage nerves as was once the case when impotence following surgery was a real risk. DIF: Cognitive Level: Application REF: 260 OBJ: 1 TOP: Erectile Dysfunction KEY: Nursing Process Step: Implementation MSC: NCLEX: Physiological Integrity: Physiological Adaptation 6. The 70-year-old woman who is considering coming off of hormone replacement therapy (HRT) for the reduction of postmenopausal discomfort asks the nurse what advantage is offered by tamoxifen, also called a "designer estrogen." The nurse's best response is that tamoxifen: a. provides estrogen to some tissues while acting as an antiestrogen to others. b. has no side effects. c. needs to be taken only once a week. d. improves the skin turgor and complexion. ANS: A "Designer estrogens" selectively act as an estrogen on some tissues and an antiestrogen on others. There are side effects, and the drug is usually taken twice a day. DIF: Cognitive Level: Analysis REF: 260 OBJ: 1 TOP: Tamoxifen KEY: Nursing Process Step: Implementation MSC: NCLEX: Physiological Integrity: Pharmacological Therapies 7. The nurse teaches that some persons have found relief from postmenopausal discomfort by using phytoestrogens, which act as estrogens on some tissue and antiestrogens on others. Phytoestrogens are found in: a. mangos. b. soybeans. c. broccoli. d. lima beans. ANS: B Soybeans are rich in phytoestrogens. DIF: Cognitive Level: Comprehension REF: 260, Complementary and Alternative Therapies OBJ: 1 TOP: Phytoestrogens KEY: Nursing Process Step: Implementation MSC: NCLEX: Physiological Integrity: Pharmacological Therapies 8. The nurse recognizes a need for further instruction about sexual activity when the 65-year-old man who had a myocardial infarction 6 months ago says: a. "I'm enjoying the same sexual activities now as I did before my heart attack." b. "I'm still pretty cautious, but our sex lives are very satisfactory." c. "I've been told that I'm at risk for another heart attack if we have sex." d. "My heart medications have made me impotent, but we've found other methods of sexual expression." ANS: C Fear, rather than an actual threat, can cause cessation of sexual relations following a heart attack. DIF: Cognitive Level: Application REF: 260 OBJ: 2 TOP: Illness and Sexual Function KEY: Nursing Process Step: Evaluation MSC: NCLEX: Physiological Integrity: Physiological Adaptation 9. The nurse makes it clear to older adults in a long-term care facility that condoms are available from the medicine cart on request to: a. guarantee safe sex practices. b. reduce the incidence of sexually transmitted diseases (STDs). c. show acceptance of sexual expression. d. prevent soiling bed linens or furniture. ANS: B STDs can still be transmitted in the older adult. Studies have shown that 10% of persons older than 50 years have been exposed to the human immunodeficiency virus (HIV). Nothing can guarantee safe sex practices. DIF: Cognitive Level: Application REF: 261-262 OBJ: 6 TOP: STDs and the Older Adult KEY: Nursing Process Step: Planning MSC: NCLEX: Health Promotion and Maintenance: Prevention and Early Detection of Disease 10. The home health nurse stresses to the 70-year-old gay man who has been in a monogamous relationship for 20 years that it is especially important to name his partner as his medical power of attorney (POA) and file advance directives because: a. all persons older than 60 years should have a medical POA and advance directives. b. gay and lesbian couples are estranged from their families. c. life partners frequently abandon unions when terminal illness occurs. d. life partners have no legal standing and can be prohibited from medical decisions by family. ANS: D Life partners have no legal standing for medical decisions unless named as medical POA. DIF: Cognitive Level: Application REF: 262 OBJ: 5 TOP: Sexual Orientation KEY: Nursing Process Step: Implementation MSC: NCLEX: Psychosocial Integrity: Coping and Adaptation 11. The horrified CNA runs to the nursing station and blurts out, "Do you know what Mr. and Mrs. Smith are doing? They're having sex right there in room 210!" The nurse's best response would be to: a. knock discreetly on the door and enter the room. b. call the charge nurse to report inappropriate sexual activity. c. request an order from the physician to allow intercourse. d. give the CNA a "Do Not Disturb" sign to put on the door of room 210. ANS: D Privacy should be protected for appropriate sexual expression. Orders and notifying the charge nurse are not necessary, but recording the event in the nursing notes as a conjugal visit is suggested. DIF: Cognitive Level: Application REF: 262-263 OBJ: 7 TOP: Privacy KEY: Nursing Process Step: Implementation MSC: NCLEX: Psychosocial Integrity: Psychosocial Adaptation 12. When the cognitively impaired man is openly masturbating in the dayroom of a long-term care facility, the nurse's best response would be to say: a. "This is inappropriate behavior in public areas." b. "Oh my God! Don't you know that will make you go blind?" c. "I think you may need some privacy" and take him to his room. d. "Here's a blanket for your legs" and cover his lap with it. ANS: C A person who is masturbating in public should be conducted to the privacy of his or her room. DIF: Cognitive Level: Application REF: 263 OBJ: 7 TOP: Masturbation KEY: Nursing Process Step: Implementation MSC: NCLEX: Psychosocial Integrity: Psychosocial Adaptation 13. When a married couple is admitted to the long-term care facility, the admitting nurse should show willingness to discuss sexual matters by asking: a. "How often do you have intercourse?" b. "Do you have any sexual concerns related to your admission here?" c. "What can we do to keep your sex life active?" d. "Any sexual activity should be done in private." ANS: B Asking about sexuality in broad terms communicates a willingness to discuss the issue. DIF: Cognitive Level: Application REF: 261 OBJ: 7 TOP: Sexual Concerns KEY: Nursing Process Step: Implementation MSC: NCLEX: Psychosocial Integrity: Coping and Adaptation 14. When the CNA tearfully reports that a resident deliberately exposes himself to her and makes lewd suggestions, the nurse could best deal with this behavior by modifying the nursing care plan to: a. assign only males to care for the resident. b. chastise the resident for lewd behavior. c. restrain the hands of the resident during routine care. d. modify clothing choices to one-piece garments or elastic waist trousers. ANS: D Modification of the clothing to make exposure more difficult is the best initial approach to the problem. DIF: Cognitive Level: Application REF: 263 OBJ: 7 TOP: Inappropriate Genital Exposure KEY: Nursing Process Step: Planning MSC: NCLEX: Psychosocial Integrity: Psychosocial Adaptation MULTIPLE RESPONSE 1. The nurse discusses factors that alter the sexual practice of older adults, which are __________. (Select all that apply.) a. altered health status b. inability to achieve orgasm c. erectile dysfunction d. loss of sexual partner e. slowed sexual response time ANS: A, C, D, E Although the response is slowed, the ability to achieve orgasm remains throughout life. DIF: Cognitive Level: Comprehension REF: 259 OBJ: 1 TOP: Age-Related Sexual Impairment KEY: Nursing Process Step: Implementation MSC: NCLEX: Physiological Integrity: Physiological Adaptation 2. The nurse lists the age-related changes in the female reproductive system that affect sexual intercourse, which are __________. (Select all that apply.) a. pruritus vulvae b. atrophic vaginitis c. frequent yeast infections d. dyspareunia e. decreased response time ANS: A, B, C, D The response time is not decreased. All of the other options are age-related changes in the female that affect satisfactory intercourse. DIF: Cognitive Level: Application REF: 259 OBJ: 1 TOP: Age-Related Impediments to Intercourse KEY: Nursing Process Step: Implementation MSC: NCLEX: Physiological Integrity: Physiological Adaptation 3. The nurse is aware that many women have abandoned hormone replacement therapy (HRT) because of the risk of __________. (Select all that apply.) a. stroke b. myocardial infarction c. liver disorders d. cancer e. bone demineralization ANS: A, B, C, D Bone demineralization is not a risk associated with HRT, which is prescribed to prevent bone demineralization. DIF: Cognitive Level: Application REF: 259 OBJ: 1 TOP: Hormone Replacement Therapy Risks KEY: Nursing Process Step: Planning MSC: NCLEX: Physiological Integrity: Pharmacological Therapies 4. The nurse lists the sexual changes experienced by the older man, which are impairments to intercourse, such as __________. (Select all that apply.) a. erections take longer to achieve and are less firm than in younger years b. orgasm takes longer to achieve c. orgasm occurs at each intercourse d. erection loss occurs immediately after orgasm e. an increase in time between orgasms ANS: A, B, D, E Orgasms are not always achieved with each intercourse. DIF: Cognitive Level: Application REF: 260 OBJ: 1 TOP: Sexual Impairment KEY: Nursing Process Step: Implementation MSC: NCLEX: Physiological Integrity: Physiological Adaptation 5. The nurse screening patients who complain of erectile dysfunction is aware that many drugs cause this problem, such as __________. (Select all that apply.) a. digitalis b. pancreatic enzyme replacement c. antihistamines d. antiparkinson drugs e. antidepressants ANS: A, C, E Digitalis, antihistamines, and antidepressants all have a negative effect on sexual function. DIF: Cognitive Level: Application REF: 261, Table 16-1 OBJ: 2 TOP: Medication and Sexual Function KEY: Nursing Process Step: Assessment MSC: NCLEX: Physiological Integrity: Pharmacological Therapies 6. The nurse is aware that many constraints prevent an older widow from engaging in sexual activity, which are that __________. (Select all that apply.) a. older women outnumber older adult men by 4:1 b. there is an expectation that men should initiate contact c. display of sexual interest is considered improper d. older women do not desire sexual expression e. older women associate sexual activity with marriage ANS: A, B, C, E Older women do retain a sexual appetite. DIF: Cognitive Level: Application REF: 261 OBJ: 4 TOP: Loss of Sexual Partner KEY: Nursing Process Step: Assessment MSC: NCLEX: Psychosocial Integrity: Psychosocial Adaptation 7. Marriage between older adults is frequently impeded by __________. (Select all that apply.) a. disapproval of children b. financial loss in case of remarriage c. concerns about chronic illness with either partner d. the lack of opportunity for sexual expression outside of marriage e. religious and personal values that inhibit cohabitation ANS: A, B, C Lack of opportunity for sexual expression outside of marriage and distaste for cohabitation are factors that might encourage marriage rather than impede it. DIF: Cognitive Level: Analysis REF: 261 OBJ: 1 TOP: Marriage and the Older Adult KEY: Nursing Process Step: N/A MSC: NCLEX: N/A 8. The home health nurse takes into consideration the concerns of aging gay and lesbian couples, which include __________. (Select all that apply.) a. health care providers being willing to discuss a homosexual union b. the health care system's focus on heterosexuality as the norm c. difficulty finding senior housing as a homosexual couple d. fear of discrimination based on sexual orientation, resulting in substandard health care e. delay of entry to a long-term care facility because of the possibility of not being able to share a room ANS: B, C, D, E Health care providers are reluctant to discuss homosexual unions. All other options are possible problems related to the health care of older gay and lesbian couples. DIF: Cognitive Level: Comprehension REF: 262 OBJ: 5 TOP: Sexual Orientation and Health Care KEY: Nursing Process Step: Planning MSC: NCLEX: Psychosocial Integrity: Coping and Adaptation 9. While asking questions about the patient's reproductive system, the nurse is aware that sexual function can be affected by __________. (Select all that apply.) a. increasing age b. grief c. illness d. religion e. physical changes ANS: A, B, C, E Religion does not affect sexual function. All other options can affect the individual physically and result in sexual dysfunction. DIF: Cognitive Level: Comprehension REF: 259 OBJ: 1 TOP: Age-Related Sexual Impairment KEY: Nursing Process Step: N/A MSC: NCLEX: Physiological Integrity: Physiological Adaptation 10. Questions that the nurse might ask to assess sexual function include __________. (Select all that apply.) a. "How many alcoholic beverages do you drink a day?" b. "What over-the-counter and prescription medications do you take?" c. "How often do you engage in sexual activity?" d. "Have you had any recent health problems?" e. "When do you see your significant other?" ANS: A, B, C, D Asking when the individual sees the significant other does not necessarily indicate that the individual is sexually active. Sexual function can be affected by excessive alcohol intake, medications, availability of sexual partners, and illness. DIF: Cognitive Level: Application REF: 260-261 OBJ: 3 TOP: Age-Related Sexual Impairment KEY: Nursing Process Step: Assessment MSC: NCLEX: Physiological Integrity: Physiological Adaptation

;-0

Which of the following tasks could not be performed by an unlicensed home health aide?

Measure or dispense medication

What happens when the older adult has a decrease in subcutaneous tissue?

Reduces the older adults ability to regulate body temperature

An older adult has recently been placed in a long-term care facility. What would the nurse expect to see exhibited in the patient? a. Feeling unloved and unlovable b. Feeling angry with the isolation c. Feeling unworthy for attention d. Feeling determined to be his or her own support

A

melena

A farm worker who has been kicked in the stomach by a mule passes a foul, black, tarry stool. What is this called? a. Loose stool b. Melena c. Hematuria d. Hemoptysis

Which of the following is descriptive of mitochondrial function? a. Lactic acid production b. Glycolysis c. Aerobic catabolism d. Protein synthesis

aerobic catabolism

malignant epithelial neoplasm that tends to invade surrounding tissue and to metasize to distant regions

carcinoma

What can caregivers do for a dying patient who suffers from diminished vision? a. Keep lights bright to increase visual acuity. b. Stand slightly away from the bed and identify themselves before speaking. c. Keep all lights on in the room, day and night. d. Come close to the bed and stand directly in front of the patient.

Come close to the bed and stand directly in front of the patient

All of the following are true regarding the diet of an infant EXCEPT:

low fat or skim milk should be given when taken off breast milk or formula

splint the arm

A visitor in the hospital slips and falls. The patient's arm appears dislocated and the visitor is unable to move it. What is the first action the nurse should implement? a. Apply cold packs b. Check range of motion c. Splint the arm d. Apply an Ace bandage

What is urea?

A waste product formed by the liver and excreted by the kidneys

dullness

low pitched thudlike sound on percussion of the body.

The nurse explains that a normal adult will lose approximately _____ mL of water through respiration in the course of a day.

350

endorphins

A young athlete asks the nurse why he felt little pain when he broke his leg during a game. What does the nurse describe as having an effect on this patient's perception of pain? a. Hormones b. Enzymes c. Adrenaline d. Endorphins

Fat -soluble vitamins

A,D,E,K,

prone

lying face down in horizontal position

prone

lying face down on the abdomen

dorsal (supine)

lying horizontally on the back

On what day should an outing to the park for a group of older adults be canceled? a. 75-degree sunny day in Texas b. 70-degree cloudy day in Oregon c. 80-degree sunny day in Florida d. 75-degree ozone alert day in California

75 degree ozone alert day in california

The long-term care facility nurse is aware that the resident most at risk for suicide related to depression is the a. 70-year-old man with diabetes. b. 75-year-old woman with chronic obstructive pulmonary disease. c. 80-year-old woman with a bipolar disorder. d. 85-year-old man with schizophrenia.

80-year-old woman with a bipolar disorder

The nurse reminds the CNAs that most older adults require a minimum of _____ hours of sleep per day.

7.5

What should lactating women increase their caloric intake to?

500kcal/day

What factors might interfere with health maintenance activities of a 76 year old patient?

A belief that diminished health is part of aging Loss of a spouse 3 months ago Having no family

27

A burn patient is brought into the emergency department with the following burns: half of the front torso, entire left arm, and front of left leg. The nurse should record that the patient has a ______% burn.

bradycardia

A circulatory condition in which the myocardium contracts steadily but at a rate of less than 60 contractions per minute is known as _________________.

cyanosis

A condition in which a patient experiences bluish discoloration of the skin and mucous membranes caused by an increase of deoxygenated hemoglobin in the blood is known as _________________.

Anorexia

A condition in which there is a lack of appetite resulting in the inability to eat is known as _______________.

Syncope

A condition in which there is a temporary loss of consciousness associated with an increased rate of respiration, tachycardia, pallor, perspiration, and coolness of the skin is known as _________________.

asthenia

A condition of debility, loss of strength and energy, and depleted vitality is known as _________________.

purulent drainage

A creamy, viscous, pale yellow or yellow-green fluid exudate that is the result of fluid remains of liquefied necrosis of tissues is known as ___________ ___________.

The nurse performs a skin assessment of an older adult. Which finding is abnormal and needs to be reported?

A dark, elevated patch that bleeds when touched

Internal

A disaster situation occurs and involves an explosion in a hospital laundry. What would this be classified as ? a. Active b. External c. Life-threatening d. Internal

Patient controlled analgesia (PCA)

A drug delivery system that dispenses a present intravenous dose of opioid analgesic into a patient vein when the patient pushes a switch on an electrical cord

sims position

lying on left side and right knee and thigh drawn upward to chest

Which of the following represents base pairing? a. CCGTTACTG b. Sugar-phosphate, sugar-phosphate, sugar-phosphate c. Ribose-deoxyribose, ribose-deoxyribose, ribose-deoxyribose d. A-T, T-A, C-G, A-U

A-T, T-A, C-G, A-U

Which of the following represents a base sequence of tRNA that will bind to mRNA UUU? a. UUU b. AAA c. TTT d. CCC

AAA

Which nucleotide carries the individual amino acids from the cytoplasm to the ribosome for assembly into a protein?

mRNA

What happens if a person has a deficiency of sodium?

Cramps, mental confusion, apathy, appetite loss (usually secondary to diarrhea or disease) increased risk of osteoporosis and MI

The reaction of -COOH and -NH2 a. forms urea. b. creates a peptide bond. c. forms glycogen. d. forms a ketone body.

Creates a peptide bond

What is the name for vitamin B12?

Cyanocobalamin

.What is a cardiovascular sign of impending death? a. Cheyne-Stokes respiration b. Bounding pulse c. Cyanosis of the extremities d. Increased blood pressure

Cyanosis of the extremities

What is the two-stranded double helix twisted ladder formation of nucleotides that hold the genetic code?

DNA

Which of the following is a double-stranded nucleic acid that is called the blueprint for life? a. Urea b. DNA c. Gene d. RNA

DNA

mRNA "gets" the genetic code from a. DNA. b. tRNA. c. the ribosomes on the ER. d. centrioles.

DNA

New research has shown that individuals with a low Vitamin D level tend to have which of the following?

Decaying teeth

Constipation can occur in elderly adults due to all of these factors EXCEPT:

Decreased kilocalorie needs and vitamin requirements

What is the toxic effect of ammonia on the brain?

Hepatic encephalopathy

To simplify food values, the measurement of energy obtained by food is defined as the ________.

Kilocalorie

Signs of dehydration include:

Poor skin turgor

What is the most abundant electrolye in the body?

Sodium

How does the body get rid of urea?

Urine

What is the function of the 5-carbon monosaccharides?

Used in the synthesis of RNA and DNA

Which lipoprotein accounts for almost all the triglycerides in the blood?

VLDLs (very low density lipoprotein)

inspection

Visual examination of the external surface of the body and its movement and posture including observation of moods and all responses and nonverbal behaviors

Promotes growth, sustains normal vision, maintain healthy skin and mucous membranes

Vitamin A

Which of the following would be considered positive outcomes of Erikson's "life review"? (Select all that apply.) a. Wisdom and integrated self-image b. Comparing self with others c. Understanding self and relationships d. Seeking another's opinion of his or her achievement e. Acceptance of self

Wisdom and integrated self- image Understanding self and relationships Acceptance of self

The nurse is volunteering at a homeless shelter. What intervention should be taken for a patient admitted with severe hypothermia? a. Give the person hot coffee or soup. b. Place the person in a warm bath. c. Briskly rub the person's hands. d. Wrap the person in blankets.

Wrap the person in blankets

Terrorism

___________ is a violent or dangerous act used to intimidate or coerce a person or government to further a political or social agenda.

Cultural beliefs

_______________ _________and personal characteristics determine health behavior in individuals and families. More than half of all health problems are the result of behavior and lifestyle.

Hyperventilation is most likely to cause a. a blood pH higher than 7.45. b. diuresis. c. edema. d. acidosis.

a blood pH higher than 7.45

physical disuse

a state in which an individual is at risk for deterioration of body systems due to prescribed or unavoidable inactivity

ostomy

artificial opening

dentures

artificial teeth not permanently fixed or implanted

a person's concept of his or her physical appearance

body image

pathogenic

capable of causing disease

RACE

formula to enable nurses to be prepared when safety is threatened by fire.

ileostomy

opening in the ileum

Hazard communication act

requires hospitals to inform employees about the presence of or potential risk of exposures

a socially accepted behavior pattern

role

Define gluconeogenesis

the making of glucose from a nonglucose sources especiallh protein

axilla

underarm area or armpit

What is the status of any role based upon? a. Age b. Responsibility c. Society d. Health

Society

Retention of which ion causes water retention and edema formation? a. Potassium b. Hydrogen c. Sodium d. Bicarbonate

Sodium

Which of the following is the chief extracellular cation? a. Chloride b. Calcium c. Sodium d. Potassium

Sodium

Orthodox Jewish

Some _____________ patients consider sundown Friday to sundown Saturday to be the Sabbath, which is a time of rest.

Pancreas

The nurse is discussing the origin of diabetes with a diabetic patient. What will the nurse discuss as the most appropriate explanation for the cause of this disease? a. Pituitary b. Adrenals c. Pancreas d. Thyroid

gate control

The nurse is giving a backrub to a patient to relieve pain. What pain theory is the nurse using? a. Synergism b. Gate control c. Distraction d. Guided imagery

Sacral area

Where should a nurse performing a backrub begin? a. Shoulder b. Base of the neck c. Sacral area d. Lumbar area

What is the kcal/g for carbohydrates?

4 kcal/g

Which of the following lab levels is indicative of hyperkalemia?

6.0

What is the optimal weight gain for an average pregnant woman ?

25-35lbs

unrelieved pain can cause setbacks

Why should a nurse promptly administer a prescribed analgesic after a pain assessment? a. The physician has ordered it b. It is an efficient use of time c. Unrelieved pain can cause setbacks d. It meets the goals of the nursing care plan

Which of the following is most likely to cause acidosis? a. Persistent vomiting b. Accumulation of plasma H+ c. Renal loss of albumin d. Diuresis

Accumulation of plasma H+

Which of the following can be defined as an H+ donor? a. Ion b. Tincture c. Acid d. Bicarbonate

Acid

Synergistic

Action of two or more substances or organs to achieve an effect of which each is capable

autologus blood transfusion

Something that has its origin within its individual especially a factor present in tissues or fluids.

What does the presence of a caregiver provide for the older adult living at home?

Source of motivation

What stage of sleep is the initial phase of deep sleep? a. Stage 1 nonrapid eye movement (NREM) b. Stage 3 NREM c. Stage 5 NREM d. Rapid eye movement (REM) sleep

Stage 3 NREM

In what stage of sleep is sleepwalking most likely to occur? a. Stage 1 NREM b. Stage 2 NREM c. Stage 4 NREM d. REM sleep

Stage 4 NREM

What drugs are used to reduce LDLs?

Statin

subjective data

Symptoms that are perceived by the patient are known as _____________ ____________.

What age-related illnesses are thought to cause the accumulation of free radicals? (Select all that apply.) a. Arthritis b. Colon cancer c. Osteoporosis d. Diabetes e. Atherosclerosis

Arthritis Diabetes Atherosclerosis

Which of the following diseases is not a disease associated with diets in America that are believed to be associated with dietary excesses and imbalances

Asthma

What type of abuse is demonstrated by restraining an older adult in a recliner? a. Physical abuse b. Neglect c. Emotional abuse d. Self-neglect

Physical Abuse

peroxide or alcohol

Because of its effect on epithelization, the LPN/LVN should confirm the order to use ____________ or _____________ on a stage III pressure ulcer.

disorientation

Because of the stress caused by hospitalization, the nurse assesses a newly admitted older adult patient for ________________________

the patient understands the discharge instructions

Before the actual discharge occurs, what must the nurse ensure?

What is the average adult fluid intake?

Between 2200-2700 (2500)

What is the name for vitamin B7?

Biotin

Which ion determines the resting membrane potential of nerve and muscle? a. Sodium b. Calcium c. Hydrogen d. Potassium

Potassium

A patient diagnosed with renal failure is unable to excrete protein waste products and develops a condition that requires a protein-restricted diet. The nurse instructs the patient that azotemia can be diminished by substituting other food groups for protein. What is an example of a food that this patient can substitute for protein?

Potatoes

What fact explains the shift of health care focus toward the older adult in the late 1960s? a. Disability was viewed as unavoidable. b. Complications from disease increased mortality. c. Older adults' needs are similar to those of all adults. d. Preventive health care practices increased longevity.

Preventive health care practices increased longevity

EMERGENCY MEDICAL SERVICES (EMS)

National network of services coordinated to provide aid and medical assistance from primary response to definitive care, involving personal trained in the rescue stabilization, transportation and advanced treatment of traumatic or medical emergencies

What is the toxicity of Vitamin B3?

Nausea, vomiting, flushing, pruritus (itching) of the skin, and abnormal liver function

What would be considered the most important indicator of a positive self-image in an older adult living in a long-term care facility? a. Feeding self independently b. Maintaining urinary continence c. Having family visitors every week d. Neat grooming and wearing fresh clothing

Neat grooming and wearing fresh clothing

What is the name for vitamin b3?

Niacin

If a patient BMI is between 18.5 & 24.9?

Normal range

What is the most frequent response to elder abuse by the abused older adult? a. Anger b. Physical retaliation c. Notification of authorities d. Nothing at all

Nothing at all

pain

The nurse assesses for the fifth vital sign, which is______________.

NSAIDS

The nurse clarifies that the term peripheral analgesics describes the group of drugs also referred to as ___________.

continuity, care

The nurse completes thorough documentation before, during, and after a transfer to ensure ________________ of __________________

What behavior would indicate that the bereaved widow has entered the searching and yearning stage of grief? a. Renewed interest in social activities b. Signs of depression c. Making plans for the future d. Denial of the loss of her husband

signs of depression

a condition in which a feeling of aloness is experienced which the person acknowledges as a negative or threatening state imposed by others

social isolation

korotkoff sound

sounds heard while measuring Blood pressure when using a sphygomomanometer &stethoscope

What can the nurse do to improve communication with an older adult patient who has a hearing impairment?

speak in a low tone, bending close to the patient

respiration

taking in of O2 its use in the tissues and giving off of O2

The older man in a long-term care facility consistently wakes at 3 AM and does not return to sleep. The nurse records this behavior as _____ insomnia. a. sleep initiation b. sleep maintenance c. terminal d. undifferentiated

terminal

active transport

the movement of materials through a cell membrane of a cell to admit larger molecules than would otherwise be able to enter

cerumen

yellowish or brownish waxy secretion produced by vestigal apocrine sweat glands in the external ear canal

What condition is frequently caused by hyperventilation

Respiratory alkalosis

The person acts prudently

The Good Samaritan law will protect all people who offer assistance. What is necessary for this protection? a. A license b. The person acts prudently c. Licensed supervision d. The patient improves

head of bed is at a 30-degree angle

The LPN/LVN assists a patient into the semi-Fowler position per physician order. What would indicate that this patient is in the correct position? a. Patient is leaning over the bedside table b. Head of bed is at a 30-degree angle c. Knee is drawn toward the chest d. Arms are flexed toward the head

compression

The acronym RICE directs the nurse in the care of a sprain. The "C" in the acronym stands for ________.

mobility

The nurse is educating a patient on ways to regain the ability to perform ADLs and maintain normal physiological activities. What will the nurse relay as a requirement? a. Strength b. Wellness c. Alertness d. Mobility

10 to 20 minutes

The nurse is educating a patient regarding a tub bath. What is the maximum length of time the nurse should instruct the patient to remain in the water? a. 5 to 10 minutes b. 10 to 20 minutes c. 20 to 30 minutes d. 30 to 40 minutes

Pharynx

The nurse is obtaining a throat culture. What area will the nurse swab with a cotton-tipped applicator?

Your older female patient is complaining because she is having frequent urinary tract infections. Which normal age-related change is most likely to be a contributing factor?

decreased bladder muscle tone

A drug that decreases the synthesis of aldosterone a. decreases blood pressure. b. causes hypernatremia. c. causes hypokalemia. d. does all of the above.

decreased blood pressure

What age-related changes in the stomach can cause increased drug absorption and possibly drug toxicity?

decreased gastric motility

The nurse encourages the patient to maintain a steady weigh in the recommended range to decrease risk of which common endocrine disease observed in older adults?

diabetes mellitus

The nurse weighs a patient at the same time of day with the same scale and same clothing. What is this a simple and accurate method of determining? a. An accurate weight b. Water balance c. Adequate nutrition d. Urinary output

Water balance

What is the definition of osmosis

Water move from high to low concentration

What is the function of cellulose?

it is the structural component of plant cell walls; straight chain polysaccharides found in plants

level of consciousness (LOC)

level of orietntation

The nursing health history

What does the nurse recognize as the initial step in conducting an assessment of a patient? a. A body systems review b. The nursing health history c. Biographical data d. The present illness

disorder characterized by porous, brittle fragile bones that are susceptible to breakage caused by excessive loss of calcium from bone combined with insufficient replacement

osteoporosis

Transcription produces a. tRNA. b. a complementary strand of DNA. c. deoxyribose. d. mRNA.

mRNA

tRNA reads the genetic code stored in a. DNA. b. ribosomes. c. mRNA. d. nuclear membrane.

mRNA

Translation involves a. ribose and deoxyribose. b. thymine and uracil. c. DNA and mRNA. d. mRNA and tRNA.

mRNA and tRNA

The nurse explains that the Minimum Data Set (MDS) 3.0 used in extended-care facilities is designed to a. identify ethnic populations in long-term care. b. group residents into specified activity levels. c. organize information relative to diagnostic categories. d. make assessment processes more consistent.

make assessment processes more consistent

Which of the following is characteristic of acidosis? a. pH of 7.55 b. A decrease in the concentration of hydrogen ion in the blood c. Urinary excretion of bicarbonate d. pH less than 7.35

pH less than 7.35

What activity would be the most difficult for an older woman who has diminished dexterity? a. Working a crossword puzzle b. Playing a round of golf c. Taking a walk d. Painting with watercolors

painting with watercolors

as in palliative treatment, therapy designed to relieve or reduce intensity of uncomfortable symptoms but not to produce a cure, such as narcotics to relieve pain

palliative

a method of physical assessment that uses the sense of touch in the fingers and hands to obtain data

palpation

ova and parasites

What type of stool specimen must be sent to the laboratory immediately?

mobility

a persons ability to move around freely in his or her environment

language problem

dysphasia

As in a heterogeneous society a more complex one in which the members of many diverse cultures with different historical and cultural experiences interact.

heterogeneous

a condition in which part of the stomach pushes up through the diaphragm muscle

hiatal hernia

a state in which an individual sees limited or no alternatives in personal choices available and is unable to mobilize energy on his or her own behalf

hopelessness

supine

horizontal position lying face upward

a multidisciplinary system of family centered care designed to assist the terminally ill person to be comfortable and to maintain a satisfactory lifestyle through the phases of dying

hospice

functional disease

may be manifested as an organs disease, but careful examination fails to reveal evidence of structural or psychological abnormals

tympanic

membranous eardrum

What condition greatly increases calcium loss in women?

menopause

The patient complains that her medications stick in her throat. What would be an appropriate response of the nurse?

offering a sip of water before and after each pill

An 80-year-old woman complains of pruritus from dry skin. What would be an appropriate bathing schedule for the patient? a. A hot shower every night before going to bed. b. A cool shower every morning using a detergent soap. c. A soak in a warm sudsy bath, leaving a film of soap on the skin. d. One shower a week, with sponge baths in between.

one shower a week, with sponge baths in between

How can glaucoma be detected and treated?

ophthalmic examination (everyone older than 40) medications, surgery, and laser therapy

hygiene

science of health and its maintenance

as in health screening which are done to identify older people who are in need of further more in depth assessment

screening

turgor

the normal resiliency of the skin caused by outward pressure of the cells and interstitial fluid, may be assessed as increased or decreased skin turgor

an abstract statement formulated to predict explain or describe the relationships among concepts, constructs, or events

theory

The long-term care facility nurse requests a dental consult to treat gingivitis in a resident. If left untreated, what can gingivitis ultimately cause? a. Receding gums b. Tooth loss c. Bleeding d. Halitosis

tooth loss

Filtration

transfer of water and dissolved substances from an area of high to low concentration

Why would the nurse anticipate an adverse reaction in an older adult who is taking the protein-bound drug warfarin (Coumadin)?

unbound active drug molecules continue to circulate in the bloodstream

disaster situation

uncontrollable, unexpected, psychologically shocking event

Which condition could cause delirium in the older adult? a. Uncontrolled pain b. Death of a loved one c. Relocation to a long-term care facility d. Altered sleep patterns

uncontrolled pain

What is a melanoma?

unusual appearance of moles

Which of the following is characteristic of RNA but not DNA? a. Deoxyribose b. Adenine and thymine c. Double stranded d. Uracil and adenine

uracil and adenine

When teaching a client about how to plan a healthy diet, the nurse should teach all of the following EXCEPT:

use IBW to determine your "healthy" weight

bioterrorism

use of biologic agents to create fear and threat in a terroist act

What is a symptom of theophylline overdose?

tachycardia

What is the most reliable method of patient identification for administration of medications?

an identification bracelet

pertaining to organisms;biologic aging views aging from a genetic perspective

biologic

Which condition happens in the mitochondria with oxygen?

Aerobic catabolism

Which condition produces more energy?

Aerobic catabolism

Where is the pattern of amino acid coded and stored?

Deoxyribonuclei Acid (DNA)

Which of the following is not one of the 4 D's associated with pellagra?

Depression

Anorexia

During a physical assessment, the nurse notes a patient has a lack of appetite resulting in an inability to eat. What should the nurse document that the patient is experiencing? a. Dyspnea b. Asthenia c. Anorexia d. Ecchymosis

If a patient BMI below 18.5, What are they considered ?

Underweight

What urine specfic gravity value would indicate

Concentration urine

What is the blueprint for life?

DNA

Which of the following best defines therapeutic communication?

Has specific intent or purpose

Myocardial infarction diet

Low sodium low fat

What is the name for Vitamin b6

Pyridoxine

neoplastic

Uncontrolled or abnormal growth of cells

tachypnea

abnormal rapid rate of breathing

What is pernicious anemia?

associated with decreased intake or absorption of vitamin B12

Which condition happens in the cytoplasm in the absence of oxygen?

Anaerobic catabolism

What is the normal pH of blood?

7.35-7.45 (74)

How many kcal/g does 1 g of alcohol provide?

7kcal/g

What is the toxicity of Vitamin D ?

Calcification of soft tissue

What are solutes?

Dissolved particles

What is the function of Vitamin B3?

General metabolism

Which of the following anions buffers H+? a. Ca2+ b. HCO3- c. Na+ d. K+

HCO3-

Which type of solution cause the cells to shrink?

Hypertonic

Good Samaritan law

Legal stipulation for protection of those who give first aid in an emergency situation

What is the known as the power house of the cell?

Mitochondria

1,2

When performing a venipuncture, the tourniquet should be left on no more than ____ to ____ minutes.

urinal

device for receiving urine

What are the treatments for Rheumatoid arthritis?

stress reduction, balance rest and exercise, and joint care using splints to supports joints

The home health nurse would be most concerned about self-medicating errors for the older adult living alone who is a type 1 diabetic and is

visually impaired

disaster manual

written agency instructions specifying department responsibilities, chain of command, callback procedures for the receipt and management of casualities and policies related to the overall management of supplies

What are the three buffer systems of the body? (Select all that apply.) a. Bicarbonate/carbonic acid system b. Respiratory system c. Renal system d. GI system e. Integumentary system

Bicarbonate/carbonic acid system Respiratory system Renal system

When discussing the digestion and metabolism of fat, the nurse tells the patient who has a history of cholecystitis and who is on a low-fat diet that fat must be emulsified to digested. What is the substance necessary for emulsification?

Bile

What are the 3 buffer systems?involved in keeping the ph normal

Blood buffers, lungs (respiratory) and kidneys

Which of the following are functions of Calcium?(select all that apply) A. Blood clotting B. carbohydrate metabolism C. bone and teeth formation and maintenance D. muscle contraction

Blood clotting bone and teeth formation and maintenance muscle contraction

What is the function of magnesium?

Bone mineralization, muscle contraction and relaxation

What are Heberden nodes? a. Yellow longitudinal lines in the nails b. Thickened discolored fingernails c. Darkened areas under the fingernail d. Bony enlargements of distal joints of the fingers

Bony enlargements of distal joints of the fingers

Which of the following diseases are associated with protein deficiency? A. Marasmus B. Kwashiokor C. Diabetes D. Both a and b

Both a and b

The distraught wife of a terminally ill patient complains to the nurse, "My husband has not been shaved, and he has that miserable gown on instead of his own pajamas. Don't you people care about things like that?" What would be the best response of the nurse? a. "I delayed his morning care because he was sleeping comfortably. I'll complete his care now that he's awake." b. "We're running late today and I have six other patients to care for. What do you want?" c. "Of course we care! Someone will come to do his care before lunch." d. "I'm sorry you feel we're doing such a poor job. I'm doing my best."

"I delayed his morning care because he was sleeping comfortably. I'll complete his care now that he's awake."

Which statement by the patient would indicate a need for further instruction regarding prevention of constipation? a. "I eat bran flakes or oatmeal every day to add bulk to my diet." b. "Since I started eating three servings of fruit a day, I haven't been constipated." c. "I'm never constipated. I take a gentle laxative every night." d. "My daily walks have kept my bowels working regularly."

"Im never constipated. I take a gentle laxative every night."

The nurse in an extended-care facility finds an 86-year-old female resident in tears and her hearing aids on the floor. The resident says, "I'll just be deaf! I can't stand those things in my ears! All I can hear is static, hums, and whistles!" What would be the most helpful response by the nurse? a. "Everybody says that. I'm going to put these back in the box in your bedside table." b. "Those are very expensive pieces of equipment. Because you paid for them, it seems to me to just be good sense to use them." c. "Let's put them back in. You'll get used to them in a few days." d. "It's frustrating to have something not work. Let me help you replace them and after 10 minutes, I'll help you take them out."

"It's frustrating to have something not work. Let me help you replace them after 10 minutes, I'll help you take them out."

The 60 year old Asian man tells the home health nurse that he has stopped taking his antihypertensive medication because it causes him to be impotent. He reports that he is using acupuncture to control is hypertension. What would be the most effective response by the nurse?

"Let me check your blood pressure to see how acupuncture is working."

What statement by the patient would indicate stress incontinence? a. "Lifting my grandchild makes me wet my pants." b. "I frequently wet myself because I just can't get to the bathroom in time." c. "My arthritis makes me so clumsy that I can't get my pants down in time." d. "Every time I have a urinary infection, I experience incontinence."

"Lifting my grandchild makes me wet my pants."

The Certified Nursing Assistant (CNA) caring for an older adult asks if the yellow, waxy, crusty lesions on the patient's axilla and groin are contagious. Which response shows the nurse's understanding for the cause of the lesions? a. "Yes. It is cellulitis caused by bacteria." b. "No. It is seborrheic dermatitis caused by excessive sebum." c. "Yes. It is an indication of scabies." d. "No. It is the lesion seen with basal cell carcinoma."

"No. It is seborrheic dermatitis caused by excessive sebum."

A 70 year old male patient taking corticosteroids for rheumatoid arthritis asks the nurse if he should take the vaccination to prevent shingles. What would be the nurse's most helpful response?

"No. Persons with impaired immune systems should not take that vaccine."

Which of the following examples represent "elderspeak?" (Select all that apply.) a. "Ok, honey, let's get a bath now." b. "Oh, dear! We better get you a clean diaper." c. "Today is Tuesday, Mr. Brown. It's your dialysis day." d. "My gracious, Mary! What in the world are you doing out here in the hall?" e. "You naughty girl! Just look at the front of your dress."

"Ok, honey let's get a bath now." "Oh, dear! We better get you a clean diaper." "My gracious, Mary! What in the world are you doing out here in the hall?" You naughty girl! Just look at the front of your dress

The 65 year old overweight, hypertensive male farmer tells the home health nurse that he eats two fried eggs, four pieces of bacon, and biscuits with gravy every morning for breakfast because he believes that a robust breakfast keeps him healthy. What would be the most appropriate response by the nurse?

"One egg and whole wheat toast would be even healthier."

What statement would put the older woman most at ease during a health interview? a. "The interview will take quite a while." b. "Please have a seat over there across from the desk." c. "There are lots of questions that we must finish." d. "The bathroom is behind that green door. We'll be taking a break in about 30 minutes."

"The bathroom is behind that green door. We'll be taking a break in about 30 minutes."

What would be the most helpful response to a 70-year-old postmenopausal woman who asks whether her hormone replacement therapy (HRT) will prevent bone loss? a. "No. HRT is not helpful after the age of 60." b. "Yes. HRT will prevent bone loss but can cause a stroke, heart attack, or breast cancer." c. "No. HRT is reliant on some natural estrogen production from the ovaries." d. "Yes. HRT is a widely accepted therapy for prevention of bone loss."

"Yes. HRT will prevent bone loss but can cause a stroke, heart attack, or breast cancer."

Plasma oncotic pressure a. is caused primarily by salt (sodium chloride). b. "pulls" water into the capillaries from the interstitium. c. pushes water out of the capillaries into the interstitium. d. increases the flow of lymph.

"pulls" water into the capillaries from the interstitium

The nurse is entering the room of a new 85 year old female patient to complete the admission process. How should the nurse initiate the conversation?

"welcome to 4B, Mrs. Miller. I'd like to get some additional information, If I may."

What are some causes of metabolic acidosis?

*renal failure (decreased production of HCO3 as well as decreased excretion of H+ ions) *severe diarrhea (increased loss of HCO3) *lactic acidosis (increased production of acids) *asprin toxicity (AKA : ASA Toxicity) *starvation *anaerobic metabolism secondary to hypoxia (remember cell injury leads to lactic acid production and accumulation)

What are external factors that may be a safety threat to the older adult? (Select all that apply.) a. Fire hazards b. Lack of home security c. Vehicular accidents d. Thermal hazards e. Sensory deficit

- Fire hazards -Lack of home security -Vehicular accidents -Thermal hazards

What are appropriate roles of the nurse when working with the bereaved? (Select all that apply.) a. Guidance b. Resource c. Advocacy d. Support e. Spiritual counseling

-Guidance -Resource -Advocacy -Support

What are common side effects of sleep medications? (Select all that apply.) a. Hangover effect b. Urinary retention c. Hypotension d. Dizziness e. Diarrhea

-Hangover effect -Urinary retention -Hypotension -Dizziness

What conditions could result in constipation related to pain? (Select all that apply.) a. Hemorrhoids b. Anal fissures c. Reduction of bowel mucus d. Diminished abdominal muscle tone e. Slowed peristalsis

-Hemorrhoids -Anal fissures -Reduction of bowel mucus

What statements by the resident would indicate that she may be experiencing changes in sleep and rest patterns? (Select all that apply.) a. "I don't know why everything seems to bother me lately." b. "I've been so clumsy." c. "I'm having trouble concentrating." d. "My daughter says I talk in my sleep." e. "I cry for no reason at all."

-"I don't know why everything seems to bother me lately." -"I've been so clumsy." -"I'm having trouble concentrating." -"I cry for no reason at all."

What must an older adult develop in order to achieve Erikson's task of integrity versus despair? (Select all that apply.) a. A positive attitude toward aging b. Positive self-esteem c. A manageable degree of illness d. A ready support system e. Control of all life choices

-A positive attitude toward aging -Positive self-esteem -A manageable degree of illness -A ready support system

What are causes of pruritus in the older adult? (Select all that apply.) a. A reduction of sebaceous gland function b. A reduction in the amount of perspiration c. Excessive bathing d. Use of emollients e. Environmental conditions

-A reduction of sebaceous gland function -A reduction in the amount of perspiration -Excessive bathing -Environmental conditions

How can the nurse demonstrate a positive attitude toward care planning? (Select all that apply.) a. Acknowledge the impact of a traumatic amputation on therapy. b. Encourage the resident's input on the care plan. c. Invite the resident and family member to attend the care plan meetings. d. Make a list of questions for the resident to ask at the care plan meeting. e. Help the resident to perform all activities of daily living (ADLs).

-Acknowledge the impact of a traumatic amputation on therapy. -Encourage the resident's input on the care plan. -Invite the resident and family member to attend the care plan meetings.

What are the three types of confusion? (Select the three that apply.) a. Acute confusion b. Mixed confusion c. Idiopathic confusion d. Generic confusion e. Dementia

-Acute confusion -Idiopathic confusion -Dementia

How can a nurse encourage communication with a dying patient? (Select all that apply.) a. Address the patient by name at every opportunity. b. Use direct questions to inquire about the patient's concerns relative to health and end-of-life issues. c. Complete care with a minimum of conversation to encourage questions from the patient. d. Provide empathetic touching during care to show concern. e. Sit down in the room to converse with the patient.

-Address the patient by name at every opportunity. -Provide empathetic touching during care to show concern. -Sit down in the room to converse with the patient

What are a person's perceptions regarding end-of-life care dependent upon? (Select all that apply.) a. Age b. Gender c. Cultural background d. Caregivers e. Life experiences

-Age -Gender -Cultural background -Life experiences

What actions by the nurse would help a patient who is experiencing a loss of self-control? (Select all that apply.) a. Allowing the patient to make choices whenever possible b. Assisting the patient to perform all self-care activities c. Respecting the patient's right to refuse treatment d. Explaining all procedures ahead of time e. Adapting the environment to enhance self-care

-Allowing the patient to make choices whenever possible -Respecting the patient's right to refuse treatment -Explaining all procedures ahead of time -Adapting the environment to enhance self-care

The nurse instructs a group of older adults on how to maintain intact skin. What would be included in the teaching plan? (Select all that apply.) a. Altering their body position every 30 minutes while sitting in the chair b. Changing incontinent products when they become soiled c. Using a pressure-relieving device like the "donut" to sit on d. Routinely checking their feet for redness and indentations e. Patting the skin dry after a shower or bath

-Altering their body position every 30 minutes while sitting in the chair -Changing incontinent products when they become soiled -Patting the skin dry after a shower or bath

Which of the following are signs of Vincent angina? (Select all that apply.) a. An advanced state of malnutrition b. Enlargement of the cervical lymph nodes c. Epistaxis d. Dysphagia e. A discolored tongue

-An advance state of malnutrition -Enlargement of the cervical lymph nodes -Dysphagia

What measures would be appropriate for the nurse to use to assess dysfunctional grieving in the older adult? (Select all that apply.) a. Ask the patient to describe his or her relationship within the family. b. Observe the behavior of the patient when family members and friends come to visit. c. Ask questions about the patient's profession and work history. d. Observe for changes in eating habits. e. Monitor for changes in mental status.

-Ask the patient to describe his or her relationship within the family. -Observe the behavior of the patient when family members and friends come to visit. -Observe for changes in eating habits -Monitor for changes in mental status

How can an older adult with a failing memory assume control to improve self-image? (Select all that apply.) a. Attitude toward aging b. Financial needs c. Physical appearance d. Time use e. Relationships

-Attitude toward aging -Physical appearance -Time use -Relationships

What are benefits of dehydration for a person who is near death? (Select all that apply.) a. Better gas exchange because of reduction of fluid in the lungs b. Decreased need to suction because of reduction of respiratory secretions c. Improved bowel elimination because of reduced bulk in the stool d. Decreased pain related to release of endorphins e. Improved drug distribution via the circulating volume

-Better gas exchange because of reduction of fluid in the lungs. -Decreased need to suction because of reduction of respiratory secretions. -Decreased pain related to release of endorphins

What interventions by the nurse will aid in maintaining skin integrity in the long-term care facility residents? (Select all that apply.) a. Changing the briefs and bed linens when damp b. Maintaining the temperature in the room at 80 degrees c. Rinsing excess soap off the skin during a shower d. Administering frequent pericare on the continent resident e. Laying residents down after lunch

-Changing the briefs and bed linens when damp -Rinsing excess soap off the skin during a shower -Laying residents down after lunch

The home health nurse is assessing the patient's home. Which of the following would be identified as a fall risk? (Select all that apply.) a. Brightly lit rooms b. Pantry food at an accessible level c. Colorful scatter rugs marking doorways and steps d. Wearing comfortable laced tennis shoes e. Attractive, low, magazine rack beside a chair

-Colorful scatter rugs marking doorways and steps -Attractive,low,magazine rack beside a chair

In what ways can a nurse help a demented person maintain orientation? (Select all that apply.) a. Consistently calling the patient by name, usually the first name b. Referring the patient to a calendar to note special events c. Reminding the patient about the time of day by pointing to the clock d. Calmly taking the patient to an appointment without explanation e. Reminding the patient of her or his whereabouts frequently

-Consistently calling the patient by name, usually the first name. -Referring the patient to a calendar to note special events -Reminding the patient about the time of day by pointing to the clock -Reminding the patient of her or his whereabouts frequently

Which of the following complaints would indicate a possible fecal impaction in an older adult? (Select all that apply.) a. Cramping b. Rectal pain c. Abdominal distention d. Anorexia e. Passing large amounts of liquid stool

-Cramping -Rectal pain -Abdominal distention -Anorexia

What are causes of bowel and bladder elimination problems in the older adult? (Select all that apply.) a. Daily use of laxatives for defecation b. Inability to get to adequate water c. Fear of pain with defecation d. Total privacy during elimination e. Use of a bedpan for elimination

-Daily use of laxatives for defecation -Inability to get adequate water -Fear of pain with defecation -Use of a bedpan for elimination

What are internal factors that threaten the safety of the older adult? (Select all that apply.) a. Decrease in flexibility b. Slowed reaction time c. Gait changes d. Thermal hazards e. Postural changes

-Decrease in flexibility -Slowed reaction time -Gait changes -Postural Changes

What age-related changes are related to constipation? (Select all that apply.) a. Diminished abdominal muscle tone b. Reduced activity level c. Inadequate fluid intake d. Increased dietary fiber e. Dependence on laxatives

-Diminished abdominal muscle tone -Reduced activity level -Inadequate fluid intake -Dependence on laxatives

What are reasons that most long-term care residents go to sleep early and awaken early? (Select all that apply.) a. Increased blood pressure b. Drop in core temperature c. Diminished food intake d. Diminished hormone production e. Decreased exposure to light

-Drop in core temperature -Decreased exposure to light

What would be appropriate actions of the nurse when dealing with a bereaved family at the time of death? (Select all that apply.) a. Review the stages of dying with them. b. Ensure privacy. c. Observe cultural and spiritual preferences. d. Legally pronounce the death. e. Allow the bereaved to express their feelings.

-Ensure privacy. -Observe cultural and spiritual preferences. -Allow the bereaved to express their feelings.

What influences the older adult's self-perception? (Select all that apply.) a. Family support b. Ethnic heritage c. Health status d. Financial status e. Sense of independence

-Family support -Health status -Financial Status -Sense of independence

What influences an older adult's self-concept? (Select all that apply.) a. Financial security b. Social life c. Physical health d. Mobility e. Cognitive function

-Financial security -Social Life -Physical Health -Mobility

What are the causes of altered oxygen exchange, and resulting lessened stamina, in the older adult? (Select all that apply.) a. Flattened diaphragm b. Loss of elasticity in the lung c. Nutritional deficiencies d. Decrease in size of chest cavity e. Fragility of capillaries

-Flattened diaphragm -Loss of elasticity in the lung -Nutritional deficiencies -Decrease in size of chest cavity

What does the nurse expect will occur in a patient as a result of decreased fluid intake? (Select all that apply.) a. Fluid will be withdrawn from the stool b. The stool becomes hard and dry c. Less mucus is formed in the colon d. Lumen of bowel constricts because of smaller bulk e. Peristalsis slows

-Fluid will be withdrawn from the stool -The stool becomes hard and dry -Less mucus is formed in the colon

What should be stressed in a teaching plan for a family caring for an older adult who is experiencing pain? (Select all that apply.) a. Give medication before the pain becomes severe. b. Distract the patient instead of medicating. c. Pain reported by the older adult may be exaggerated. d. Delay administration to reduce the risk of addiction. e. Observe the effectiveness of the medication.

-Give medication before the pain becomes severe. -Observe the effectiveness of the medication

What should be done with copies of a signed advance directive? (Select all that apply.) a. Given to the person who has been named as power of attorney for medical decisions b. Posted in the home for emergency personnel c. Placed in the medical record in the physician's office d. Filed with the hospital e. Given to the neighbor

-Given to the person who has been named as power of attorney for medical decisions. -Posted in the home for emergency personnel. -Placed in the medical record in the physicians office -Filed with the hospital

What should the nurse consider when planning an exercise program for a person with activity intolerance? (Select all that apply.) a. Identification of factors that contribute to activity intolerance b. Arranging activities that progress from mild to more demanding c. Rapid pacing activities to build up stamina d. Individualizing the plan to include activities that the patient particularly likes e. Including the patient in the planning phase

-Identification of factors that contributes to activity intolerance -Arranging activities that progress from mild to more demanding -Individualizing the plan to include activities that the patient particulary likes -Including the patient in the planning phase

What are the broad general causes of bowel incontinence? (Select all that apply.) a. Inability to recognize defecation urge related to mental impairment b. Inability to respond to defecation urge related to immobility c. Inflammatory bowel disease d. Increased fiber in the diet e. Unexpected defecation when passing gas

-Inability to recognize defecation urge related to mental impairment -Inability to respond to defecation urge related to immobility -Inflammatory bowel disease -Unexpected defecation when passing gas

A nurse is teaching a family member about her mother's memory loss. Which of the following would be included in the teaching regarding memory loss? (Select all that apply.) a. Increases with age b. Decreases in a person with more education c. Increases with the use of antihistamines d. Decreases with the use of vitamin A e. Decreases in persons who have many varied memories

-Increases with age -Decreases in a person with more education -Decreases in persons who have many varied memories

What are the overall goals in caring for a patient with impaired physical mobility? (Select all that apply.) a. Increasing the patient's participation in physical activities b. Preserving the patient's anatomical position and function of joints c. Increasing the patient's former level of mobility d. Avoiding unnecessary restraints e. Using assistive devices to maintain mobility

-Increasing the patient's participation in physical activitiies -Preserving the patient's anatomical position and function of joints -Avoiding unnecessary restraints -Using assistive devices to maintain mobilty

The nurse instructs a hearing-impaired patient on some methods to improve communication, which include __________. (Select all that apply.) a. informing others of the hearing deficit b. focusing on the speaker c. facing the speaker d. requesting the speaker to shout if necessary e. asking the speaker to repeat what is not clear

-Informing others of the hearing deficit -Focusing on the speaker -Facing the speaker -Asking the speaker to repeat what is not clear

What are the benefits of an advance directive? (Select all that apply.) a. It is legally binding. b. It remains in effect until the family changes it. c. It prevents last-minute confusion about the wishes of the patient. d. It prevents violation of end-of-life choices. e. It clarifies end-of-life issues for the family.

-It is legally binding. -It prevents last minute confusion about the wishes of the patient -It prevents violation of end of life choices -It clarifies end of life issues for the family

What is true of the health assessment in a long-term care facility? (Select all that apply.) a. It is the basis for the assignment of nursing diagnoses. b. It is done only at admission to the facility. c. It is performed only by a registered nurse. d. It is a platform for the nursing care plan. e. It is ongoing for the duration of the stay.

-It is the basis for the assignment of nursing diagnoses. -It is a platform for the nursing care plan -It is ongoing for the duration of the stay

What influences regular bowel patterns? (Select all that apply.) a. Level of activity b. Diet c. Medication d. Fluid intake e. Lifestyle

-Level of activity -Diet -Fluid intake -Lifestyle

Why is end-of-life planning often neglected? (Select all that apply.) a. There is a clear procedure to follow. b. People are uncomfortable talking about death. c. Young people do not see the need for end-of-life planning. d. End-of-life planning is a relatively new concept. e. Many persons are not really sure what they want to do.

-People are uncomfortable talking about death. -Young people do not see the need for end of life planning. -End of life planning is a relatively new concept. -Many persons are not really sure what they want to do.

What are the benefits of regular exercise? (Select all that apply.) a. Maintenance of joint mobility b. Enhancement of muscle tone c. Promotion of sense of general well-being d. Guarantee of weight loss e. Promotion of regular elimination

-Maintenance of joint mobility -Enhance of muscle tone -Promotion of sense of general well being -Promotion of regular elimination

What are goals of rehabilitative nursing? (Select all that apply.) a. Minimize the impact of disability b. Return to maximum level of function c. Increase level of independence d. Adjust to change in lifestyle e. Increase control of their life

-Minimize the impact of disability -Increase level of independence -Adjust to change in lifestyle -Increase control of their life

Why is self-medication with over-the-counter (OTC) drugs hazardous to the older adult? (Select all that apply.) a. OTC drugs can increase the effect of a prescribed drug. b. OTC drugs can interfere with the efficacy of a prescribed drug. c. OTC drugs can mask significant symptoms of primary disease. d. OTC drugs are easily obtained. e. OTC drugs can lead to overdose because they are not considered to be "real drugs."

-OTC drugs can increase the effect of a prescribed drug -OTC drugs can interfere with the efficacy of a prescribed drug -OTC drugs can mask significant symptoms of primary disease -OTC drugs can lead to overdose because they are not considered to be "real drugs."

The nurse notices there has been a change in the behavior of an 84-year-old home patient over the past few weeks. What methods can be used to assess the patient's self-perception and self-concept? (Select all that apply.) a. Observing his physical appearance b. Monitoring for changes in his vital signs c. Encouraging him to verbalize his feelings d. Observing for changes in activities e. Participating in group activities

-Observing his physical appearance -Monitoring for changes in his vital signs -Encouraging him to verbalize his feelings -Observing for changes in activities

Research by the National Institute on Aging found which of the following to be correct? (Select all that apply.) a. Older patients receive less information regarding health management than younger patients. b. Information on lifestyle changes were directed at younger people. c. Older patients were denied timely appointments for evaluation of acute illnesses. d. Rehabilitation programs offered limited services to the older adult. e. Older patients receive less evaluation and fewer treatment options for acute illnesses.

-Older patients receive less information regarding health management than younger patients. -Information on lifestyle changes were directed at younger people. -Rehabilitation programs offered limited services to the older adult. -Older patients receive less evaluation and fewer treatment options for acute illnesses

The daughter of a home health patient is concerned about her 92-year-old father who continues to drive regularly. Which observations would indicate deteriorated driving skills? (Select all that apply.) a. Paint scrapes on the mailbox at the curb b. Friends calling him to get rides to the grocery store c. Choosing not to drive at night because of night blindness d. Difficulty turning his head e. Carefully planning routes to avoid heavy traffic

-Paint scrapes on the mailbox at the curb -Difficulty turning his head

The home health nurse has evaluated the community for measures that support pedestrian safety. What safety measures would be appropriate modifications to increase safety in the community? (Select all that apply.) a. Pedestrian-controlled crosswalks b. Safety islands on wide street intersections c. Decreased time to cross at walks d. Clearly marked crosswalks at intersections e. Overhead crossings over busy streets

-Pedestrian-controlled crosswalks -Safety islands on wide street intersections -Clearly marked crosswalks at intersections -Overhead crossing over busy streets

Snacking may constitute 30% or more of an adolescent's total caloric intake each day

True

What should be included in a plan of care for an older adult with diarrhea? (Select all that apply.) a. Perineal care immediately after the diarrhea episode b. Application of lotion to the buttocks c. Maintenance of dry linens d. Patting the anal area dry rather than wiping e. Leaving excoriated areas open to the air

-Perineal care immediately after the diarrhea episode -Maintenance of dry linens -Patting the anal area dry rather than wipin -Leaving excoriated areas open to the air

What telephone modifications would increase safety for the older adult? (Select all that apply.) a. Placement of phones at bedside and next to a favorite chair b. Programming an auto dial function for quick dialing c. Using an answering machine with a male voice d. Replacing the phone cord with a 15-foot cord for ease in carrying around the phone e. Selecting a phone with large numbers

-Placement of phones at bedside and next to a favorite chair -Programming an auto dial function for quick dialing -Using an answering machine with a male voice -Selecting a phone with large numbers

What are the long-term goals for rehabilitation? (Select all that apply.) a. Prevention of further disability b. Modifying the impact of the disability on lifestyle c. Supporting adaptation to a changed lifestyle d. Focusing on a complete return to the former level of activity e. Reestablishing the patient's control of her or his life

-Prevention of further disability -Modifying the impact of the disability on lifestyle -Supporting adaptation to a changed lifestyle -Reestablishing the patient's control of her or his life

What are causes of xerostomia in the older adult? (Select all that apply.) a. Reduction in saliva b. Allergy c. Eating highly seasoned foods d. Inadequate fluid intake e. Use of diuretic medications

-Reduction in saliva -Inadequate fluid intake -Use of diuretic medications

What can the nurse suggest as nonpharmacological alternatives to treating insomnia? (Select all that apply.) a. Relaxation therapy b. Taking a cool bath or shower before bedtime c. Listening to relaxing music d. Arranging the sleep environment to promote sleep e. Going to bed at a regular time after observing routine "sleep rituals"

-Relaxation therapy -Listening to relaxing music -Arranging the sleep environment to promote sleep -Going to bed at a regular time after observing rountine "sleep rituals"

Which of the following are indicators of hearing loss? (Select all that apply.) a. Understanding the female voice better than the male voice b. Reluctance to have telephone conversations c. Becoming irritable with background noise d. Turning the TV up to the loudest volume e. Responding with "off-the-wall" answers to a question

-Reluctance to have telephone conversations -Becoming irritable with background noise -Turning the TV up to the loudest volume -Responding with "off-the-wall" answers to a question

A resident in the long-term care facility has just expired, and the family is on the way to say their last goodbyes. What should the nurse do prior to the arrival of the family? (Select all that apply.) a. Remove equipment (e.g., IV poles and tubing, feeding tubes, oxygen equipment). b. Dress the resident in a clean gown and cover him or her with a clean sheet. c. Remove dentures. d. Apply a diaper to catch draining body fluids. e. Provide privacy by drawing the curtain or moving the other resident to an unoccupied room.

-Remove equipment (e.g., IV poles and tubing, feeding tubes, oxygen equipment). -Dress the resident in a clean gown and cover him or her with a clean sheet. -Apply a diaper to catch draining body fluids. -Provide privacy by drawing the curtain or moving the other resident to an unoccupied room

The nurse in a long-term care facility awards stickers to certified nursing assistants who consistently __________. (Select all that apply.) a. report broken tiles in the shower room and bathrooms b. mop up spills c. assist residents to hurry d. remind residents to use walkers e. retie residents' shoelaces

-Report broken tiles in the shower room and bathrooms -Mop up spills -Remind residents to use walkers -Retie residents shoelaces

When performing a skin assessment for pressure ulcers on an older man who is bedridden and prefers to lie on his right side, the nurse will pay special attention to the __________. (Select all that apply.) a. right ear b. lateral edge of the right foot c. sacrum d. medial edge of the left foot e. right scapula

-Right ear -Lateral edge of the right foot -Medial edge of the left foot

What can alter an older adult's diurnal patterns? (Select all that apply.) a. Shift work b. Time zone changes c. Altered nutrition d. Illness e. Medications

-Shift work -Time zone changes -Illness -Medications

The wife of a patient tells the home health nurse that she suspects her husband has sleep apnea because he __________. (Select all that apply.) a. snores loudly b. interrupts snoring with several seconds of silence c. complains of daytime drowsiness d. frequently is incontinent of urine e. has episodes of myoclonus

-Snores loudly -Interrupts snoring with several seconds of silence -Complains of daytime drowsiness

What are the benefits of a holistic caregiver at the end-of-life? (Select all that apply.) a. The caregiver communicates and responds to the patient's concerns about death. b. The caregiver embraces the value of palliative care rather than curative care. c. The caregiver helps the patient and family face loss and grieve in anticipation of loss. d. The caregiver encourages the patient to focus on wellness and recovery. e. The caregiver collaborates with other professionals for patient support.

-The caregiver communicates and responds to the patients concern about death. -The caregiver embraces the value of palliative care rather than curative care. -The caregiver helps the patient and family face loss and grieve in anticipation of loss. -The caregiver collaborates with other professionals for patient support.

The charge nurse on an oncology unit encourages her staff to complete a values clarification exercise. What type of insight is provided by the exercise? (Select all that apply.) a. The nurses' own values b. Values that influence care decisions c. Values that must be abandoned or changed d. Values that may cause an ethical dilemma e. Negative values

-The nurses own values -Values that influences care decisions -Values that may cause an ethical dilemma

Which of the following would be considered palliative care? (Select all that apply.) a. Treat symptoms of pain, dyspnea, and nausea. b. Promote acceptance of death. c. Provide CPR. d. Monitor mechanical ventilation machines. e. Enable the patient to have a better quality of life.

-Treat symptoms of pain, dyspnea, and nausea -Promote acceptance of death -Enable the patient to have a better quality of life

A dark-complexioned African American has a suspected skin breakdown over the trochanter. What would be appropriate measures for the nurse to implement during the assessment? (Select all that apply.) a. Use a halogen light to examine the area. b. Palpate for local edema in the area. c. Touch the area to feel for changes in tissue temperature. d. Assess for localized pain. e. Press the area to test for blanching.

-Use a halogen light to examine the area -Palpate for local edema in the area -Touch the area to feel for changes in tissue temperature

What forms an individual's self-identity? (Select all that apply.) a. Values b. Ambition c. Interaction with others d. Self-esteem e. Ability to control his or her life

-Values -Interaction with others -Self-esteem -Ability to control his or her life

What factors should the older adult consider when planning an exercise program? (Select all that apply.) a. Purchasing weights b. Wearing clothing appropriate to the type of exercise c. Considering membership in a health club d. Establishing realistic goals e. Committing time for consistent regular exercise

-Wearing clothing appropriate to the type of exercise -Considering membership in a health club -Establishing realistic goals -Committing time for consistent regular exercise

The nurse should include which factor(s) in a pain assessment? (Select all that apply.) a. What provokes it b. Location c. Race d. Severity e. Frequency

-What provokes it -Location -Severity -Frequency

Anticoagulant- Coumadin is counteracted by ?

Vitamin K

What is grief a normal response to? (Select all that apply.) a. Loss of loved ones b. Loss of professional roles c. Loss of dependence d. Loss of health e. Loss of relationships

-loss of loved ones -loss of professional roles -loss of health -loss of relationships

How can the nursing staff encourage sleep in long-term care facility residents? (Select all that apply.) a. Using the minimum light necessary when making rounds b. Making necessary sleep interruptions at the same time every night c. Keeping conversational noise at the nursing station to a minimum d. Answering call lights promptly e. Providing heavy blankets for warmth

-using the minimum light necessary when making rounds -Making necessary sleep interruptions at the same time every night -Keeping conversational noise at the nursing station to a minimum -Answering call lights promptly

morphine

.A nurse is caring for a patient who requires long-term management for severe pain. What should be the drug of choice for this patient? a. Aspirin b. Morphine c. Oxycodone d. Acetaminophen

ecchymosis

.Discoloration of an area of the skin or mucous membrane that is caused by the extravasation of blood into the subcutaneous tissues as a result of trauma to the underlying blood vessels or by fragility of the vessel walls is known as _________________.

on his or her right side

.Following a liver biopsy, the nurse should observe for hemorrhage and ensure that the patient is kept on bed rest for 24 hours. How should the nurse keep the patient for the first 1 to 2 hours?

Arrange the steps for preparing crushed medications to be given by feeding tube in order of priority. a. Flush the tube to clear feeding. b. Thoroughly crush the medication. c. Administer each medication separately. d. Dissolve each crushed medication in a medicine cup. e. Flush the tube to clear the medication from the tube. f. Reconnect the feeding tube.

1.Thoroughly crush the medication 2.Dissolve each crushed medication in a medicine cup 3.Flush the tube to clear feeding 4.Administer each medication separately 5.Flush the tube to clear the medication from the tube 6. Reconnect the feeding tube

What is the average fluid intake necessary for safe administration of a bulk-forming laxative to a dying patient? a. 1000 mL b. 1500 mL c. 2000 mL d. 2500 mL

2000ml

The nurse expects an adult with normal kidney function to void a minimum of ____ mL of urine in 4 hours.

120

To what age group does the term "aged" apply? a. 55-64 years of age b. 65-74 years of age c. 75-84 years of age d. 85 and older

75-84 years of age

Homeostasis of the hydrogen ion concentration in body fluids depends on the ratio of carbonic acid to bicarbonate in the extracellular fluid. What is this ratio? a. 1:5 b. 1:10 c. 1:15 d. 1:20

1:20

What is the ratio of bicarbonate to carbonic acid?need to keep the body in homesostatis

1:20

On the admission assessment of a resident to a long-term care facility, the nurse notes a painless area on the patient's coccyx that has partial skin loss. The nurse would record this as a stage _____ pressure ulcer

2

in the early morning

A sputum specimen is ordered on a patient diagnosed with pneumonia. When is the best time for the nurse to the attempt to collect this specimen?

What is hyperkalemia?

A sreum potassium level that exceeds 5.1 mEq/L. Excessive potassium in the blood

Which of the following numbers is considered a healthy BMI? (select all that apply) A. 35 B. 40 C. 22 D. 28

22 & 28

Noxious

A stimulation of the sensory nerve endings that harmful injurious or detrimental to physical health

The nurse must keep an accurate intake and output record to assess kidney efficiency. In order for the kidneys to remove waste, what is the least amount of hourly urine output the kidneys must produce to remove waste? a. 10 mL b. 20 mL c. 30 mL d. 40 mL

30ML

The body mass index (BMI) of a man 6 feet tall weighing 250 pounds is _______.

33.9

An adult's total body weight is _______ water.

60%

The nurse reminds a patient who was born in 1965 that eligibility for full Social Security benefits for persons of this age is now the age of __________.

67

What percent of communication is transmitted by verbal methods?

7

Which of the following is a pH descriptive of ketoacidosis? a. 7.50 b. pH > 7.45 c. 14.0 d. 7.25

7.25

Which of the following would be most likely to experience a problem with role conflict? a. 65-year-old grandmother who cares for her grandchildren while the mother works. b. 70-year-old retired business owner who comes to help his son run the business. c. 75-year-old retired physician who volunteers at a medical screening clinic. d. 80-year-old retired school administrator who takes educational foreign cruises several times a year.

70-year-old retired business owner who comes to help his son run the business.

What is the range of death for Alzheimer's ?

8 yearss to more than 20 years

Lateral recumbent

A physician needs to assess a patient for a heart murmur. In what position should the nurse place the patient? a. Sims b. Prone c. Lithotomy d. Lateral recumbent

The daughter of a patient who has been diagnosed with terminal cancer asks which documents are required to allow her to make health care decisions for her parent. Which response would provide the most information to the daughter? a. Advance directives indicate the degree of intervention desired by the patient. b. A 'Do Not Resuscitate' document signed by the patient transfers authority to the next of kin. c. A durable power of attorney for health care transfers decision-making authority for health care to a designated person. d. A living will transfers authority to the physician.

A durable power of attorney for health care transfers decision-making authority for health care to a designated person

Which caregiver could be found guilty of elder abuse? a. A daughter who uses her mother's Social Security money to purchase her mother's medication b. A son who puts an alarm on the front door to prevent his mother from wandering out of the house c. A wife who allows her mentally competent husband to refuse to take a bath for a week d. A frail spouse who is unable to bathe or change the clothes of her physically dependent husband

A frail spouse who is unable to bathe or change the clothes of her physically dependent husband

By what means can a health screening be conducted? (Select all that apply.) a. A health professional b. Telephone interview c. Telecomputer d. Pen and paper surveys e. Lay persons

A health Professional Telephone interview telecomputer pen and paper surveys

-Frequent orientation to surroundings -Explain all procedures and treatments -Maintain toileting rountines

A long-term care facility is committing to a restraint-free environment. What will the health care workers implement to encourage this environment? (Select all that apply.) a. Frequent orientation to surroundings b. Explain all procedures and treatments c. Discourage visitors d. Maintain toileting routines e. Minimize exercise and ambulation

The admitting nurse gives the new long-term care facility resident a score of 20 on both the Norton Risk Assessment Scale and the Braden Scale for Predicting Pressure Sore Risk. These scores indicate that the resident has a. a high probability of developing a pressure ulcer. b. a moderate risk of developing a pressure ulcer. c. a low risk of developing a pressure ulcer. d. at least one pressure ulcer at the time of admission.

A low risk of developing a pressure ulcer

wash the laceration with an antiseptic

A machinist visits the industrial nurse's clinic with a deep laceration of the thigh. What should be the nurse's first action? a. Splint the thigh and apply tape to approximate the edges b. Apply ice and a pressure dressing to the thigh c. Give a tetanus booster injection d. Wash the laceration with an antiseptic

Taking an anxiolytic medication

A male patient reports to the home health nurse that he does not feel rested although he has slept 8 hours. For what should the nurse assess? a. Having vivid dreams b. Eating a heavy meal before going to bed c. Consuming an excessive amount of alcohol d. Taking an anxiolytic medication

-the skin of older adults is more fragile and susceptible to injury -Aging tends to result in loss of flexibility and joint mobility -Older adults sometimes become fearful when hydraulic lifts are used for transfers

A newly hired group of graduate practical/vocational nurses are attending orientation at a long-term care facility. What information will be included regarding considerations of mobility and the older adult? (Select all that apply.) a. The skin of older adults is more fragile and susceptible to injury. b. Always support older adults under the soft tissue when moving them in bed. c. Weakness and hypertension are common signs and symptoms noted in an older adult on bed rest. d. Aging tends to result in loss of flexibility and joint mobility. e. Older adults sometimes become fearful when hydraulic lifts are used for transfers.

Record the findings

A nurse assesses a neonate's temperature by using a temporal artery scanner. What intervention should the nurse implement if the neonate's temperature is 96° F? a. Record the findings b. Notify the physician c. Check the axillary temperature d. Check the tympanic temperature

record the findings

A nurse assesses a neonate's temperature by using a temporal artery scanner. What intervention should the nurse implement if the neonate's temperature is 99.5° F? a. Record the findings b. Notify the physician c. Check the axillary temperature d. Check the tympanic temperature

bounding pulse

A nurse assesses a patient's dorsalis pedis pulse. The pulse feels full and springlike even under moderate pressure. How should the nurse document this finding? a. Weak pulse b. Normal pulse c. Thready pulse d. Bounding pulse

Thready pulse

A nurse assesses a patient's dorsalis pedis pulse. The pulse is difficult to feel and not palpable when only slight pressure is applied. How should the nurse document this finding? a. Weak pulse b. Normal pulse c. Thready pulse d. Bounding pulse

normal pulse

A nurse assesses a patient's dorsalis pedis pulse. The pulse is easily felt but not palpable when moderate pressure is applied. How should the nurse document this finding? a. Weak pulse b. Normal pulse c. Thready pulse d. Bounding pulse

Weak pulse

A nurse assesses a patient's dorsalis pedis pulse. The pulse is not palpable when light pressure is applied. How should the nurse document this finding? a. Weak pulse b. Normal pulse c. Thready pulse d. Bounding pulse

right brachial artery

A nurse assesses an accident victim who has bright red blood spurting from a laceration on his right forearm. Where will the nurse apply pressure after applying direct pressure and elevating the limb? a. Right subclavian artery b. Right radial artery c. Right ulnar artery d. Right brachial artery

Collapse of blood vessels

A nurse assesses an area of sustained redness on the coccyx area of a resident in long-term care. What is the most likely cause of this pressure area? a. Heat from pressure b. Collapse of blood vessels c. Friction from pressure d. Collapse of skin tissue

the nursing assistant is pulling the patient across bed linens

A nurse instructs a nursing assistant about moving older adult patients in bed. When should the nurse intervene when observing the nursing assistant perform a return demonstration? a. The nursing assistant is using simple language. b. The nursing assistant is avoiding jerky movements. c. The nursing assistant is avoiding sudden movements. d. The nursing assistant is pulling the patient across bed linens.

Knee chest

A physician needs to assess a patient's rectal area. In what position should the nurse place the patient? a. Sims b. Prone c. Lithotomy d. Knee-chest

nursing assistant is walking on the patient's strong side

A nurse instructs a nursing assistant about the proper use of a gait belt and is observing a return demonstration. What action by the nursing assistant should cause the nurse to intervene? a. Nursing assistant is walking on the patient's strong side b. Nursing assistant is walking to the side of the patient c. Nursing assistant is securing the gait belt securely around the patient's waist d. Nursing assistant is grasping the handles of the gait belt while the patient ambulates

Greet financial information during interview

A nurse is admitting a patient to an acute care facility. During the admission procedure, what nursing intervention would best help reduce patient anxiety?

abnormal breathing

A nurse is assessing victims in an emergency situation. What will the nurse assess for first? a. Hemorrhage b. Fractures c. Mobility d. Abnormal breathing

Allowing the patient to walk out of the hospital when discharged

A nurse is caring for a Haitian American patient. How might the nurse demonstrate cultural sensitivity?

Dyspnea

A nurse is caring for a patient with congestive heart failure. During the physical assessment, the nurse notes the patient is experiencing difficulty breathing. What should the nurse document that the patient is experiencing? a. Dyspnea b. Cyanosis c. Diaphoresis d. Ecchymosis

The patient expectorates red-tinged sputum

A nurse is gathering objective data when admitting a patient. Which assessment finding is considered objective data? a. The patient complains of chest pain. b. The patient states, "I am having trouble breathing." c. The patient complains of coughing up sputum. d. The patient expectorates red-tinged sputum.

appears to be anxious

A nurse is gathering objective data when admitting a patient. Which assessment finding reported by the patient is considered objective? a. Complains of nausea b. States, "I hurt all over." c. Complains of feeling anxious d. Appears to be anxious

Complains of chest pain

A nurse is gathering subjective data when admitting a patient. Which assessment finding reported by the patient is considered subjective data? a. Complains of chest pain b. Is experiencing dyspnea c. Appears to be anxious d. Expectorates red-tinged sputum

complains of diplopia

A nurse is gathering subjective data when admitting a patient. Which assessment finding reported by the patient is considered subjective data? a. Complains of diplopia b. Is experiencing nystagmus c. Demonstrates facial grimacing d. Has a generalized rash

Complains of pruritus

A nurse is gathering subjective data when admitting a patient. Which assessment finding reported by the patient is considered subjective data? a. Complains of pruritus b. Is experiencing erythema c. Appears to be experiencing pruritus d. Has a generalized rash

Sitting

A nurse needs to auscultate a patient's lung sounds. In what position should the nurse place the patient? a. Sims b. Prone c. Sitting d. Lithotomy

an individualized plan of care

A nursing assessment is a process of collecting data to establish a database. The information contained in the database is a basis for: a. a complete physical examination. b. a medical assessment. c. an individualized plan of care. d. writing nursing orders.

cover the wound with an airtight dressing taped on three sides

A patient arrives in the emergency department with a sucking wound to the left chest. What is the first action the nurse should take? a. Place several layers of gauze dressing over the wound b. Place the patient in a supine position c. Cover the wound with an airtight dressing taped on three sides d. Turn the patient to the left side

scrape the stinger with the side of a knife

A patient has been stung by a bee and is brought to the emergency department. The nurse observes the sting site and identifies that the stinger is still in the skin. What action should the nurse take? a. Remove it with sterile tweezers b. Soak the area with a cold compress c. Scrape the stinger with the side of a knife d. Squeeze the surrounding tissue to expel the stinger

ecchymosis

A patient has discoloration of an area of their mucous membrane caused by extravasation of blood into the subcutaneous tissue. What should the nurse document that the patient has? a. Dyspnea b. Cyanosis c. Diaphoresis d. Ecchymosis

10-15 seconds

A patient has edema of the lower extremities. The nurse is assessing whether it is pitting and to what degree. After pressing the skin against a bony prominence for 5 seconds, the nurse identifies 2+ pitting edema. When did the edema disappear? a. 10-15 seconds b. 20-25 seconds c. 30-35 seconds d. 40-45 seconds

tell the patient to avoid coughing

A patient has just had a liver biopsy. What should the nurse do immediately following this procedure?

subacute pain

A patient has pain in the left arm secondary to coronary insufficiency. This is an example of what type of pain? a. Acute pain b. Chronic pain c. Referred pain d. Subacute pain

Judgment of need by the nurse Patient's condition

A patient is admitted to a medical surgical unit. What factors will determine how frequently vital signs will be assessed? (Select all that apply.) a. Desire of the patient b. Judgment of need by the nurse c. Discretion of the family d. Orders of the health care provider e. Patient's condition

flat on the back

A patient is admitted to the hospital after receiving a blow to the head. The patient begins to show signs of shock. How should the patient be positioned? a. With the head lower than the body b. Flat with the legs elevated c. Flat on the back d. In a side-lying position

constipation

A patient is receiving an opioid narcotic. What common side effect should the nurse be aware of when assessing this patient? a. Addiction b. Vomiting c. Constipation d. Diarrhea

Prone

A physician needs to assess extension of a patient's hip joint. In what position should the nurse place the patient? a. Sims b. Prone c. Lithotomy d. Dorsal recumbent

decrease in respirations from 16 to 14

A patient is receiving epidural analgesics. What should the nurse monitor closely in this patient? a. Temperature elevation to 99.2° F from 98° F b. Increase in pulse rate from 88 to 99 c. Decrease in respirations from 16 to 14 d. Decrease in blood pressure from 120/80 to 110/68

Remove the patient from the sitz bath and return to bed

A patient is recovering from a hemorrhoidectomy and experiences dizziness within 5 minutes when taking a sitz bath. What action should the nurse implement? a. Cover the patient to prevent chilling b. Stay with the patient until the full time for the bath has elapsed c. Remove the patient from the sitz bath and return to bed d. Assess vital signs every 5 minutes during the remainder of the sitz bath

Instruct patient to obtain his or her own specimen

A patient is required to provide a sample of body excretions per physician order. What action can the nurse take when providing proper instructions to lessen the patient's embarrassment?

Pulse deficit

A patient is suspected of having a cardiac arrhythmia. The nurse is concerned with the findings of an apical rate of 88 and a radial rate of 80. What is the term for the difference between these two rates? a. Pulse pressure b. Unequal pulses c. Pulse deficit d. Tachycardia

perform tracheal suctioning

A patient is unable to obtain a sputum specimen by coughing and expectorating. What is the best way for the nurse to collect this specimen?

Pain is subjective for the patient

A patient reports to the nurse that he is experiencing a moderate amount of back pain rated 6 out of 10 on the pain scale. What should the nurse recognize about this assessment? a. Pain is objective for the nurse. b. Pain is easy to recognize. c. Pain is subjective for the patient. d. Pain is easily relieved if found early.

encourage the patient to try gain

A patient tearfully declares the use of relaxation techniques does not work for her. What is the best action for the nurse to implement? a. Give up on the idea b. Encourage the patient to try again c. Assure the patient that not everyone is successful d. Give the patient a sedative

Depression Respiratory dysfunction Decreased GI motility Irritability

A patient tells the nurse he is reluctant to report his pain because he does not want to be a bother. What problems is the nurse aware that unrelieved pain can cause? (Select all that apply.) a. Decreased oxygen demand b. Depression c. Respiratory dysfunction d. Decreased GI motility e. Irritability

Focused

A patient was admitted with a complaint of abdominal pain. Later, the nurse observed the patient demonstrating dyspnea. What type of assessment does this change in condition require? a. Individualized b. Focused c. Specialized d. Systematic

contact the poison control center

A patient who had taken a poisonous substance is brought to the emergency department. What is the first action the nurse should take? a. Give syrup of ipecac b. Contact the poison control center c. Give milk to coat the stomach d. Observe for symptoms

notify the physician that the patient is threatening to leave AMA

A patient who is alert and orientated is threatening to leave the hospital against medical advice. What action should the nurse take?

The 80-year-old woman who is recovering from a stroke is being sent to a skilled care facility. She is concerned about the expense. The nurse can decrease anxiety by explaining that Medicare will cover extended-care facility costs for what period of time? a. A period of 30 days b. A period of 45 days for physical therapy c. A period of 100 days for needed skilled care d. Until she is able to be discharged home

A period of 100 days for needed skilled care

yellow color to the skin

A physician documents that a patient has a sallow complexion. How does the nurse interpret this information? a. Yellow color to the skin b. Blue color to the skin c. Red color to the skin d. Gray color to the skin

Bilirubin

A physician documents that a patient has a scleral icterus. What is the cause of this coloring? a. Bilirubin b. Hemoglobin c. Serum potassium d. Serum magnesium

Bacterial infection

A physician documents that a patient is having purulent drainage from a wound. What does the nurse understand is most likely the cause? a. Ringworm b. Viral infection c. Fungal infection d. Bacterial infection

What is the pathophysiology of a myocardial infarct? a. A portion of the myocardium necroses and scars over. b. The coronary vessels are narrowed during the attack. c. The ischemic myocardium causes pain during the attack but is able to regenerate. d. There is damage to the myocardium but no serious alteration of cardiac output.

A portion of the myocardium necroses and scars over

The nurse is accompanying a group of older adults on a July 4th outing to monitor heat prostration. What factor is related to heat intolerance in the older adult? a. An increase in melanin b. A reduction of perspiration c. A reduction in body temperature d. Increased capillary fragility

A reduction of perspiration

What would be indicative of dysfunctional grieving in an 80-year-old man? a. Despair after 3 months b. Guilt and self-doubt after 4 months c. A sense of disorganization after 6 months d. A sense of depression and despair after 1 year

A sense of depression and despair after 1 year

pruritus

A symptom of itching and an uncomfortable sensation leading to an urge to scratch is known as _____________.

How does a theory differ from a fact? a. A theory proves how different influences affect a particular phenomenon. b. A theory attempts to explain and give some logical order to observations. c. A theory is a collection of facts about a particular phenomenon. d. A theory shows a relationship among facts about a particular phenomenon.

A theory attempts to explain and give some logical order to observations

J tube

A tube inserted into the small intestine through the abdominal wall

Transcutaneous electric nerve stimulation (TENS)

A type of pain control that is managed with a pocket sized battery operated device that provide a continuous mild electrical current to the skin via electrodes

IV fluids

A variety of sterile solutions that the body needs injected directly into the veins

Help to synthesize body compounds like bone and blood, regulate body processes, and extracting energy, carbohydrates, fats, and proteins

Vitamins

Which of the following represents a complementary strand of mRNA: TTT CGC GGG TCG? a. UUU GCG CCC UGC b. UUU GCG CCC AGC c. AAA GCG CCC AGC d. AAA GCG CCC UGC

AAA GCG CCC AGC

Which of the following represents a strand of DNA complementary to TTT CGC GGG TCG? a. UUU GCG AAA UGC b. UUU GCG CCC AGC c. AAA GCG CCC AGC d. AAA GCG CCC UGC

AAA GCG CCC AGC

What are signs of hyponatremia

ABDOM. CRAMPS, AMS, HEADACHE, MUSCLE TWITCHING/TREMORS/WEAKNESS, NAUSEA, SZ'S, CRACKLES, JUGULAR VEIN DISTENTION, PERIPHERAL EDEMA, BOUNDING PULSE, WEIGHT GAIN, ORTHOSTATIC HYPERTENSION

What would have the largest impact on maintaining a positive self-perception? a. Ability to control life's choices b. Financial success attained in life c. Family relationships d. Degree of wellness

Ability to control life's choices

intentional or unintentional mistreatment or harm physical psychological, emotional, or financial, of another person

Abuse

Which of the following is most likely to develop in a patient who hypoventilates because of a chronic respiratory disease, such as emphysema? a. Kussmaul respirations b. A decrease in the plasma concentration of hydrogen ion c. An increase in plasma pH d. Acidosis

Acidosis

How are they paired?

Adenine can only pair with thymine and cytosine can only pair with guanine

Name the bases of DNA

Adenine, Thymine, Cytosine, Guanine

Name the bases of RNA

Adenine, Uracil, Cytosine, Guanine

What is the term for stored fat that insulate the body and serves as a cushion to protect organs?

Adipose tissue

How should the nurse provide written discharge instructions for a patient with macular degeneration? a. Write the instructions in bold print. b. Adjust the table and light to assist the patient to use peripheral vision to read. c. Place written document directly in front of the patient to read. d. Read the document to the patient.

Adjust the table and light to assist the patient to use peripheral vision to read

What is a common reason that an older adult may deny that he has fallen? a. Fear that he will fall again b. Fear of being hospitalized for treatment c. Afraid of being seen as frail and dependent d. Fear of being considered clumsy

Afraid of being seen as a frail and dependent

urinary retention

After a Foley catheter has been removed, the nurse should assess the patient for: a. hemorrhage. b. constipation. c. urinary retention. d. bladder spasm.

warm

After a bone scan, the nurse assesses a hematoma at the injection site of the dye. The nurse should apply ______ soaks or compresses.

What are 3 important substances to avoid in pregnancy ?

Alcohol Caffeine Nicotine

Which hormone causes the kidneys to retain sodium and to excrete potassium? a. ADH b. Erythropoietin c. Aldosterone d. PTH ANS: C

Aldosterone

Which condition is caused by vomiting stomach contents? a. Hyperkalemia b. Hypernatremia and blood volume expansion c. Hypocalcemic tetany d. Alkalosis

Alkalosis

Which of the following is most likely to develop in an anxious hyperventilating patient? a. Respiratory acidosis b. Alkalosis c. A decrease in plasma pH d. Ketoacidosis

Alkalosis

The home health nurse is caring for a patient who has been recently diagnosed with a cognitive disorder. How can the nurse best prepare the family for the patient's care? a. Leave them literature about the disorder. b. Instruct them about the physiological changes that cause the disorder. c. Allow them time for expression of their feelings and grief. d. Discuss options for placement in a long-term care facility.

Allow them time for expression of their feelings and grief.

The home health nurse is assessing the home environment of an 85-year-old patient with Parkinson disease. What symptom of Parkinson disease makes the patient at an increased risk of falls? a. Postural hypotension b. Cognitive changes c. Altered vision d. Altered gait

Altered gait

sleep deprivation

Although denying pain, a patient is irritable, responds slowly, and exhibits periods of tachycardia. What should the nurse assess for in this patient? a. Electrolyte imbalance b. Allergic response c. Sleep deprivation d. Constipation

What does the liver remove from the blood and convert to urea?

Ammonia (NH3)

Air embolism

An abnormal circulatory condition in which air travels through the bloodstream and becomes lodged in a blood vessel

orthopnea

An abnormal condition in which a person must sit or stand to breathe deeply or comfortably is known as ___________________.

Tachycardia

An abnormal condition in which the heart contracts regularly but at a rate greater than 100 beats per minute is known as ___________________

tachypnea

An abnormally rapid rate of breathing that is seen in many disease conditions is known as ___________________.

What is hypernatremia?

An elevated serum sodium that may occur with water loss or sodium gain.

Contusions

An injury that does not break the skin is caused by a blow and is characterized by edema, discoloration, and pain

Visual analog scale

An objective means of assessing pain severity it consists of a straight line representing a continuum of intensity and has visual descriptors at each end

Dehydration

An older adult patient is being assessed for skin turgor. The nurse identifies decreased skin turgor demonstrated by slow return of the skin to the previous position after being grasped and raised. What can the nurse conclude is responsible for this assessment? a. Dehydration b. Edema c. Skin breakdown d. Malnutrition

Name the two conditions in how glucose is broken down

Anaerobic catabolism (absence of oxygen) Aerobic catabolism (presence of oxygen)

sterile syringe tip

Anaerobic organisms tend to grow within body cavities. What will the nurse use to collect an anaerobic specimen?

Which of the following is least descriptive of dehydration? a. Tenting b. Anasarca c. Hypovolemia d. Concentrated plasma

Anasarca

What is the food source for vitamin B12?

Animal products (meat, fish, poultry, milk, dairy, products, and eggs )

What eating disorder is characterized by body image distortion, excessive exercise, and vicarious enjoyment of food?

Anorexia Nervosa

An older adult states that her physician ordered her to drink a glass of wine daily. The nurse understands that alcohol is occasionally recommended for which of the following reasons ?

Appetite stimulant

What is age spots?

Areas most often seen on body areas that are most exposed to sunlight

How should the nurse adapt the environment for a person with right-sided hemianopsia? a. Approach the patient from the right side. b. Arrange personal articles on the left side of the bed. c. Remind the patient to avoid turning his or her head to reduce added perceptual problems. d. Touch the patient on the right side to get his or her attention.

Arrange personal articles on the left side of the bed

inspection palpation auscultation Percussion

Arrange these assessment techniques in correct order of a standard physical examination. Put a comma and space between each answer choice (A, B, C, D, etc.). a. Auscultation b. Percussion c. Inspection d. Palpation

What is DNA called the genetic code?

Arranged in hereditary units called genes

in medicine and nursing, evolution or appraisal of a condition or the process of making such and evaluation, including the patients subjective report of an symptoms and the examiner objective findings of data obtained through laboratory test, physical examination and medical history.

Assessment

How can continuity of care be provided for a demented patient in the hospital setting? a. Keep the patient in the room. b. Reduce environmental stimuli such as the TV or radio. c. Assign care to a consistent group of staff. d. Attach a bed alarm to the patient.

Assign care to a consistent group of staff

What housing option for the older adult offers the privacy of an apartment with restaurant-style meals and some medical and personal care services? a. Government-subsidized housing b. Long-term care facility c. Assisted-living center d. Group housing plan

Assisted Living Center

Why are LDLs known as the "bad cholesterol"?

Atherosclerotic heart disease

closing of the AV valves

Auscultating the heart sounds should result in a "lubb-dupp" sound when using the bell and the diaphragm of the stethoscope. What causes the "lubb" sound? a. Opening of the AV valves b. Opening of the semilunar valves c. Closing of the AV valves d. Closing of the semilunar valves

What is a typical response for friends when dealing with a grieving person? a. Avoid discussing the source of grief. b. Encourage discussion of the source of grief. c. Schedule an intervention. d. Try to resolve the grief.

Avoid discussing the source of grief

epidemic

disease that emerges rapidly at uncharacteristic time or unusual pattern

The nurse is educating a patient on a vegan diet. What supplement will the nurse encourage this patient to take to avoid a deficiency?

B12

Which of the following has the greatest amount of potassium? A. pickles, canned soup B. milk,meat, beans C. banana and orange juice D. egg and toast

Banana and orange juice

What would be the most effective action by the home health coordinator when interacting with an unpaid caregiver?

Be generous with positive feedback

Vitamin a is needed for ?

Breast milk production

The center for speech located in the brain is the __________ area.

Broca

In the 1980s, Medicare initiated a program of diagnosis-related groups (DRGs) to reduce hospital costs. How did the DRGs reduce hospital costs? a. By classifying various diagnoses as ineligible for hospitalization b. By allotting a set amount of hospital days and prospective payment on the basis of the admitting diagnosis c. By specifying particular physicians to treat specified diagnoses d. By using frequency of a particular diagnosis to set a payment schedule

By allotting a set amount of hospital days and prospective payment on the basis of the admitting diagnosis

assessing a palpable carotid pulse during each compression

CPR has been initiated on an adult patient. How will the nurse confirm the effectiveness of CPR? a. Assessing an EKG pattern with each compression b. Assessing a palpable carotid pulse during each compression c. Assuring a compression depth of to 2 inches d. Observing pupils that change from pinpoint to dilated

More than 99% of which cation is stored in the bones and teeth? a. Potassium b. Hydrogen c. Chloride d. Calcium ANS: D

Calcium

Which of the following is essential in forming and maintaining bones?

Calcium

A person who has diabetes mellitus diet is ?

Carbohydrate controlled

Careful attention to carbohydrate consumption can improve metabolic control of diabetes. The nurse teaches a meal planning approach that focuses on the total amount of carbohydrates eaten at a meal. What is this meal planning approach called?

Carbohydrate counting

What is the major source of energy?

Carbohydrates

Which are the energy-providing food groups? (Select all that apply.) a. Carbohydrates b. Fats c. Proteins d. Vitamins e. Minerals

Carbohydrates Fats Proteins

What are the two substances that the body uses for fuel?

Carbohydrates and fats

What elements are amino acids composed of?

Carbon Hydrogen Oxygen *NITROGEN*

What is the end product anaerobic catabolism?

Carbon dioxide & water

Whats the toxicity for potsssium?

Cardiac arrest

What is the nurse closely assessing for in a patient with hypokalemia? a. Systemic edema b. Cardiac complications c. Muscle cramping d. Impaired kidney function

Cardiac complications

Which of the following is a sign of potassium deficiency? A. apathy B. growth failure in children C. dermatitis D. cardiac dysrhythmias

Cardiac dysrhythmias

What phospholipid is used in synthesis of steroids?

Cholesterol

The nurse assesses a diabetic patient and finds the patient to be pale, edematous, and listless with a blood urea nitrogen (BUN) level of 35 mg/dL and a creatinine level of 4 mg/dL. What would the results of the nursing assessment indicate? a. Diverticulitis b. Congestive heart failure c. Chronic renal failure d. Benign prostatic hypertrophy

Chronic renal failure

What is the food source iron?

Clams, liver, oysters, meat, poultry, fish, legumes, whole and enriched grains, fortified cereals

Which of the following is true about interstitial fluid, plasma, lymph, and transcellular fluid? a. Contain no electrolytes b. Protein-free fluids c. Classified as extracellular d. pH less than 6 (highly acidic)

Classified as extracellular

Folate metabolism and blood cell formation intrinsic factor must be present fat absorption

Cobalamin (B12)

The daughter of an 80-year-old woman asks the home health nurse for advice in selecting a cane for her mother, who has an unsteady gait. What cane would be a poor choice? a. Wooden cane with a rubber tip b. Four-footed cane with a rubber grip c. Clear acrylic cane with a nonslip tip d. Colorful carved cane with a wooden tip

Colorful carved cane with wooden tip

What should be considered when planning a health interview with an older adult? (Select all that apply.) a. Comfort of the physical setting b. Methods to develop trust and rapport c. Timing for minimal distractions d. The age and ethnicity of the patient e. Income level

Comfort of the physical setting Methods to develop trust and rapport Timing for minimal distractions The age and ethnicity of the patient

The nurse is reviewing recommended health practices with her old patient. What would be included in the teaching?

Compliance with dietary restrictions Performance of regular exercise daily Cessation of smoking Arrangement of regular medical examinations

hyperthermia

Condition of abnormally high body temp

Antipsychotic medication can be prescribed to control which of the following? a. Disruptive verbal behavior b. Constant yelling and hitting others c. Hallucinations d. Disorientation

Constant yelling and hitting others

What is the toxicity of calcium

Constipation; increased risk for urinary stone formation in men; reduced absorption of iron and zinc

New research indicates that the greatest impact on preventing osteoporosis in older women is

Consuming an adequate calcium intake before and after puberty to help maximize peak bone mass

peripheral parenteral nutrition (PPN)

Contains lesser concentrations of TPN, administered into a peripheral vein

What benefits can the older adult female expect with even a small amount of daily exercise?

Control weight Retard bone loss Control the blood glucose level Promote a sense of well-being

The nurse palpates a very weak pedal pulse in the right foot. What other findings would the nurse expect to find in the right foot and leg? (Select all that apply.) a. Bruising b. Darkened color c. Cool skin d. Diminished hair on limb e. Capillary refill of 3 seconds

Cool skin Diminished hair on limb

Calcium

Positive charge ion found mainly in bones and teeth silvery yellow metal most abundant mineral in the body

Which of the following can attribute to a sleep-wake cycle disturbance in the older adult? a. Increase in angiotensin b. Decrease in insulin c. Increase in growth hormone d. Decrease in melatonin

Decrease in melatonin

Why are older adults more at risk of dehydration?

Decreased consumption of fluids and therfore increased risk for fluid volume feficit

Know the musculoskeletal changes associated with aging?

Decreased fluid in intervertebral disks Decreased bone calcium Decreased blood supply to muscles Decreased tissue elasticity Decreased muscle mass

Why does the nurse modify the environment to keep it warmer for the older adult? a. A change in the metabolic rate b. Decreased subcutaneous tissue c. Changes in the musculoskeletal system d. A weakened peripheral vascular system

Decreased subcutaneous tissue

Which are normal age related changes? Select all that apply

Decreased visual acuity increased gastric pH decreased serum albumin decreased rate of peristalsis

A nurse is caring for a client with protein-calorie malnutrition (PCM). What effect of PCM should the nurse monitor for in the client?

Delayed wound healing

What provisions should be included in the plan of care for a 70-year-old extended-care facility resident who will be self-administering his medications? (Select all that apply.) a. Delivery of adequate supply of medication b. Payment for medication c. Locked medication storage at bedside d. Medication administration record e. Assessment of effectiveness of medication

Delivery of adequate supply of medication Locked medication storage at bedside Medication administration record Assessment of effectiveness of medication

What has replaced the USDA's Recommended Dietary Allowance (RDA)?

Dietary Reference Intake (DRI)

Older adults are at risk for nutritional problems because of

Difficulty in chewing related to loss of teeth

Osmosis

Diffusion(passage) of water through a selectively permeable membrane; the water move from a less to more concentration

Ecchymoses

Discoloration of an area of the skin or mucous membrane caused by the extravasatuon of blood into the subcutaneous tissues

Complications of tube feeding are

Distention Diarrhea Cramping Nausea This indicates the formula strength, volume, or rate is too great; or formula is too cold

What would be the most effective intervention by the nurse when a patient with a cognitive disorder begins to exhibit delusional behavior as a result of excessive stimulation? a. Medicate with a psychoactive drug such as lorazepam or diazepam. b. Send the patient to his or her room for "time out." c. Remind the patient that acting out behavior will not be tolerated. d. Distract the patient with a quiet activity.

Distract the patient with a quiet activity

The home health nurse counsels a family in making safe driving "rules" for their 85-year-old father. Which rule would not be effective in promoting safety? a. Limit driving to nearby areas with easy access. b. Plan ahead and know where you are going. c. Wear prescribed glasses and hearing aids. d. Drive below the speed limit to maintain control of the car.

Drive below the speed limit to maintain control of the car

What are signs of hypernatremia

Dry mouth, dry mucous membranes, thirst, swollen tongue

Perineal area

During a head-to-toe assessment, the nurse assesses the patient's abdomen. Which area should the nurse assess next? a. Chest b. Arms c. Legs and feet d. Perineal area

Cyanosis

During a physical assessment, the nurse notes a patient has a bluish discoloration of the skin and mucous membranes. How should the nurse document this finding? a. Dyspnea b. Cyanosis c. Diaphoresis d. Ecchymosis

Asthenia

During a physical assessment, the nurse notes a patient has a loss of strength and energy. What should the nurse document that the patient is experiencing? a. Dyspnea b. Cyanosis c. Asthenia d. Ecchymosis

Diaphoresis

During a physical assessment, the nurse notes a patient has profuse secretions of sweat. What should the nurse document that the patient is experiencing? a. Dyspnea b. Cyanosis c. Diaphoresis d. Ecchymosis

Diarrhea

During a physical assessment, the nurse notes a patient passes frequent loose liquid stools. What should the nurse document that the patient is experiencing? a. Dyspnea b. Cyanosis c. Diaphoresis d. Diarrhea

Bleeding in the lower intestinal tract

During a physical assessment, the nurse notes that a patient has bright red blood in the feces. What does the nurse recognize as the most likely cause of this bleeding? a. Bleeding in the upper intestinal tract b. Bleeding in the lower intestinal tract c. Bleeding in the entire intestinal tract d. Consumption of cranberry juice

Bradycardia

During a physical assessment, the nurse notes that a patient's heart rate is 56 beats per minute. What should the nurse document that the patient is experiencing? a. Dyspnea b. Cyanosis c. Diaphoresis d. Bradycardia

Coughing

During a physical assessment, the nurse observes a patient experiencing a sudden audible expulsion of air from the lungs. What should the nurse document that the patient is experiencing? a. Dyspnea b. Cyanosis c. Coughing d. Ecchymosis

Constipation

During a physical assessment, the patient complains of difficulty in passing stools. What should the nurse document that the patient is experiencing? a. Dyspnea b. Cyanosis c. Constipation d. Ecchymosis

Advance the catheter into the bladder

During insertion of a Foley catheter, the patient grimaces as the balloon is inflated. What is the immediate reaction of the nurse? a. Withdraw the catheter b. Ask the patient to bear down c. Continue to inflate the balloon d. Advance the catheter into the bladder

Past health history

During the nursing interview, several histories are taken. What is the history that involves data concerning habits and lifestyle patterns? a. Family history b. Environmental history c. Past health history d. Psychosocial history

A dying patient is receiving 2 mg of morphine sulfate every hour. The patient's daughter is worried about oversedation. What does the nurse explain that this small dose of morphine controls? a. Dyspnea b. Pain c. Hallucinations d. Fatigue

Dyspnea

The patient who had a gastrostomy complains to the nurse about frequent episodes of dumping syndrome. What can the nurse recommend to this patient to decrease this problem?

Eat small, frequent meals

What would be an acceptable alteration in the plan of care for a patient with a hiatal hernia who is experiencing gastrointestinal reflux? a. Encouraging the patient to lie down after meals b. Drinking two full glasses of liquid after the evening meal c. Eating smaller, more frequent meals d. Using caffeine drinks to assist with digestion

Eating smaller, more frequent meals

What are the food sources of Vitamin A?

Egg yolks Liver Milk Sweet potatoes Carrots Winter squash Spinach Collards Kale Broccoli Apricots Cantaloupe

What type of thermometer would be the best choice when assessing the temperature of an older adult? a. Electronic thermometer, because it only takes a few seconds to assess temperature b. Oral thermometer, because the presence of dry mucous membranes gives a more valid temperature c. Axillary thermometer, because its position is nearer the heart d. Rectal thermometer, because it is the best indicator of the body core temperature

Electronic thermometer, because it only takes a few seconds to assess temperature

Which type of elder abuse is demonstrated by a health care worker eating a resident's candy without permission? a. Physical abuse b. Neglect c. Emotional abuse d. Self-neglect

Emotional abuse

An 84-year-old female resident with dementia in an extended-care facility rapidly paces the halls and the common areas from right after breakfast to bedtime. What should be included in the nursing plan of care? a. Restrain the resident from pacing. b. Apply a bracelet that sounds an alarm if the resident leaves the building. c. Encourage rest by asking her to sit and have a glass of juice or a snack. d. Pace with her and engage her in conversations.

Encourage rest by asking her to sit and have a glass of juice or a snack

The nurse recognizes that when a patient is unable to consume adequate nutrition by mouth, an alternative route such as a feeding ostomy may be used. What is the proper term for feeding a patient by this method?

Enteral

What does decreasing fluid intake in the late evening prevent? a. Increased digestive processes in the bowel b. Episodes of nocturia c. Gastroesophageal reflux d. Changes in body temperature

Episodes of nocturia

The nurse assesses an area of skin on the patient's upper thigh that is different in appearance than the surrounding skin. What documentation would be the most informative? a. Red area on upper right thigh. Patient denies discomfort b. Erythematous scaly patch 2 2 cm on lateral aspect of right thigh. Patient denies pain c. Painless red patch on right thigh 2 2 cm d. Medium-size red scaly patch on right thigh. 0 drainage. 0 pain

Erythematous scaly patch 2x2 cm on lateral aspect

What is the difference between essential and nonessential amino acids?

Essential cannot be synthesized by the body at a rate sufficient for growth and maintenance, 9 of the 20 amino acids are essential. Nonessential relies on nitrogen being available for the body to synthesize the other 11 needed.

A patient taking a diuretic is assessed by the nurse as having an erratic pulse and muscle weakness. What should the nurse suspect is deficient?

Potassium

The nurse reading the history of a newly admitted 88-year-old man with dementia sees that this resident is prone to catastrophic reactions. What behavior does the nurse expect to see in the patient? a. Excessive emotional reactions b. Combativeness with little stimulus c. Displays of self-destructive behaviors d. Displays of public exposure or sexual advances

Excessive emotional reactions

Mega doses of vitamin C may decrease the resistance of cancer cells to chemotherapy treatment

False

Vitamin E is essential in the formation of several proteins that are needed for the clotting of the blood

False

Vitamin deficiencies of the fat soluble vitamins will develop more quickly than those of the water-soluble vitamins

False

What electrolytes is found primarily in the intracellular fluid and is the major intracellular cation ?

Potassium

The patient complains to the nurse that he feels terrible since he has been taking several different kinds of vitamin preparations. What should the nurse assess for indications of vitamin toxicity?

Fatigue

What's the toxicity of Vitamin A ?

Fatigue Headache N/V Blurred vision Liver abnormalities Bone & Skin Change

Which of the following tests should be performed annually in a 55-year-old woman? a. Fecal occult blood test b. Sigmoidoscopy c. Hearing screening d. Mammogram

Fecal Occult blood test

G tube or Percutaneous Endoscopic Gastrostomy(PEG)

Feeding tube places into the stomach through the abdominal wall

Which of the following complaints by the 80-year-old resident would indicate a need for the nurse to complete a focused bowel assessment on the resident? a. The inability to have a bowel movement every day b. Feeling pressure and fullness in the rectum but is unable to defecate c. Having had one loose stool after breakfast d. Ingestion and flatulence

Feeling pressure and fullness in the rectum but is unable to defecate

What characteristics are typical for a caregiver of an aging family member? (Select all that apply.) a. 32 years of age b. Female c. Having full-time employment d. Having a care recipient older than 70 e. Giving care for an average of 18 years

Female Having full-time employment Having a care recipient older than 70 Giving care for an average 18 years

Where would the nurse place the head of the stethoscope when assessing an apical pulse? a. Third intercostal space at proximal edge of the clavicle b. Fourth intercostal space at the edge of the sternum c. Fifth intercostal space even with the middle of the clavicle d. Sixth intercostal space above the diaphragm

Fifth intercostal space even with the middle of the clavicle

What is the objective of pain control for the dying patient? a. Keep the patient unconscious and relaxed to avoid the perception of pain. b. Delay medication until the patient reports that the pain is intense. c. Find a control level that reduces pain but allows the patient to interact. d. Eradicate pain completely.

Find a control level tht reduces pain but allows the patient to interact

A nurse is caring for a client with high cholesterol. The physician has ordered a megadose of niacin. Which of the following side effects of niacin should the nurse monitor for in the client? (select all that apply) A. flushing of skin B. confusion C. seizures D. liver damage E. hypotension

Flushing of skin Liver Damage Hypotension

How should the nurse begin the health interview in order to most effectively build rapport with the patient? a. Focus on the problems that the patient sees as important. b. Explain the importance of health maintenance. c. Inform the patient of the number of questions that will be asked. d. Reassure the patient that the interview is private.

Focus on the problems that the patient sees as important

An important vitamin for pregnant women to take is ________ to help with normal growth of fetus.

Folic acid

2 to 6 hours

Following an intravenous pyelogram, the nurse should watch the patient closely for a delayed reaction to the dye, usually occurring within ___ to ___ hours following the procedure.

What is the function of calcium

Formation and maintenance of bones and teeth, blood clotting, nerve conduction, muscle contraction

What is the function vitamin K?

Formation of blood clotting factors

What is the food sources of Vitamin D?

Fortified milk Fortified margarine Egg yolks Liver Fish oils Sunlight on skin

Potassium

Found in intercellular fluid and is major intracellular cation (dominant intracellular cation)

The older adult complains of multiple bruises on his extremities. What are the marks the result of? a. Arteriosclerotic changes in the vessels b. Prolonged clotting time c. Fragility of capillary walls d. Reduction of subcutaneous fat

Fragility of capillary walls

When is clear liquid diets used?

Frequently used postoperatively until peristalsis returns and sometimes during episodes of vomiting and diarrhea !

Which of the following is not high in sodium? A. pickles and soup mixes B. garlic salt and ketchup C. sauerkraut and ham D. fresh cranberries and apples

Fresh cranberries and apples

Which of the following of foods retain the most vitamins?

Fresh foods

Which of the following would be selected for a diet high in antioxidants? (Select all that apply.) a. Fruits b. Vegetables c. Organ meat d. Folic acid e. Vitamin D

Fruits Vegetables Vitamin D

a condition in which acidic stomach contents leak backward from the stomach into the esophagus

GERD

Aspirin in long term therapy can irregulate Vitamin C levels causing ?

GI BLEEDING, vitamin k depletion, and loss of iron

What is GERD and how is it treated?

Gastric contents moving backward into the esophagus, small frequent meals and avoid laying down too soon after eating

The nurse is aware that the urge to defecate is increased peristalsis stimulated by the defecation reflex and the __________ reflex.

Gastrocolic

The nurse must tell a 94-year-old resident of a nursing home that his wife has fallen and has been hospitalized with a broken hip. What is the most effective method for the nurse to deliver the news?

Gather all pertinent information that is accurate

How does the body use protein to regulate blood sugar?

Gluconeogenesis

What monosaccharide is measured as a "blood sugar"?

Glucose

Which of the following is a monosaccharide that is measured as the "blood sugar"? a. Glycerol b. Glycogen c. Glucose d. Lactose

Glucose

Name the 3 six carbon simple sugars?

Glucose Fructose Galactose

What is the body's storage form of carbohydrates, usually found in the liver with some storage in the muscles?

Glycogen

What is stored form of carbohydrates?

Glycogen (animal starch)

In what form does humans store glucose?

Glycogen is the form in which humans store glucose

What is the aerobic process that occurs in the cytoplasm?

Glycolysis

What is the deficiency of iodine?

Goiter; enlarged thyroid gland, weight gain, skin and hair changes. CRETINISM: mental and physical retardation of fetus

What are HDLs known as?

Good cholesterol

If a patient is on a blood thinner like Heparin or Coumadin, what type of foods should the nurse teach them to avoid due to the high amounts of Vitamin K?

Green leafy vegetables

What is the food source for Vitamin K ?

Green leafy vegetables Milk Dairy products Liver Meat Egg yolks Green tea(synthesis by intestinal bacteria)

Which emotional response would be expected from a family who is coping with an aging loved one's diminishing abilities and increased care needs? (Select all that apply.) a. Grief b. Anger c. Frustration d. Loss e. Resentment=

Grief Anger Frustration Loss

What's is the function of Vitamin B1?

Growth of cell metabolism Neurologic function

Respiratory rate is most sensitive to the effects of which ion? a. Na+ b. K+ c. H+ d. Cl-

H+

With carbon dioxide retention, CO2 + H2O a. H2O2. b. H+ + HCO3-. c. bicarbonate. d. H2O + H2O.

H+ +HCO3-

Which of the following is a buffer pair? a. PTH, aldosterone b. ADH, ANF c. HCO3-, H2CO3 d. Sodium, potassium

HCO3-, H2CO3

What stool description would indicate constipation? a. Firm stool passed without difficulty every 3 days b. Hard stool passed without difficulty every 2 days c. Soft brown stool passed with difficulty every 2 days d. Hard dry stool passed with difficulty every 3 days

Hard dry stool passed with difficulty every 3 days

A recently widowed woman moved to an assisted living community because of her hypertension and joined a group to learn how to do water color painting with other women her age. Which theory of aging does the nurse assess the patient to be following? a. Jung b. Havighurst c. Erikson d. Newman

Havighurst

Isotonic

Having equal tension designating or of a salt solution; having the same osmotic pressure as blood

A patient with pulmonary edema is receiving furosemide (Lasix), a kaliuretic diuretic. He has lost 6.6 lb over a 48-hr period. Which of the following statements is true? a. He is probably hyperkalemic secondary to the diuretic therapy. b. He has lost 12 L of water. c. His skin is "tenting." d. He has lost 3 L of water

He has lost 3L of water.

A patient with pulmonary edema is receiving furosemide (Lasix), a kaliuretic diuretic. He has lost 6.6 lb over a 48-hr period. Which statement is true? a. His urinary output has decreased. b. He has lost water weight. c. He will become hyperkalemic if the furosemide is continued. d. The loss of 6.6 lb causes "tenting."

He has lost water weight

A nurse is counseling an older man who retires from his work and changes his role status. What will likely result in the man having fewer adjustment problems? a. He is financially secure. b. He has other roles and relationships. c. He is healthy. d. He has a supportive family.

He has other roles and relationships

Which imbalance can cause life threatening cardiac conduction abnormalities

Hpokalemia

Medicare Part C allows eligible persons to receive Medicare benefits via the services of private insurance companies through which of the following? (Select all that apply.) a. Health maintenance organization (HMO) b. Preferred provider organization (PPO) c. Provider-sponsored organization (PSO) d. Private fee for service organization (PFFS) e. Medical service organization (MSO)

Health Maintenance Organization (HMO) Preferred Provider Organization (PPO) Provider- Sponsor Organization (PSO) Private fee for service organization (PFFS)

What factors influence the timing and extent of age-related changes? (Select all that apply.) a. Health maintenance b. Ethnicity c. Heredity d. Attitude e. Environment

Health maintenance Hereditary Environment

What are communication barriers that are encountered when conversing with an older adult? (Select all that apply.) a. Hearing impairment b. Language differences c. Dementia d. Pain e. Decreased mobility

Hearing impairment language differences dementia pain

The nurse explains that a patient with a heart problem should follow a decreased sodium diet. What will this diet help reduce the risk for or prevetn?

Heart attacks

Arrange these common diseases of the older adult in order of their mortality rate. (Separate letters by a comma and space as follows: A, B, C, D) a. Cancer b. Pneumonia c. Stroke d. Chronic obstructive pulmonary disease (COPD) e. Heart disease

Heart disease Cancer Stroke Pneumonia Chronic Obstructive Pulmonary Disease

The 75-year-old woman newly admitted to a long-term care facility seems fearful of her surroundings and is frequently tearful, saying, "I don't know what to do!" What action by the nurse would most help decrease her fear? a. Help the resident identify and verbalize her specific fears. b. Assure the resident that she has nothing to be afraid of. c. Keep the light on in the resident's room 24 hours a day. d. Play quiet music on the resident's radio.

Help the resident identify and verbalize her specific

Which of the following is (are) true of Na+? a. Is the chief extracellular anion b. Helps regulate extracellular volume c. Is regulated by ADH d. All of the above

Helps regulate extracellular volume

Patients who retain an abundance of sodium often have

High blood pressure and edema

Which lipoprotein account for 20% to 30% of all cholesterol in the blood?

High density lipoprotein (HDLs)

What are common diets ?

High fluid intake, high protein, low fat/ fat RESTRICTION, sodium restriction

What type of diet might contribute to hypertension and obesity?

High sodium high fat

Osteoporosis diet

High vitamin d and calcium intake

When conducting a health assessment, which finding would be recorded as crackles? a. High-pitched sounds in the lung bases on inspiration b. Continuous low-pitched snoring sounds over major bronchi c. Squeaky musical sounds on expiration d. Coarse grating sounds on inspiration and expiration

High-pitched sounds in the lung bases on inspiration

What data would the nurse expect to find when performing an assessment of a 90-year-old suspected of having an upper respiratory infection? a. Temperature elevation over 101° F b. Elevated white blood count c. History of recent periods of confusion d. Record of increased fluid intake

History of recent periods of confusion

acid-base balance

Homeostasis of the hydrogen ion (H+) concentration in the body fluids

Which group is incorrect? a. Anions: bicarbonate, chloride, phosphate b. Lines of defense against acid-base imbalance: buffers, lungs, kidneys c. Transcellular fluids: aqueous humor, cerebrospinal fluid, synovial fluid d. Hormones: aldosterone, ADH, ANP, urea

Hormones: aldosterone, ADH, ANP, urea

Improvement of circulation Relief of edema Consolidation of exudates

Hot moist compresses have which positive effect(s)? (Select all that apply.) a. Improvement of circulation b. Relief of edema c. Consolidation of exudates d. Enhancement of scabbing e. Relief of pain

Which of the following are expected standards within various roles in a culture? (Select all that apply.) a. Housing b. Clothing c. Choice of vehicle d. Family size e. Support system

Housing Clothing Choice of vehicle Family size

Hypoventilation is most likely to cause a. acidosis. b. edema. c. renal excretion of bicarbonate. d. hypokalemia.

acidosis

Show the patient how bedside equipment works Give simple explanation of policies Keep family interventions to a minimum

How can the nurse help reduce the stress of a hospital admission? (Select all that apply)

The aquathermia pad has circulating water for temperature control

How does an Aquathermia pad differ from a traditional heating pad? a. The Aquathermia pad can be folded to fit the anatomic location snugly. b. The Aquathermia pad can be placed under the patient. c. The Aquathermia pad has circulating water for temperature control. d. The Aquathermia pad can be left on for as long as 2 hours.

At least every 8 hours

How frequently should the nurse clean the nares of patients who have a nasogastric tube or are receiving oxygen by nasal cannula? a. At least every 2 hours b. At least every 6 hours c. At least every 8 hours d. At least every 10 hours

At least twice a day

How often should the nurse cleanse the meatal-catheter junction of a patient with an indwelling catheter? a. At least once a day b. At least twice a day c. At bedtime d. Each shift

administer medication before ambulation

How should the nurse assist the patient with moving when pain is anticipated? a. Be supportive b. Apply heat before moving them c. Administer medication before ambulation d. Obtain assistance if the patient is heavy

From the meatus outward

How should the nurse cleanse the meatal opening when performing male perineal care? a. From the meatus outward b. With an alcohol swab c. In a circular motion d. With a cotton-tipped applicator

Downward and back

How should the nurse position the ear pinna when using the tympanic thermometer on a child? a. Upward and back b. Parallel c. Downward and back d. Upward and forward

Downward

How should the nurse position the earpieces on a stethoscope to ensure optimum reception? a. Backward b. Parallel to the ears c. Toward the face d. Downward

inserting the upper denture first

How will the nurse correctly replace a patient's dentures after cleaning? a. Inserting the lower denture first b. Asking the patient to insert them c. Inserting both dentures together d. Inserting the upper denture first

When the nurse is inquiring about the patients level of pain, which of the following statements would best support the patient?

How would you describe your pain?

What does urinary specific gravity measure? a. Plasma H+ concentration b. Hydration status of the body c. Plasma potassium d. The buffering capacity of the blood

Hydration status of the body

What does actively transporting electrolytes from an area of higher concentration to an area of lower concentration require? a. Hydrostatic pressure b. Osmotic pressure c. Blood pressure d. Pulse pressure

Hydrostatic pressure

A nurse assesses an edematous cardiac patient. The nurse is aware that this condition is a result of retained fluid. What is the patient considered to be? a. Hyponatremic b. Hypokalemic c. Hypernatremic d. Hypercalcemic

Hyernatremic

Which electrolytes imbalances could lead to formation of renal calculi

HyperCalcemia

What are common causes of respiratory Acidosis

Hypo ventilating, asthma, COPD, phneumonia, sleep apnea

Which type of solution cause the cells to enlarge?(Swell)

Hypotonic

The patient denies smoking, although the smell of tobacco is strong in his hospital room. Which of the following statements by the nurse would be most effective?

I can smell the tobacco, and I see your lighter on the bedside table.

What is the purpose of a health screening? a. Identifies persons with unmet health needs who may need a referral. b. Assesses local health needs for the Public Health Department. c. Collects data that will be used for research. d. Provides appropriate treatment for identified health needs.

Identifies persons with unmet health needs who may need a referral

Stop the transfusion Take and record vital signs Notify physician and blood bank Return blood and tubing to the blood bank Monitor urine output

If a patient has a transfusion reaction, the nurse should perform the following interventions in which priority order? Put a comma and space between each answer choice (A, B, C, D, etc.). a. Take and record vital signs b. Notify physician and blood bank c. Stop the transfusion d. Monitor urine output e. Return blood and tubing to the blood bank

97.2

If a patient has an axillary temperature of 96.2° F, the nurse understands that the true temperature is ______.

One facility to another

If a patient has an order for an interagency transfer where does the nurse explain that the patient will be moved?

jaw thrust

If a spinal injury is suspected, before the rescuer starts CPR, the trachea should be opened with a _______ _______ maneuver.

When administering intravenous (IV) fluids, the nurse ensures that the IV fluids are infusing as ordered to prevent dehydration in an adult. When could dehydration become lethal? a. If the patient loses 5% of body fluid b. If the patient loses 10% of body fluid c. If the patient loses 15% of body fluid d. If the patient loses 20% of body fluid

If the patient loses 20% body fluid

What does the nurse hope to achieve by teaching tai chi daily in the long-term care facility? a. Stimulate intellectual activity b. Encourage interaction c. Improve coordination d. Demonstrate cultural awareness

Improve coordination

What is the primary focus of a fall prevention program in a long-term care facility? a. Improving balance b. Improving muscle mass c. Improving circulation d. Increase in the knowledge base about falls

Improving balance

A patient's toenails are brittle and thick. What other assessment should the nurse be sure to include? a. Respiratory sounds b. Pedal pulses c. History of gout d. Intake of dietary calcium

Pedal pulses

What position would be most appropriate to reduce pressure ulcers in a bedridden patient? a. Directly on his side, with the trochanter bearing the weight b. Supine, with the sacrum and iliac crest bearing the weight c. In a semi-Fowler position, with the sacrum and ischium bearing the weight d. In a lateral position, with body rotated 30 degrees with gluteus bearing the weight

In a lateral position, with body rotated 30 degrees with gluteus bearing the weight

What happens when you increase your respiratory the rate and depth

Increase rapid breathing Hyperventilation 12/20 normal

What is the deficiency vitamin K ?

Increased prothrombin time Hemorrhaging (in severe cases)

What is the toxicity vitamin E ?

Increases tendency to hemorrhage

Clear liquid diets are low in ?

Kilocalories, protein, and most nutrients

An older adult man has been diagnosed as having diminished depth perception. What does the nurse expect him to have difficulty with in his everyday activities? a. Judging the height of steps. b. Reading small print on food labels. c. Reading street signs. d. Seeing in dim light.

Judging the height of steps

When attempting to communicate with a patient who is hearing-impaired, what should the nurse consider? a. Keep the message simple. b. Provide lengthy explanations and information. c. Assume understanding if the patient does not ask for clarification. d. Use many hand gestures.

Keep the message simple

Which condition stimulates Kussmaul respirations? a. Respiratory acidosis b. Respiratory alkalosis c. Ketoacidosis d. Hypocalcemic tetany

Ketoacidosis

Which of the following is least related to the aerobic metabolism of glucose? a. Intramitochondrial b. Ketones c. CO2 + H2O + ATP d. Oxygen

Ketones

What is a symptom of inadequate insulin coverage in a patient with diabetes mellitus type I? a. Diminished urine output b. Ketones in the urine c. Shallow and slow respirations d. Extreme diaphoresis

Ketones in the urine

What two organs are involved in acid base balance

Kidneys Liver

Metabolic acidosis, such as diabetic ketoacidosis, is most likely to cause a. hypoventilation. b. Kussmaul respirations. c. the renal excretion of bicarbonate. d. the renal retention of H+.

Kussmaul respirations

A person who experienced a hemorrhagic cardiac vascular accident (CVA) to the left hemisphere would demonstrate which type of symptoms? a. Language disturbances b. Poor impulse control c. Inappropriate affect d. Confabulation

Language disturbances

What would be an appropriate suggestion for an 80-year-old woman who recently placed a deadbolt lock on her door? a. Keep the door securely locked. b. Apply similar locks on the windows. c. Leave the door unlocked, with the key in place. d. Replace the lock with a security chain.

Leave the door unlocked, with the key in place

The nurse is aware that a person's attitude about aging is influenced mainly by his or her __________. (Select all that apply.) a. life experiences b. income level c. level of education d. current age e. occupation

Life experiences Current ages

What preventative action can be taken to prevent skin trauma from shearing force? a. Slide the patient across the bed linens to change position. b. Apply generous amounts of lotion to the patient's skin. c. Lift the patient on draw sheets when pulling up in bed. d. Give the patient frequent tub baths to soften the skin.

Lift the patient on draw sheets when pulling up bed.

The 84 year old male complains of dry mouth. What would be appropriate for the nurse to suggest?

Limit alcohol intake

4-32

Listening for bowel sounds should be done over all four quadrants of the abdomen using the diaphragm of the stethoscope. What is the normal rate of bowel sounds per min-ute? a. 2-10 b. 3-20 c. 4-32 d. 5-40

The 94-year-old woman has come to the health assessment interview with her 70-year-old daughter, who answers all the interview questions for her mother. What response by the nurse would be the most effective? a. Say, "I'm speaking to your mother. Please let her answer for herself." b. Continue to interact with the daughter to facilitate completion of the interview. c. Look directly at the patient and say, "Mrs. Smith, now I'd like to hear from you about your health." d. Document that all answers to the interview came from a third party.

Look directly at the patient and say, "Mrs. Smith, now I'd like to hear from you about your health."

Ketone body formation is most likely to cause a. infection. b. acidosis. c. bleeding. d. protein deficiency.

acidosis

What would cause the most damage to the self-concept of an older adult who lives in a long-term care facility? a. Being perceived as a single group rather than an individual b. Having individual needs that are not met c. Losing many personal belongings d. Losing social contact

Losing many personal belongings

pancreatitis diet

Low fat , small, frequent feedings

The nurse has assessed a patients body mass index (BMI) to be 19.6. This assessment of weight versus height indicates that this patient's weight category is in which category?

Low health risk

dicerticulitis/colitis/ulcerative colitis diet

Low sodium low fat

Renal insufficiency diet

Low sodium low fluid

What is the food source for B3?

Meat, poultry, enriched and fortified grains and cereals! (High protein foods that contain tryptophan )

What are risk factors for osteoporosis? (Select all that apply.) a. Menopause b. Smoking c. White female d. Excessive high-impact exercise e. Long-term use of phenytoin (Dilantin)

Menopause Smoking White female Long- term use of phenytoin (Dilantin)

When reading the laboratory report of a patient with excessive diarrhea, the nurse notes that the pH is 7.10, and the PaCO2 and the PaO2 are normal. What should the nurse recognize as this patient's state from this information alone? a. Respiratory acidosis b. Metabolic acidosis c. Respiratory alkalosis d. Metabolic alkalosis

Metabolic acidosis

A patient began vomiting and continued to do so for several hours. What is the result of this loss of stomach contents? a. Metabolic acidosis b. Metabolic alkalosis c. Respiratory acidosis d. Respiratory alkalosis

Metabolic alkalosis

adduction

Movement of an extremity toward the axis of the body

Diffusion

Movement of soild particles in a fluid move from an area of higher concentration to an area of lower concentration.

CAB

Mnemonic for assessing status of emergency patient circulation, airway, breathing; used in one or two rescuer CPR

Which of the following is the trigger for the release of ADH? a. Decreased blood volume b. Renin c. Dehydration d. More than one of the above are true.

More than one of the above are true

Which of the following is not true of extracellular fluid? a. Most body water is extracellular. b. Plasma is extracellular fluid. c. Transcellular fluid is extracellular. d. There is more interstitial fluid than intravascular fluid.

Most body water is extracellular

Sodium restricted diet

Most often used in the treatment of hypertension and heart failure but may be used prior to Myocardial infarction or for a patient with chronic renal failure and/or cirrhosis. consists of low sodium and low fat

The nurse is teaching a newly diagnosed diabetic regarding the diabetic diet. Which of the following factors will have the biggest impact on compliance?

Motivation of the patient

abduction

Movement of an extremity away from the midline of the body

The nurse is informing a patient about an upcoming diagnostic procedure. What statement by the nurse would demonstrate effective communication?

Mr. Brown your leg is to be x-rayed in the x-ray department in an hour

The home health nurse conducts a safety assessment in a patient's home. Which of the following would be identified as a fire hazard? a. Baking soda near the stovetop b. A smoke detector in the kitchen c. Multiple appliances plugged into one outlet d. A metal container for cigarettes

Multiple appliances plugged into one outlet

What should the nurse expect when assessing a patient with respiratory alkalosis? a. Slow respirations b. Muscle weakness c. Strong, even heart rate d. Flushed face

Muscle weakness

potassium modified diet

Necessary for patients experiencing renal failure and other kidney diseases. This diet restricts potassium intake which controls the buildup of potassium in the bloodstream

High fiber diet

Needs adequate fluid, a variation of the regular diet often used in treatment of constipation. Fiber should be increased gradually no nuts and seeds (can get stuck in pockets)

Which type of elder abuse is demonstrated by the nonprovision of medical care? a. Physical abuse b. Neglect c. Emotional abuse d. Self-neglect

Neglect

Informed consent

New physician orders are transcribed for a patient to receive a colonoscopy. What must be completed before the colonoscopy to indicate the patient has been given full knowledge about what will be done along with its risks and complications?

Release of energy from carbohydrate, fat, and protein, deficiency may lead to disease known as pelegra

Niacin (B3)

When reviewing a patient's dietary intake, the nurse recommends that sugar consumption be reduced to the recommended daily level. What is this level?

No more than 8% of total daily kilocalories

Which symptom would be a characteristic of a stage I pressure ulcer on an older adult's coccyx? a. Clear blister b. Nonblanchable area of erythema c. Scaly abraded area d. Painful reddened area

Nonblanchable area of erythema

What foods provide alot of energy?

Nutrient dense foods

The young woman who is breastfeeding will need an increase of calories and protein. What foods should the nurse suggest as sources of protein?

Nuts

What is the food sources of magnesium ?

Nuts, legume, whole grains, green leafy vegetables, and fortified cereals

If a patient BMI is 30 and above they are considered ?

OBESE

Which of the following acts as a base? a. H+ b. OH- c. Calcium ion d. Gastric juice

OH-

How is objective data collected? (Select all that apply.) a. Observation b. Patient complaints c. Physical examination d. Laboratory findings e. Family input

Observation Physical Examination Laboratory Findings

Clinics death

Occurs when heartbeat and respiration have ceased

A person with Parkinsons disease has a nursing diagnosis of "nutrition, less than body requirements related to difficulty swallowing." What change would the nurse make in the nursing care plan? a. Feed the patient at each meal. b. Place the patient in a semi-Fowler position for mealtime. c. Offer a thick, high-nutrition shake as a snack. d. Encourage the patient to drink a sip of water after each bite of solid food.

Offer a thick, high nutrition shake as a snack

To stimulate appetite

One reason the nurse focuses on oral hygiene is to maintain a healthy state of the oral cavity. What is another reason to promote oral hygiene? a. To improve self-esteem b. To stimulate appetite c. To restore tooth destruction d. To assist with periodontitis

Excess diuresis is most likely to cause a. edema. b. overhydration. c. blood volume depletion. d. acidosis.

blood volume depletion

What is the sign of calcium deficiency that leads to bone pain and fractures?

Osteoporosis

What is calcium deficiency?

Osteoporosis(adults); weak; more porous bones; stunted growth in children

What is the pathophysiology of emphysema? a. Constriction of the bronchial tree, excessive mucus, and nonproductive cough b. Calcification of the alveoli and a dry cough c. Overinflation of the alveoli, making them ineffective for gas exchange d. Inflammation of the trachea and bronchioles, excessive mucus, and productive cough

Overinflation of the alveoli, making them ineffective for gas exchange

If a patient BMI is between 25-29.9

Overweight

The chemical reaction that occurs in the mitochondria requires what to occur?

Oxygen

The home heatlh nurse suggests to the 82 year old man that he should wear a Medic Alert bracelet. Which of the following health conditions prompted the nurse to make this suggestion?

Pacemaker

Chronic pain

Pain lasting longer than 6 months; can be as intense as acute pain can be continuous or intermittent

referred pain

Pain that is felt at a site other than in the injured or diseased organ or part of the body

The patient complains of feeling the need to urinate and fullness and tenderness in the bladder area. The patient is restless and diaphoretic. What is the most appropriate initial nursing intervention? a. Help the patient into a warm tub bath to stimulate voiding. b. Catheterize the patient. c. Palpate the bladder fundus. d. Place heated towels over the bladder area.

Palpate the bladder fundus

Elevation of VLDL can cause what disease?

Pancreatitis

What is iron function ?

Part of hemoglobin and myoglobin; necessary for oxygen transport and use in the body; part of some enzymes; energy metabolism

What is the function of iodine?

Part of thyroxin, which helps regulate metabolism, growth and development

What is the method by which inhaled oxygen is moved into the intravascular compartment called? a. Active transport b. Oxygenation c. Passive transport d. Mass movement

Passive transport

Which patient would be most at risk for constipation? a. Patient taking antibiotics for an upper respiratory infection b. Patient taking hormones for postmenopausal symptoms c. Patient iron supplements for anemia d. Patient taking nonsteroidal inhalants for chronic obstructive pulmonary disease (COPD)

Patient iron supplements for anemia

Which patient would need enhanced skin care precautions? a. Patient who has one unformed stool after a bolus of tube feeding b. Patient who has an unformed stool followed by a formed stool 3 hours later c. Patient who reports cramping and nausea followed by an unformed stool d. Patient who reports no abdominal discomfort but has had three unformed stools in 8 hours

Patient who reports no abdominal discomfort but has had three unformed stool in 8 hours

Which patient would require special precautions when the nurse is performing a digital rectal exam? a. Patient with chronic obstructive pulmonary disease (COPD) b. Patient with diabetes c. Patient with Parkinson disease d. Patient with congestive heart failure

Patient with congestive heart failure

Low fiber diet

Patients with diverticulitis-pockets are inflamed in the colon.

What is the deficiency of Vitamin B3?

Pellagra, dermatitis, constipation or diarrhea, dementia, and depression

The home health nurse is caring for a patient that has undergone removal of a part of the stomach. For what should the nurse carefully assess this patient?

Pernicious anemia

What ph level indicates for alkalosis

Ph greater than 7.45

What ph level indicates acidosis

Ph less 7.35

The lactating mother is counseled by the nurse to eat adequate amounts of meat and legumes. What level will this help to increase? a. Potassium b. Chloride c. Magnesium d. Phosphorus

Phosphorus

What would be the most effective action of the nurse when assisting an 85-year-old adult with weak abdominal muscles to defecate? a. Encourage the use of a bedpan before getting up in the morning. b. Place a footstool under the feet of the patient when seated on the toilet. c. Insert a finger in the patient's rectum to stimulate the urge to defecate. d. Instruct the patient to do isometric exercises to strengthen the abdominal muscles.

Place a footstool under the feet of the patient when seated on the toilet

-Inform the patient of activity -Roll up the head of the bed -Dangle the patient at the sife of the bed -Apply a gait belt -Assist the patient to stand

Place the nursing activities in priority order for the preparation of a patient to ambulate. Put a comma and space between each answer choice (A, B, C, D, etc.). a. Dangle the patient at the side of the bed b. Apply a gait belt c. Assist the patient to stand d. Inform the patient of activity e. Roll up the head of the bed

What would be the most acceptable intervention for the nurse dealing with a patient with a threat of aspiration? a. Perform deep tracheal suctioning of the patient every hour. b. Place the patient in a side-lying position, with the head turned to the side. c. Slightly elevate the foot of the bed, with the patient in a supine position. d. Give the patient only thickened fluids.

Place the patient in a side-lying position, with the head turned to the side

blood pressure

Pressure exerted by the circulating volume of blood on the arterial walls, vein, and chambers of the heart

The nurse modifies the care plan for the immobilized patient after assessing a calcium level of 6.2 mEq/L. What nursing assessment should the nurse include when modifying this care plan? a. Osteoporosis b. Tooth loss c. Renal calculi d. Contractures

Renal calculi

What should the nurse explain when discussing expected changes in the female reproductive system to an older adult?

Production of vaginal secretions decreases

Name the biologic theories

Programmed theory run out of program theory rate of living theory gene theory error theory somatic mutation theory crosslink theory (connective tissue theory) Clinker theory Wear and tear theory neuroendocrine theory

What are some causes of metabolic alkalosis?

Prolonged vomiting with depletion of ECF and chloride (Hypochloremic metabolic acidosis), GI suctioning

How does HDLs help lower cholesterol?

Promote removal of cholesterol from the blood (reduce the risk)

How does gluconeogenesis assist in restoring the blood sugar back to normal?

Protein is converted to glucose in the liver and released into the blood thereby restoring blood sugar to normal

A 75-year-old resident in a long-term care facility has problems with sleep onset. What would be an appropriate nursing intervention to aid with sleep? a. Provide a back rub at bedtime. b. Provide a heavy snack at bedtime. c. Coach the resident in 10 minutes of exercise before bedtime. d. Provide a cola drink at bedtime.

Provide a back rub at bedtime

What can a nurse do to encourage a normal bowel movement in an older adult? a. Encourage the patient to decrease fluid intake. b. Provide a warm beverage at breakfast. c. Administer a mild laxative at bedtime. d. Provide a warm shower each morning.

Provide a warm beverage at breakfast

An older adult has expressive aphasia. What would be the most effective intervention for the nurse to use to improve the patient's communication? a. Provide flash cards with text and pictures. b. Be patient and ask him to repeat himself. c. Encourage him to practice slow speech. d. Arrange with him to blink the eyes once for "yes" and twice for "no."

Provide flash cards with text and pictures

Which stage of grief may last up to 2 years? a. Disorientation b. Shock and numbness c. Reorganization d. Searching and yearning

Reorganization

What food provides a natural laxative effect? a. Orange b. Pineapple c. Prunes d. Raw apple

Prunes

What is the correct term for the difference between the apical pulse and the radial pulse? a. Pulse pressure b. Pulse deficit c. Pulse ratio d. Pulse quality

Pulse deficit

The patient tells the home health nurse that he has flung himself out of bed three times in the course of a violent nightmare. What sleep disorder does the nurse suspect? a. Myoclonus b. Restless legs syndrome c. Rapid eye movement (REM) sleep disorder d. Epilepsy

Rapid eye movement (REM) sleep disorder

The nurse clarifies that the biologic theory that proposes that aging is based on the using up of a finite number of breaths or heartbeats is the __________ of __________ theory

Rate of Theory

A 70 year old male patient has been newly diagnosed with hypertension. What would be the most effective health practice to reduce sodium intake?

Read food labels on food containers carefully

erythema

Redness or inflammation of the skin or mucous membranes that is the result of dilation and congestion of superficial capillaries is known as _________________.

What age-related changes in the respiratory system explain why the older adult is at risk for infection? (Select all that apply.) a. Reduced ciliary movement b. Decrease in alveolar elasticity c. Pooling of secretions d. Flattened diaphragm e. Calcification of costal cartilage

Reduced ciliary movement Decrease in alveolar elasticity Pooling of secretions

What is a common feeling for an older adult who is placed in a long-term care facility? a. Rejection b. Safety c. Making a fresh start d. Immediate assistance at hand

Rejection

Why does the home health nurse give his 90-year-old patient a framed poster that says "We need each other."? a. Insure that the patient will take care not to fall. b. Remind the patient to ask for assistance when needed. c. Encourage the patient to take pride in his independence. d. Reinforce that the patient should not attempt any activity without help.

Remind the patient to ask for assistance when needed

A friend asks the nurse what could be done to improve the chance of a long life. Using current biologic theories of aging, the nurse recommended that her friend discuss the first with her physician, but advises that the approach more likely to cause harm than good is which one ?

Replacing of hormones, such as HGH, DHEA, and estrogen

A patient is admitted with severe emphysema and a PO2 of 85. He also has a blood pH of 7.25 and a serum bicarbonate of 40 mEq/liter. Which of the following is the accurate description? a. Metabolic acidosis and respiratory compensation b. Metabolic alkalosis with a respiratory compensation c. Respiratory alkalosis with a renal compensation d. Respiratory acidosis with a renal compensation

Respiratory acidosis with a renal compensation

A patient admitted in a state of extreme anxiety has vital signs of: T 98.6° F, P 81, BP 130/86, R 32. What will result if this hyperventilation continues? a. Metabolic acidosis b. Metabolic alkalosis c. Respiratory acidosis d. Respiratory alkalosis

Respiratory alkalosis

Biological death

Results from permanent cellular damage caused by lack of oxygen

The recently widowed 65-year-old man has told the nurse that he is so lost without his wife that he is planning to retire, sell his home, and move to a retirement village in another state. What assessment by the nurse would be the most correct? a. Positive, because it represents a new beginning b. Positive, because it allows him more time to resolve his grief c. Risky, because he is giving up significant supports to his self-image and grief resolution process d. Negative, because he has not thought his plan through

Risky, because he is giving up significant supports to his self-image and grief resolution process

What is the deficiency of Vitamin D?

Rockets (children): abnormal shape and structure of bones Osteomalacia (adults) weakening and softening of bones

The nurse is aware that the older adult is at greater risk for hypothermia than a younger person because the older adult has a diminished ability to: a. convert glycogen to glucose. b. select appropriate clothing or bed linen. c. shiver. d. constrict vessels.

SHIVER

Epistaxis digitorum

Self inflicted local digital trauma (from nose picking)

What is the food sources for sodium?

Salt, processed foods, saurakrat, pickles , soup, ham, and ketchup

What is an appropriate response by the nurse to a demented resident in a long-term care facility who becomes combative when being prepared for a shower? a. Call for assistance to complete the shower. b. Say, "I understand you don't want a shower, so I'll give you a sponge bath." c. Medicate the patient with a sedative and complete the bath when the patient is more cooperative. d. Say, "Okay. It's your right to remain dirty."

Say, "I understand you dont want a shower, so I'll give you a sponge bath."

The 80-year-old woman newly admitted to a long-term care facility complains of intense itching in her axillae and antecubital fossa. There are small red lesions in linear patterns. What condition does the nurse suspect? a. Rosacea b. Keratosis c. Pruritus d. Scabies

Scabies

What is the deficiency of vitamin C?

Scurvy

A 75-year-old male is worried that his wartlike dark macules with distinct borders are melanomas. What would be the most likely cause for the macules? a. Senile lentigo b. Cutaneous papillomas c. Seborrheic keratoses d. Xerosis

Seborrheic keratoses

What observation would cause the nurse to suspect cataracts in the older adult? a. Holds the newspaper a good distance away while attempting to read small print b. Seeks an area in a room that is free from glare in order to read the newspaper c. Holds a hand over one eye while attempting to read small print d. Uses only peripheral vision while attempting to read a newspaper

Seeks an area in a room that is free from glare in order to read the newspaper

The nurse cautions the Certified Nursing Assistants (CNAs) to use care when transferring or handling older adults. The nurse understands that the vascular fragility of the older adult can result in which of the following conditions? a. Altered blood pressure b. Pressure ulcers c. Pruritus d. Senile purpura

Senile Purpura

What is base sequencing?

Sequence or pairing of the bases in a strand of DNA or RNA carrier of genetic code

What is the deficiency of potassium ??

Severe cardiac dysrhythmias

What is the definition of diffusion

Shbstances/ particles moves from high concentration to low concentration

The totally competent 76-year-old female with terminal cancer is fatigued and tearful about the ineffectiveness of her treatment. She tells the nurse that she wishes she had never started it but now feels obligated to continue. What would be the most appropriate explanation by the nurse regarding the treatment? a. Once treatment has begun, the doctor should decide about any changes. b. She may change her mind about treatment at any time. c. Decisions about treatment should be made by the person who is her medical power of attorney. d. Cessation of treatment will shorten her life.

She may change her mind about treatment at any time

dyspnea

Short of breath or difficulty in breathing

Objective data

Signs that are perceived by an examiner and can be seen, heard, measured, or felt are known as ___________ _________.

What are signs and symptoms of respiratory alkalosis

Signs: Cramps Light headache Tachypnea Hyperpnea Carpopedal spasms Symptoms: Dry mouth Dizziness Bloating Tingling in the arms Confusion Numbness Muscle spams

Which of the following facilities would be recommended for a patient who has had a hip replacement and needs physical therapy? a. Basic care facility b. Skilled care facility c. Subacute care facility d. Assisted-living residence

Skilled care facility

What is seborrheic keratosis?

Skin disorder in which lesions ranging in color from light to tan to black appear as slightly raised wartlike macules with distinct edges

The nurse would question the order for a sedative for a patient with which of the following diagnoses? a. Chronic obstructive pulmonary disease (COPD) b. Any form of dementia c. Hypertension d. Sleep apnea

Sleep apnea

Most diuretics increase urine production by blocking the effects of which cation? a. Calcium b. Bicarbonate c. Chloride d. Sodium

Sodium

Select all of the following true statements regarding vitamins: A. some vitamins are lost by exposure to air or during food storage B. frozen foods retain the most vitamins C. high temperatures can destroy vitamins D. the body will store excess amounts of all vitamins

Some vitamins are lost by exposure to air or during food storage. Frozen foods retain the most vitamins High temperatures can destroy vitamins

total parenteral nutrition (TPN)

Specifically formulated solution; nutritionally complete to meet specific needs of patient when GI tract is not functioning properly. GIVEN CENTRA LINE ONLY

Vitamin D, hormones, and cholesterol is examples of what?

Steroids

What are the two roles of glycogen?

Stores help regulate blood sugar. Acts as storage energy in skeletal muscle

How do Kegel exercises aid in the treatment of incontinence? a. Increases the tone of the bladder b. Reduces urinary retention c. Strengthens the urinary sphincter d. Sensitizes biofeedback

Strengthens the urinary sphincter

What should be avoided by an 82-year-old man with rosacea? (Select all that apply.) a. Stress b. Dairy products c. Sun exposure d. Spicy foods e. Alcohol consumption

Stress Sun exposure Spicy foods Alcohol consumption

Which group is incorrect? a. Ions: sodium, potassium, calcium, bicarbonate, chloride b. Lines of defense against acid-base imbalance: buffers, lungs, kidneys c. Extracellular fluid: intravascular, interstitial, transcellular d. Substances that are normally found in a sample of urine: urea, creatinine, glucose

Substances that are normally found in a sample of urine: urea creatinine, glucose

What is a nursing intervention to decrease the thirst of a patient who is on a fluid restriction?

Sucking on occasional ice chips

Cardiac arrest

Sudden cessation of functional circulation

Gate control

Suggest that pain impulses can be regulated or even blocked by gating mechanisms located along the central nervous system

The nurse makes nutrition a focus in the care plan. Where does nutrition play the most important role?

Sustained appetite

What is the food sources of potassium?

Sweet potatoes, fruits, vegetables, fresh, meat, legumes, milk , whole grain cereal

Which of the following foods should be avoided in a person who was told by their doctor to limit the sodium in their diet? (Select all that apply) A. table salt B. prepackaged frozen foods C. yogurt, cucumbers, carrots D. potato chips, pretzels

Table salt Prepackaged, frozen foods Potato chips, pretzels

What are some common causes of hyperventilating

Tachycardia Nausea Vomiting Light head Head trauma Phenomena

How would an elderly female be most likely to reduce the amount of the free radical lipofuscin? a. Avoid animal fat b. Take antioxidants daily c. Build muscle mass d. Perform outdoor exercise three times weekly

Take antioxidants daily

Which of the following would NOT indicate understanding of your client's knowledge regarding ways to obtain iron in their diet?

Take their calcium and iron supplement together at the same time everyday

168.75

The 125-pound nurse assesses the weight of a patient. What weight is the heaviest the nurse may safely lift by herself? a. 158.75 lb b. 168.75 lb c. 178.75 lb d. 188.75 lb

An 80-year-old teaches Sunday school each week and delivers food for Meals on Wheels. What theory of aging would apply? a. Newman's developmental b. The life course c. The activity d. The disengagement

The activity

Morgan therapeutic lens

The appliance that connects to an IV drip and delivers a continuous irrigation to the eye is known as a ________ _________ _________.

What does an advance directive indicate to the home health nurse? a. The degree of intervention desired for life support b. Who is to manage medical decisions in case of debilitating illness c. Who will manage finances in case of debilitating illness d. The mortuary to be used in the case of death

The degree of intervention desired for life support

93.2 degreese

The emergency department nurse quickly assesses the temperature of an unconscious patient who has been outside all night in below-freezing temperatures. What temperature is the nurse aware of that can lead to death? a. 95.2° F b. 93.0° F c. 93.2° F d. 90.8° F

Twist and Lift

The most common cause of musculoskeletal disorders in nurses involves a movement that requires the nurse to ________ and _________ at the same time.

Medications Rehabilitation techniques Referral to community agencies Medical equipment to be used

The nurse adheres to the discharge standards set by the The Joint Commission (TJC), which include that patients will receive instruction regarding which aspect(s) of care? (Select all that apply)

cool the burn immediately

The nurse arrives on the scene of a fire. What is the first thing the nurse will do for a burn victim? a. Apply dressings b. Cover with a blanket c. Cool the burn immediately d. Apply topical ointment

3 seconds

The nurse assesses a patient for capillary refill after the fingernail is compressed for 5 seconds. What should the nurse expect the refill time to be? a. 1 second b. 2 seconds c. 3 seconds d. 4 seconds

-Immobility -Risk for impaired circulation -Risk for skin impairment -Incontinence

The nurse assesses a patient in a Posey safety reminder device (SRD) for which problem(s) that may increase because of the use of SRDs? (Select all that apply.) a. Immobility b. Lethargy c. Risk for impaired circulation d. Risk for skin impairment e. Incontinence

II

The nurse assesses a red blister over the right superior iliac area of a patient. What stage is this decubitus ulcer? a. I b. II c. III d. IV

Dyspnea

The nurse assesses respirations of a patient demonstrating pursed-lip breathing, flared nostrils, and retractions. How will the nurse describe these respirations? a. Tachypnea b. Stertorous c. Dyspnea d. Cheyne-Stokes

106

The nurse assesses the blood pressure as 192/86, noting that the patient has a pulse pressure of ________.

30-degree lateral

The nurse attempts to avoid a pressure ulcer for a bedridden patient by turning the patient frequently. What is the most favorable position for the nurse to move this patient into? a. Back-lying b. Full lateral c. 30-degree lateral d. Full prone

Friction

The nurse avoids dragging the patient across the bed linen to decrease the potential risk of skin injury by _________.

incapacitating

The nurse conducting a seminar on bioterrorism reviews several types of agents that may be used as weapons. An agent that does not seriously damage or kill the target population but only impairs it is classified as _____________.

the need for some type of exercise

The nurse counsels the immobilized patient in regard to prevention of muscle atrophy and contractures. What will the nurse be sure to include when counseling this patient? a. The need for additional calcium b. The need for additional protein c. The need for some type of exercise d. The need for a special protective bed

If cardiopulmonary arrest has existed for no more 4 minutes

The nurse determines clinical death and initiates CPR immediately. How long is resuscitation considered possible? a. If cardiopulmonary arrest has existed for no more 2 minutes b. If cardiopulmonary arrest has existed for no more 3 minutes c. If cardiopulmonary arrest has existed for no more 4 minutes d. If cardiopulmonary arrest has existed for no more 5 minutes

press the area gently to assess for blanching

The nurse discovers a reddened area over a patient's hip. What should be the nurse's first intervention? a. Cover the area with an occlusive dressing b. Apply mild ointment with a cotton-tipped applicator c. Press the area gently to assess for blanching d. Rub gently to increase circulation

those that interfere with electrical

The nurse explains that electrocardiograms are graphic representations of electrical impulses generated by the heart. What type of abnormalities can an electrocardiogram identify?

0.75

The nurse explains that the measurement of radiation exposure is in multiples of Gy. The number of Gy an individual may absorb before becoming ill with radiation syndrome is _______.

20 ro 30 minutes

The nurse explains that the purpose of a sitz bath is to reduce inflammation in the perineal and anal area. What is the least amount of time the nurse will instruct for a sitz bath? a. 10 to 15 minutes b. 20 to 30 minutes c. 30 to 40 minutes d. 1 hour

blocks pain impulses

The nurse explains that transcutaneous electrical nerve stimulation (TENS) provides a continuous mild electrical current to the skin. How does the TENS unit act to reduce pain? a. Distracts the patient b. Blocks endorphin production c. Warms the skin d. Blocks pain impulses

blue

The nurse has an order to perform occult blood testing on a patient's emesis. What color will the sample turn to indicate that the test is positive for occult blood?

It distributes workload more evenly

The nurse instructs a nursing assistant to use large muscle groups when lifting. What is the rationale for this instruction? a. Workers' compensation claims will be prevented b. Big muscles work more effectively c. It guarantees no muscle strain d. It distributes workload more evenly

water

The nurse instructs a patient receiving home O2 therapy to drink plenty of fluids to help keep bronchial secretions liquefied. What is the recommended fluid? a. Milk b. Water c. Tea with artificial sweetener d. Coffee

medulla oblongata

The nurse is alarmed when a patient with a severe head injury of the occipital lobe has a respiratory rate of 10 breaths per minute. Where might this finding indicate that there is an injury? a. Cerebellum b. Medulla oblongata c. Cortex d. Cerebrum

septicemia

The nurse is alert for a serious condition called ___________ that results from pathogens being introduced into the blood stream

Nursing diagnosis

The nurse is assessing a patient for collection of subjective and objective data. What will this data provide the basis for making? a. Care plan b. Medical diagnosis c. Nursing assessment d. Nursing diagnosis

WEAK THREADY PULSE

The nurse is assessing a patient who is severely bleeding and at risk for hypovolemic shock. What can the nurse anticipate? a. Slow, labored breathing b. Hot, flushed skin c. Edematous extremities d. Weak, thready pulse

Pressure ulcer

The nurse is assessing a patient's skin for signs of impaired skin integrity. Which finding by the nurse is considered a major manifestation? a. Burn b. Laceration c. Pressure ulcer d. Infection

latino men feel it is unmanly to admit to pain

The nurse is assessing pain reported by a Latino male patient. What is important for the nurse take into consideration when observing objective data? a. Latino men are suspicious of female caregivers. b. Latino men have a cultural bias against use of narcotics. c. Latino men believe pain is necessary for cure. d. Latino men feel it is unmanly to admit to pain.

Independence

The nurse is assisting a patient to perform personal hygiene. What is the most important focus of the nurse when assisting this patient? a. Nursing care b. Independence c. Repetition d. Performance

leave the toes exposed

The nurse is assisting a victim of an accident who requires bandaging of the right lower extremity. What should the nurse do when applying the bandage? a. Use sterile material b. Leave the toes exposed c. Bandage the extremity tightly d. Bend the knee after bandaging

apply direct pressure

The nurse is attempting to control bleeding in a patient with a profusely bleeding scalp wound. What is the most effective initial treatment of this bleeding? a. Elevate the head b. Apply direct pressure c. Apply an ice pack d. Apply indirect pressure

REST

The nurse is aware that the state at which a person is mentally relaxed, free from worry, and is physically calm is __________.

Gently washing the leg and patting dry with a towel

The nurse is bathing a patient with a deep vein thrombosis in the left leg. What modification will the nurse make when attending to the left leg? a. Washing the leg with long, firm strokes and drying with a towel b. Omitting washing the leg at all c. Gently washing the leg and patting dry with a towel d. Applying lotion in long, smooth strokes

Speak slowly and clearly to the patient

The nurse is caring for a patient experiencing presbycusis. What intervention should the nursing personnel be instructed to implement? a. Speak quickly to the patient b. Speak in loud tones to the patient c. Speak slowly and clearly to the patient d. Tell the patient they must purchase a hearing aid

gag reflex has returned

The nurse is caring for a patient following a bronchoscopy and maintains NPO status for 2 hours. What additional assessment will indicate to the nurse that this patient's risk for aspiration has decreased?

Every 120 minutes

The nurse is caring for an unconscious patient with a risk for skin impairment. How often will the nurse plan to change the position of this patient? a. Every 30 minutes b. Every 60 minutes c. Every 120 minutes d. Every 180 minutes

Sign

The nurse is collecting data during an initial assessment. What can be seen, heard, measured, or felt and is objective? a. Symptom b. Observation c. Sign d. Assessment

Nursing assessment

The nurse is developing a nursing care plan for a newly admitted patient. What is the first step the nurse will take in developing this care plan? a. Health history b. Review of systems c. Family history d. Nursing assessment

introduce herself/himself

The nurse is meeting a patient for the first time. What is the first thing the nurse will do to initiate a nurse-patient relationship? a. Appear interested b. Introduce herself/himself c. Provide support d. Communicate trust

Region

The nurse is obtaining a history of a patient's present illness. The PQRST system is used for the interview. What does the R stand for in this system? a. Random b. Region c. Result d. Recent

sonorous wheezes

The nurse is performing auscultation of breath sounds on a respiratory patient. The sounds heard on inspiration and expiration are low-pitched, coarse, gurgling, and have a snoring sound. What best identifies these sounds? a. Crackles b. Plural friction rub c. Rhonchi d. Sonorous wheezes

No cooler than 68 F

The nurse is preparing to bathe a patient. What should the room temperature be set at? a. No warmer than 67° F b. No cooler than 68° F c. No cooler than 70° F d. 75° F or warmer

date and time of collection room number insurance information

The nurse is preparing to collect a urine specimen. What will this nurse include when labeling this specimen? (Select all that apply.)

-Remove spread and blanket seperately -PLACE BATH BLANKET OVER PATIENT ON TOP SHEET Position patient to far side of bed

The nurse is preparing to make an occupied bed. What procedure will the nurse follow to correctly complete this task? (Select all that apply.) a. Remove spread and blanket separately b. Place soiled sheet at end of bed c. Place bath blanket over patient on top sheet d. Slide mattress to bottom of bed e. Position patient to far side of bed

Flashlight Gloves Thermometer Scissors

The nurse is preparing to perform a physical assessment. What essential supplies should this nurse gather? (Select all that apply.) a. Flashlight b. Gloves c. Red pen d. Thermometer e. Scissors

place ice on the nose and pinch the nostrils

The nurse is teaching a patient with epistaxis about the best way to control bleeding. What information will the nurse relay to this patient? a. Place ice on the nose and pinch the nostrils b. Maintain a flat position c. Pack nostrils with cotton d. Lean backward

I believe you are in pain

The nurse is trying to establish an effective relationship with a patient in pain. What is the best statement for the nurse to make when beginning the assessment? a. "I'll check to see if you can have anything." b. "Let me give you a backrub and see if it helps." c. "I believe you are in pain." d. "When was your last medication for pain?

4

The nurse is using a pain scale of 0 to 10 to assess pain in a postoperative patient. What is considered the maximum pain level at which a patient can usually function effectively? a. 2 b. 3 c. 4 d. 5

30 degrees

The nurse lowers the bed to place the patient on the bedpan. The angle of the head of the bed should be raised to: a. 20 degrees. b. 45 degrees. c. 90 degrees. d. 30 degrees.

Depression

The nurse must be sensitive to an older adult patient experiencing separation anxiety when admitted to the hospital. When a child experiences separation anxiety they will usually cry. What will an older adult often demonstrate when experiencing separation anxiety?

Hold away from the uniform

The nurse must follow the principles of medical asepsis while making a patient's bed, including procedures for handling linens. How should the nurse handle soiled linens? a. Place on the floor b. Fan in the air c. Hold away from the uniform d. Place at the end of the bed

Stop the infusion

The nurse notes an edematous area around the insertion site of an IV that is cool to the touch and the skin of which appears blanched. Based on these assessment findings, what is the first priority of the nurse? a. Apply warm compresses to the area b. Notify the charge nurse c. Stop the infusion d. Reposition the arm to improve the fluid flow

pulmonary

The nurse notes that a patient has difficulty breathing in the supine position, and the patient admits that he sleeps in a recliner at home. These are cardinal signs of ____________ disease.

Arterial flow

The nurse observes that an older adult patient has no hair on the lower legs. The nurse should assess further for the sufficiency of _________ ________.

subjective data

The nurse obtains information from a patient about the site, severity, and duration of the pain. What type of data is this considered? a. Patient data b. Objective data c. Focused data d. Subjective data

less than 6 months

The nurse reassures a patient that most acute pain is intense and of short duration. How long does can acute pain usually last? a. 1 week b. Less than 6 months c. At least 9 months d. More than 1 year

-Pain -Sensation -Color -Skin temperature

The nurse receives a patient from the recovery room following total hip replacement surgery. What will the nurse include when assessing neurovascular status on this patient? (Select all that apply.) a. Pupils b. Pain c. Sensation d. Color e. Skin temperature

every 15 minutes

The nurse should administer Telepaque in preparation for a cholecystogram. How frequently will the nurse administer one tablet of Telepaque before this procedure?

increased heart rate from 82 to 94 ' increased muscle tension perspiration on upper lip facial grimacing

The nurse should administer an analgesic to an unconscious patient after observing which signs? (Select all that apply.) a. Increased heart rate from 82 to 94 b. Decreased systolic blood pressure c. Increased muscle tension d. Perspiration on upper lip e. Facial grimacing

gives the patient some control

The nurse teaches noninvasive pain relief techniques, such as guided imagery, biofeedback, and relaxation. What is the primary advantage of these techniques? a. Can be done any time b. Does not require a nurse c. Gives the patient some control d. Is most effective

Review of systems

The nurse uses a systematic method for collecting data on all body systems, including normal functioning and any noted changes. What is this method? a. Nursing interview b. Review of systems c. Nursing assessment d. Health history

Damage to body cells

The nurse uses cooling techniques to keep the body temperature below 105° F. What can result from an elevated temperature? a. Excessive thirst b. Excessive perspiration c. Damage to body cells d. Increased heart rate

Whether improvement is occurring

The nursing assessment of a pressure ulcer includes size, depth, pain, odor, and color of tissue. What does this evaluate? a. Treatment needed b. Effectiveness of implementation c. Whether improvement is occurring d. Need for additional interventions

What assessment finding would be indicative of delirium? a. The onset of the behavior was rapid. b. There is no change in the level of consciousness. c. The absence of disorientation. d. The absence of hallucinations.

The onset of the behavior was rapid

TENS: Transcutaneous Electrical Nerve Simulator

The pain relief intervention that stimulates large cutaneous nerve fibers to "close the gate" is the _________ unit.

circulatory shock

The patient arrived at the emergency department in pain and bleeding profusely with the following vital signs: BP 80/54, P 102, RR 22. What does the nurse recognize that these symptoms indicate? a. Inadequate perfusion b. Circulatory shock c. Massive vasodilation d. Heart failure

prone

The patient has undergone a lumbar puncture. What position will the nurse place the patient in for up to 12 hours to avoid discomfort from postpuncture spinal headache?

wrap the hands in hot moist towels

The patient is admitted to the emergency department, having suffered frostbite to the hands, which are grayish-white in color. What action should the nurse implement when attempting to warm the hands? a. Have the patient rub the hands together briskly b. Wipe the hands vigorously with a warm towel c. Run tepid water over the hands to warm slowly d. Wrap the hands in hot, moist towels

check fingers for capillary refill

The patient is brought to the emergency department after having fractured an arm 12 hours ago. The arm is very edematous from the fingers to the elbow, and the patient cannot move it. What should be the initial action of the nurse? a. Test range of motion b. Take the vital signs c. Place ice packs on the arm d. Check fingers for capillary refill

10 minutes

The patient is to be catheterized for residual urine. The nurse must perform this catheterization within how many minutes following voiding?

When using an interpreter to speak with an 84-year-old Chinese patient, on what should the nurse focus

The patient not the interpreter

RN

The patient should be assessed as soon as possible after admission. Who performs this initial assessment? a. Physician b. Charge nurse c. LPN/LVN d. RN

Which patient would be a candidate for hospice care? a. The patient with a life expectancy of only 12 months. b. The patient who agrees to life support. c. The patient who agrees to palliative measures. d. The patient who has a treatable illness.

The patient who agrees to palliative measures

remove cold packs and fan

The patient with heat stroke has been undressed and treated with cold packs and a fan. The patient's temperature is now down to 101.2° F. The patient starts to shiver. What action should the emergency department nurse take? a. Raise the head of the bed b. Offer warm liquids c. Remove cold packs and fan d. Continue with cooling interventions

immobilize the knife with dressings and tape

The patient's lower chest has been punctured with a knife that is still in place. What should the nurse's first action be? a. Remove the knife b. Apply an airtight dressing over the wound c. Place the patient in a modified Trendelenburg position d. Immobilize the knife with dressings and tape

What is the best predictor that a hypertension patient will comply with a newly prescribed low-sodium diet?

The patient's previous ability to lose 15lbs through following a weight reduction program

Unrelieved severe pain

The patient's pulse is below 60. The nurse is aware that the patient is not receiving digoxin. What does the nurse suspect is causing the bradycardia? a. Low exercise tolerance b. Unrelieved severe pain c. Excessive bed rest d. A prone position

Tilted with the food of the bed down

The physician orders a patient to be placed in the reverse Trendelenburg position. How should the nurse place the bed? a. On the floor b. Parallel with the floor c. Tilted with the head of the bed down d. Tilted with the foot of the bed down

Weigh patient at the same time each day Encourage patient to void before being weighed Ensure same amount of clothing is worn by patient

The physician orders daily weights on a patient residing in a long-term care setting. What actions should the nurse implement to assess weight accurately? (Select all that apply.) a. Weigh patient at the same time each day b. Schedule weighing immediately after breakfast c. Encourage patient to void before being weighed d. Ensure same amount of clothing is worn by patient e. Calibrate by setting scale at zero after each weight

30 minutes

The procedure for collecting a sterile urine specimen via a catheter port includes clamping the Foley catheter tubing below the catheter port. How long will the clamp remain in place?

Side of the finger

The process for collecting a blood specimen for measuring blood glucose levels begins by asking the patient to hold the selected arm at his or her side for 30 seconds. From what anatomic location is the specimen obtained?

What is transcription?

The process of making RNA from DNA

What is translation?

The recognition of mRNA by the tRNA in the cytoplasm

The daughter of a resident in a long-term care facility is frustrated with her 80-year-old mother's refusal to eat. Which response would be the most appropriate? a. The refusal to eat is an effort to maintain a portion of independence and self-direction. b. The refusal to eat is an indication of approaching Alzheimer disease. c. The refusal to eat is an effort to gain attention. d. The refusal to eat is an indication of the dislike of the institutional food.

The refusal to eat is an effort to maintain a portion of independence and self-direction

The nurse would recognize successful aging according to Jung's theory when a long-term care facility resident demonstrates which of the following behaviors? a. The resident takes special care to dress for dinner in a manner that pleases his tablemates. b. The resident asks permission to sit on the patio with other residents. c. The resident asks persons in his hall if his television is bothering them. d. The resident wears a large cowboy hat at all times because he likes it.

The resident wears a large cowboy hat at all times because he likes it

Inflammation is a protective response

The signs and symptoms of both infection and inflammation include erythema, edema, and pain. What is considered the major difference between infection and inflammation? a. Inflammation is a result of bacteria. b. Inflammation is a protective response. c. Inflammation is a disease process. d. Inflammation produces tissue damage.

The patient asks the nurse to describe the neuroendocrine theory of aging. What would be an appropriate response by the nurse? a. It relates thyroid function to age-related changes. b. Adrenal corticosteroids inhibit the aging process. c. The stimulation and/or inhibition of the hypothalamus causes age-related changes. d. The adrenal medulla inhibits epinephrine, causing age-related changes.

The stimulation and/or inhibition of the hypothalamus causes age-related changes

What is the major objection to the disengagement theory? a. The theory justifies ageism. b. The theory addresses the diversity of older adults. c. The theory does not clarify the aging process. d. The theory diminishes the self-esteem of the older adult.

The theory justifies ageism.

Prevent aspiration when eating

The wife of a patient with a cuffed tracheostomy asks why the cuff is inflated intermittently. What is the purpose of the inflated cuff? a. Prevent regurgitation after meals b. Hold the trachea open until it is completely healed c. Dilate the tracheal opening for passage of secretions d. Prevent aspiration when eating

12 pints

The worried mother of an accident victim asks the nurse how much circulating blood an average adult male is supposed to have.What will the nurse reply? a. 8 pints b. 10 pints c. 12 pints d. 14 pints

What reason is often stated by nurses today for not seeking careers in gerontology? a. The physical work is too difficult. b. Their technical skills are not used. c. There is too much challenge. d. There are limited options for employment.

Their technical skills are not used.

Risk factors

There are four categories of factors that increase an individual's vulnerability to develop a disease: genetic, physiological, age, and lifestyle. What is the term for these factors? a. Risk factors b. Causative factors c. Etiologic factors d. Hazardous factors

Why might a professional health care provider neglect to educate patients about end-of-life care? a. They fear that patients will perceive that they are giving up. b. They do not want to influence the patient in any decision. c. They want to keep up the patient's morale. d. They believe that death is a personal failure on their part.

They believe that death is a personal failure on their part

What is true of direct questions?

They can get minimum response answers of yes and no

The nurse is providing information about high cholesterol levels. What is the rationale for avoiding saturated fats?

They increase blood cholesterol

The severe deficiency of which vitamin can cause beriberi

Thiamine

Growth, cell metabolism, appetitie, and neurologic functioning; severe deficiency may lead to disease known as beriberi

Thiamine (B1)

monitor fetal heart tones

What should the nurse do when preparing the patient for an amniocentesis?

verify if the patient has been taking anticoagulants

What should the nurse do when preparing the patient for an arteriography?

"Baby boomers" is a term used to classify which of the following persons? a. Those who entered school in 1945 b. Those who served in the military in World War II c. Those who were born between 1946 and 1964 d. Those who were eligible for Social Security benefits in 2000

Those who were born between 1946 and 1964

The nurse teaches a patient who has a nonfunctioning or dysfunctional GI tract that total parenteral nutrition (TPN) will be infused. Where will the infusion occur?

Through the superior vena cava

What is the toxicity of iron?

Tissue damage, constipations, decreased zinc absorption, accidental poison in children, nausea, vomiting

The nurse is educating a group of high school students regarding nutrition. How should the nurse respond when the students ask what occurs when protein, mineral, iron, and fat combine?

Tissue is built and repaired

Why should an 86-year-old man with a terminal illness be encouraged to file an advance directive? a. To demonstrate understanding of his imminent death b. To comply with most hospital policies c. To clarify treatment protocols d. To spare his family the burden of making end-of-life decisions

To spare his family the burden of making end of life decisions

Why is it important for the home health nurse to interview an 82-year-old patient following the patient's fall in the home? a. So that the incident can be reflected in the home health nurse's documentation b. To help the patient gain insight into the cause of the fall c. In order to guarantee no further falls d. To collect data for research purposes

To help the patient gain insight into the cause of the falt

Why would a nurse lead a group of postmenopausal women on a daily 15-minute "walking tour" through the long-term care facility? a. To improve bone strength b. To orient them to their surroundings c. To improve their socialization d. To increase their appetite

To improve bone strength

1600: Patient reports sharp pain in left chest radiating to neck. Morphine sulfate 5 mg administered IM in right deltoid.

To share assessment findings and pain relief interventions, which documentation sample is the most helpful? a. 1600: Patient reports chest pain. Medicated with morphine sulfate. b. 1600: Patient reports sharp chest pain. Morphine sulfate given IM. c. 1600: Patient reports sharp pain in left chest radiating to neck. Morphine sulfate 5 mg administered IM in right deltoid. d. 1600: Patient requested medication for pain in left chest. Morphine sulfate 10 mg PO given.

The nurse becomes tearful at the death of a patient. What would be the most appropriate response of the nurse? a. Leave the room so that the family will not witness the unprofessional behavior. b. Touch the hand of the daughter and say, "We'll miss your dad." c. Become occupied with rearranging a floral bouquet until emotions are under control. d. Ask family members to leave the room so the post mortem care can be provided.

Touch the hand of the daughter and say, "We'll miss your dad."

How should the nurse approach a patient who has profound hearing loss? a. Knock on the door before entering. b. Touch the patient on the hand to gain attention. c. Give the patient a list of interventions that the nurse plans to perform. d. Speak in a higher tone of voice.

Touch the patient on the hand to gain attention

When the dying patient becomes delirious, what should the nurse encourage the family to do? a. Leave the room and wait outside until the delirium clears. b. Hold the patient's hand, but say nothing, because hearing stays intact until death. c. Remain near the bed and speak to the patient in loud tones to stimulate the patient. d. Touch the patient, call the patient by name, and speak in reassuring tones.

Touch the patient, call the patient by name, and speak in reassuring tones.

Which group is incorrect? a. Cations: sodium, potassium, calcium b. Acid-base imbalances: acidosis, alkalosis c. Lines of defense against acid-base imbalance: buffers, lungs, kidneys d. Transcellular fluids: aqueous humor, cerebrospinal fluid, plasma

Transcellular fluids: aqueous humor, cerebrospinal fluid, plasma

Which of the following occurs first? a. Reading the mRNA code by tRNA b. mRNA leaves the nucleus through the nuclear pores c. Transcription d. Translation

Transcription

With which process are both DNA and mRNA involved? a. Transcription b. Translation c. Glycolysis d. ATP production

Transcription

Older adults and people with dark skin should consume extra vitamin D from Vitamin D fortified foods or supplements

True

This substance is composed of glycerol and three fatty acids. a. Glycogen b. Polypeptide c. Steroid d. Triglyceride

Triglyceride

What are the lipids that are most commonly found in the body?

Triglycerides Phospholipids Steroids

Antioxidant vitamins are thought to help prevent certain types of cancer?

True

The nurse hears a high whistling noise coming from the hearing aids that are lying on the bedside table of the sleeping patient. What would be the most appropriate action by the nurse? a. Replace the hearing aids in the patient's ears. b. Turn off the hearing aids. c. Place the hearing aids in a drawer to prevent loss. d. Ask that an audiologist be notified of the problem.

Turn off the hearing aids

An unconscious diabetic patient with a blood sugar of 450 mg/dL has an elevated temperature and is experiencing Kussmaul respirations. You suspect that a. he is excreting large amounts of bicarbonate in his urine. b. his PO2 is normal or high-normal. c. his blood pH is 7.25. d. Two of the above are true.

Two of the above are true

A strand of DNA reads ATTCGCAGG. A strand of mRNA reads a. TAGGCGTCC. b. UUGGCGCUC. c. UTTCGCTCC. d. UAAGCGUCC.

UAAGCGUCC

With which base can adenine pair? a. Guanine b. Cytosine c. Ribose d. Uracil

URACIL

What is the food source for vitamin b1?

Unrefined whole grains Enriched and fortified grain and cereals Liver Pork Legumes Nuts

in the hospital safe

Upon admission, the nurse notes that a patient without family members present has a billfold filled with cash. Where can the nurse suggest the money be placed?

Which of the following is a nitrogen-containing waste product? a. Glucose b. Adenosine triphosphate c. Carbon dioxide d. Urea

Urea

An alert, independent, 89-year-old male resident with congestive heart failure has been on Lasix for a week. Over the past 2 days he has been frequently incontinent and does not make it to the bathroom in time. He tells the nurse that he is having spasms in his lower abdomen. What diagnosis would best apply to this resident? a. Impaired urinary elimination b. Functional urinary incontinence c. Stress urinary incontinence d. Urge urinary incontinence

Urge urinary incontinence

What would be the initial choice of interventions to help prevent a fall in a confused 85-year-old extended-care facility patient? a. Use of a vest restraint b. Use of an electronic sensor alarm c. Placement of a wheelchair between the wall and dining table d. A tray table attached to the arms of the wheelchair

Use of an electronic sensor alarm

How should the nurse interview a 90-year-old Chinese woman who is accompanied by her daughter? a. Use direct, short questions. b. Address all the questions to the daughter. c. Use pictures of body systems rather than anatomical terms. d. Use social conversation and indirect questions.

Use social conversation and indirect questions.

What type of light may be difficult with cataracts?

VISION in bright light or glare may be particularly difficult with certain types of cataracts

Density of underlying tissue

Various techniques are used by the nurse when performing a physical assessment. One of these techniques is percussion. What is percussion used to determine? a. Sounds for auscultation b. Data about physical features c. Changes in structural integrity d. Density of underlying tissue

What is the food sources of Vitamin E?

Vegetable oils Dark green leafy vegetables Wheat germ Nuts

What is the function of vitamin A ?

Vision, epithelial tissue integrity, growth, reproduction, embryonic development immune function

An older adult has recently moved to a long-term care facility. The family asks what can be done to ease the transition. What would be the most appropriate suggestion of the nurse? a. Send cards or gifts instead of personal visits. b. Visit and call often to remind the resident that she or he is cared for. c. Limit contact for several weeks to encourage independence. d. Communicate with the long-term facility's staff to inquire about the resident's well-being.

Visit and call often to remind the resident that she or he is cared for

What vitamins are needed for ossification?

Vitamin A & C

What vitamins are necessary to support ossification? (Select all that apply.) a. A b. B6 c. C d. D e. E

Vitamin A,C,D

A nurse is assessing a client with retarded growth. Which of the following vitamin deficiencies should identify as responsible for the client condition?

Vitamin B6 deficiency

Aids in the formation of collagen, enhances absorption of iron is an effective antioxidant

Vitamin C

Scurvy is related to a deficiency to which vitamin?

Vitamin C

A nurse is caring for a client who has lactose intolerance. The client has been avoiding all milk, products in the diet. What supplementation should the nurse suggest in the diet of such a client?

Vitamin D

.How would you determine an accurate weight for a patient?

Water balance

allow the patient to drink water before the test

What should the nurse do when preparing the patient for an exercise tolerance test (treadmill)?

What is best exercise needed to keep minerals in your body?

Weight bearing activities

lift the patient up in bed

What action should the nurse implement when assisting a postoperative patient with pain control and comfort? a. Pull the patient up in bed b. Lift the patient up in bed c. Tighten constricting bandages d. Restrict fluid and dietary intake

perform evaluation of outcome goals

What action should the nurse take when evaluating the effectiveness of new or revised therapies for pain relief? a. Observe the patient performing activities of daily living b. Observe the patient's facial expressions c. Frequently assess subjective data d. Perform evaluation of outcome goals

Patient's preferences

What does the nurse recognize is important to consider when using the nursing process to plan hygiene care of the patient? a. Nurse's orders b. Physician's orders c. Patient's preferences d. Outcome goals

Lung sounds

What does the nurse use the diaphragm of the stethoscope to best assess? a. Carotid sounds b. Lung sounds c. Vascular sounds d. Low-pitched sounds

Obtaining a health history

What essential part of the admission procedure is performed by the RN?

18

What gauge needle should be selected by the nurse when preparing to administer blood? a. 25 b. 22 c. 21 d. 18

The patient says, When I came to the hospital yesterday, everything got confused." What would be the best response by the nurse?

What happened

Nurse

What health care professional has the responsibility for notifying the physician when laboratory and diagnostic studies deviate from the norm?

food

What should the nurse encourage the patient to consume when preparing for an electroencephalogram (EEG)?

using a transfer belt

What implementation might the nurse use to improve safety during a transfer? a. Weighing the patient first b. Using a transfer belt c. Putting shoes on the patient d. Supporting a flaccid arm

Turn pot handles on stoves away from reach

What important safety precaution should the home health nurse teach parents in order to prevent burns to small children? a. Never leave them unattended b. Turn pot handles on stoves away from reach c. Turn hot water on first when filling the bathtub d. Keep side rails up on the crib

encourage water intake before the test

What intervention should the nurse implement when preparing the patient for a glucose tolerance test (GTT)?

2 hours

What is considered to be the minimum number of hours of daily activity necessary to prevent the negative consequences of immobility? a. 2 hours b. 4 hours c. 6 hours d. 8 hours

if patient is left-handed

What is important for the nurse to determine in order to decrease the risk for injury to a patient? a. If patient can read English b. If patient is left-handed c. If patient is able to eat unassisted d. If patient can dress independently

use a variety of pain relief methods

What is the best approach for a nurse to use when planning pain relief measures? a. Use a variety of pain relief methods b. Use only nonopioid analgesics c. Use at least three alternating methods d. Use only one method at a time

chronic

What is the defining term for continuous or intermittent pain that does not serve as a warning of tissue damage? a. Acute b. Unrelieved c. Chronic d. Subacute

98.6 F

What should the water temperature be when preparing a tepid bath for a patient? a. 98.6° F b. 100.2° F c. 104.8° F d. 110.4° F

Carotid pulse

What site should be selected if a peripheral pulse needs to be assessed quickly? a. Radial pulse b. Brachial pulse c. Carotid pulse d. Pedal pulse

transferring

What skills should health care workers frequently attend in-services about to ensure that staff has competent skills and risk for falls can be decreased? a. Bathing b. Feeding c. Transferring d. Ambulating

Type C

What type of fire extinguisher should the nurse use when the oxygen concentrator machine malfunctions and causes an electrical fire? a. Type A b. Type B c. Type C d. Type D

slow the infusion

What is the nurse's first priority when a patient receiving IV fluid therapy shows an increase in blood pressure and has bilateral crackles? a. Raise the head of the bed b. Slow the infusion c. Turn the patient to the left side d. Notify the charge nurse

Lower gastrointestinal tract

What is the probable source of bright red blood in the stool?

Contraction of the left ventricle

What is the pulse—the expansion and contraction of an artery— produced by? a. Contraction of the right atrium b. Contraction of the right ventricle c. Contraction of the left atrium d. Contraction of the left ventricle

to determine health teaching required

What is the rationale for the nurse to assess a patient's knowledge of an ordered procedure?

lumbar muscle group

What is the site of the most common strain injury acquired by the nurse when working? a. Trapezius muscle group b. Thoracic muscle group c. Lumbar muscle group d. Thigh muscle group

Apices

What is the suggested sequence for a systematic approach to begin auscultating the thorax? a. Anterior thorax b. Apices c. Left lateral thorax d. Right lateral thorax

remittent

What is the term for a fever that rises and falls but does not return to normal until the patient is well? a. Constant b. Intermittent c. Remittent d. Elevated

active

What is the term for range of motion (ROM) when it is performed by the patient? a. Assisted b. Passive c. Active d. Coordinated

Midstream specimen

What is the term for the cleanest part of a voided urine specimen that is collected after voiding is initiated and before it is finished?

internal respiration

What is the term for the exchange of carbon dioxide and oxygen that takes place at the alveolar level? a. Tachypnea b. Internal respiration c. External respiration d. Bradypnea

Hypothalamus

What part of the body maintains a balance between heat production and heat loss, regulating body temperature? a. Thymus b. Thyroid c. Hypothalamus d. Adrenal glands

nursing personnel

What profession has the highest workers' compensation claim rates of any occupation or industry? a. Firefighters b. Truck drivers c. Law enforcement d. Nursing personnel

open any outside windows

What should a nurse do when encountering a mercury spill? a. Vacuum the spill b. Open interior doors c. Close all outside windows d. Open any outside windows

The nurse has feelings of concern

What should a patient interview being conducted by the nurse convey to the patient? a. The nurse has feelings of concern. b. The nurse has limited time. c. The nurse is very intelligent. d. The nurse has answers to problems.

is allergic to iodine

What should the nurse assess the patient for before administration of contrast media?

Neurologic status

What should the nurse begin by assessing when performing a head-to-toe assessment? a. Support system b. Skin integrity c. Pain level d. Neurologic status

widen the base support in the direction of movement

What should the nurse do to reduce the effort of moving a heavy object? a. Bring the feet close together and flex the knees b. Keep the back straight and bend at the waist c. Widen the base of support in the direction of movement d. Broaden the base of support and twist toward the direction of movement

fill the cup half full

What should the nurse do when offering a cup of hot coffee to a frail, older adult patient? a. Give the patient a straw b. Dilute the coffee with cold water c. Fill the cup half full d. Offer a bib or an apron

informed the patient that a clicking noise will be heard during the scan

What should the nurse do when preparing the patient for a brain scan?

reassure the patient that he or she will be able to breathe during the procedure

What should the nurse do when preparing the patient for a bronchoscopy?

Cleansing the first 2 inches of the catheter with soap and water every shift

What would be the correct explanation of catheter care? a. Cleansing the first 2 inches of the catheter with soap and water every shift b. Disinfecting the entire catheter with alcohol every shift c. Lubricating the catheter with antiseptic lotion every 24 hours d. Cleansing the meatal-catheter junction every 24 hours

incident report

When a fall occurs, the nurse should document the incident and initiate a(n) ___________ report.

The heat can damage epithelial cells

When a patient asks if he can keep the heating pack on his leg all the time, the nurse reminds him of which of the following complication of long-term heat application? a. The heat can cause extreme vasoconstriction. b. The heat can increase the possibility of infection. c. The heat can cause the blood pressure to increase. d. The heat can damage epithelial cells.

Cerumen Impaction

When a patient complains of progressive hearing loss, crackling and ringing noises in his ear, and progressive ear pain, what should the nurse assess for? a. A dead battery in the patient's hearing aid b. Cerumen impaction c. Sinus congestion d. A middle ear infection

A discharge against medical advice form

When a patient demands to be discharged without a physician's order and is leaving the unit with his belongings, what should the nurse ask the patient to sign?

Are you choking

When a patient suddenly experiences respiratory difficulty in the cafeteria, the nurse begins assessment for foreign body airway obstruction. What is the most appropriate question to ask the victim? a. "What did you swallow?" b. "Are you choking?" c. "Are you OK?" d. "Can I help you?"

Triage

When administering first aid in emergency situations, the nurse must first survey victims for severity of injuries. What term correctly describes this process? a. The Good Samaritan law b. An emergency interview c. Triage d. Taking vital signs

Erythema

When admitting a patient to the hospital, the nurse notes the patient has mild sunburn. How should the nurse document this finding? a. Dyspnea b. Cyanosis c. Erythema d. Ecchymosis

Fear of the unknown

When admitting a patient to the hospital, the nurse observes that the patient is distracted and tense. What does this behavior suggest as a common reaction to hospitalization?

2 L/min

When an older adult patient with chronic emphysema comes to the emergency department in respiratory distress, at what rate should the nurse begin oxygen per nasal cannula? a. 2 L/min b. 3 L/min c. 4 L/min d. 5 L/min

Another nurse

When an order for eye irrigation is received, to whom can the nurse delegate the procedure to? a. The patient b. Another nurse c. A nursing assistant d. A family member

Family history of illness Diet Smoking Exercise

When assessing a female for risk factors associated with coronary artery disease, what information should the nurse include? (Select all that apply.) a. Family history of illness b. Diet c. Smoking d. Exercise e. Number of pregnancies

Erythema and blisters

When assessing a patient who has suffered a burn injury, the nurse classifies the burn as a deep partial-thickness burn. What is this observation most likely based upon? a. Painful reddened skin b. Charred skin with milky-white areas c. Erythema and blisters d. Erythema, pain, and swelling

Liver

When assessing a patient with hepatitis, the nurse notes a yellow tinge to the patient's skin. What does the nurse understand as the most likely cause of the jaundice? a. Heart b. Liver c. Brain d. Intestines

Jaundice

When assessing a patient, the nurse notes a yellow tinge to the patient's skin. How should the nurse document this finding? a. Dyspnea b. Cyanosis c. Jaundice d. Ecchymosis

Skin pallor

When assessing a patient, the nurse notes that the patient has an unnatural paleness of color to the skin. How should the nurse document this finding? a. Skin pallor b. Pruritus c. Sallow skin d. Jaundice

Orthopnea

When assessing a patient, the nurse notes that the patient is unable to lie flat to breathe. When the nurse assists the patient into a sitting position, the patient is able to breathe more easily. What should the nurse document that the patient is experiencing? a. Dyspnea b. Cyanosis c. Jaundice d. Orthopnea

Pruritus

When assessing a patient, the patient complains of an uncomfortable sensation leading to an urge to scratch. The nurse notes the patient scratches frequently. How should the nurse document this finding? a. Dyspnea b. Cyanosis c. Jaundice d. Pruritus

Age Sex Emotion Temperature

When assessing factors that may influence the patient's pulse rate, what should the nurse take into consideration? (Select all that apply.) a. Age b. Sex c. Emotion d. Temperature e. Religion

carotid

When assessing the adult victim for pulselessness, the CPR rescuer should palpate the most reliable and accessible pulse. Which pulse will be palpated? a. Radial b. Brachial c. Carotid d. Femoral

Tachycardic

When assessing vital signs on a 40-year-old male, the nurse identifies a pulse rate of 120. What is this pulse interpreted as by the nurse? a. Normal b. Bradycardic c. Arrhythmic d. Tachycardic

Cough

When auscultating the chest, a nurse hears crackles in both lower lobes. To further assess this finding, the nurse should ask the patient to ______________.

2

When collecting a stool specimen for a guaiac (occult blood in stool), the nurse should take a specimen from _____ different parts of the stool.

Appropriate interventions

When collecting data related to the present illness, the nurse must obtain detailed and comprehensive data. What does this data help to establish? a. A nursing diagnosis b. A nursing care plan c. Appropriate interventions d. Nursing orders

Chronic

When discussing diabetes with a patient, the nurse describes this disease as falling into which group in terms of duration? a. Acute b. Organic c. Chronic d. Functional

It drains semi-liquid stool

When explaining the difference between a colostomy and an ileostomy, the nurse explains which of the following about an ileostomy? a. It is always permanent b. It drains semi-liquid stool c. It has a much larger stoma d. It does not need a pouch

discards

When initiating a 24-hour urine collection, the nurse asks the patient to void. The nurse then _______ the specimen.

Apply the cuff approximately 2 inches above the antecubital fossa. Apply the cuff snugly

When instructing a primary caregiver about keeping a daily log of blood pressure readings, what instructions should the nurse include? (Select all that apply.) a. Take the reading at different times during the day. b. Apply the cuff approximately 2 inches above the antecubital fossa. c. If unable to get a reading the first time, immediately reinflate the cuff. d. Assess pulse with the bell of the stethoscope. e. Apply the cuff snugly.

Place the tip of the syringe loosely in the ear canal Introduce fluid with a slow, gentle irrigation

When irrigating an ear, the nurse should perform which intervention(s)? (Select all that apply.) a. Heat the water to 115° F b. Pull the auricle back firmly and hold it c. Place the tip of the syringe loosely in the ear canal d. Introduce fluid with a slow, gentle irrigation e. Use a stronger flow if a foreign body is present

When a bleeding tendency is present

When must the nurse remember to use an electric razor when shaving a patient? a. When a bleeding tendency is present b. When there is a risk for suicide c. When the facial hair is fine d. When speed is essential

never release the tourniquet

When other methods have failed to stop the bleeding and the victim's life is in danger, the rescuer at the scene applies a tourniquet to a young woman's leg above the knee. What is another step that is essential for the rescuer to follow? a. Never release the tourniquet b. Wrap the tourniquet around the limb twice c. Mark the patient with a "T" d. Leave the limb elevated

0.5 to 1

When performing CPR on an infant, the breastbone is depressed approximately ____ to ____ inch(es).

neurologic assessment

When performing a nursing physical assessment, the nurse uses a head-to-toe approach. Where will the nurse begin when using this method? a. Skin assessment b. Neurologic assessment c. Circulatory assessment d. Respiratory assessment

Deep palpation

When performing a physical examination of a patient, the nurse uses a technique that is particularly useful in identifying areas of tenderness or masses of the abdomen. What is this technique? a. Auscultation b. Deep palpation c. Light palpation d. Percussion

decrease environmental stimuli

When preparing a patient for sleep, diming the lights and decreasing the noise levels are examples of nursing interventions. What are these interventions designed to do? a. Mimic usual sleep patterns b. Decrease environmental stimuli c. Prepare the patient for sleep d. Provide for more rest

sweep

When reinforcing the PASS acronym for fire extinguisher use, the nurse reminds the staff that the final "S" stands for ______________.

before activity

When should a nurse administer prescribed analgesic medication when treating a postoperative patient? a. Before activity b. Only when requested by the physician c. Only when requested by the family d. Only when requested by the patient

Shortly after admission

When should discharge planning begin?

How is a peptide bond formed?

When the amine group (NH2) of one amino acid joins with the acid group (-COOH) of a second amino acid

-keep the leg nearest the patient behind the patient's knee -use a gait belt

When the nurse ambulates with a patient who has left-sided weakness, what actions should the nurse take? (Select all that apply.) a. Walk on the patient's right side b. Keep the patient away from heavy furniture c. Hold the patient's arm securely d. Keep the leg nearest the patient behind the patient's knee e. Use a gait belt

back blows

When treating an infant choking on a foreign body, the nurse should use a combination of ______ _________ and chest thrusts.

2 & 30

When two nurses perform two-person CPR, there should be _____ slow breaths for every _____ compressions.

When do the conditions of a living will go into effect? a. When the patient declares that desire in writing b. When a family member indicates the desire for curative therapy to cease c. When two physicians agree in writing that the criteria in the living will have been met d. When the physician and a family member agree that the criteria in the living will have been met

When two physicians agree in writing that the criteria in the living will have been met

Social Services

Where can a nurse refer the family of a patient to find a source of financial aid to meet medical expenses?

assessing vital signs

Where does the nurse recognize that many institutions are now including pain assessment in implementing patient care? a. The initial assessment b. Discharge planning c. Assessing vital signs d. Care planning

close to the body midline

Where should the nurse place the load when carrying heavy objects? a. In a low position b. To the side of the body c. Close to the body midline d. With another's assistance

Measles Pneumonia Tuberculosis Acquired immunodeficiency syndrome

Which are infectious diseases? (Select all that apply.) a. Measles b. Pneumonia c. Hay fever d. Tuberculosis e. Osteoarthritis f. Acquired immunodeficiency syndrome

observing the skin for abnormalities

Which guideline should be followed when giving a backrub? a. Observing the skin for abnormalities b. Massaging for at least 10 minutes c. Following massage with a brisk alcohol rub d. Conversing with patient continually throughout the backrub e. Using alcohol-based lotion for disinfection

Insert the catheter without suction

Which of the following is an appropriate nursing measure when performing tracheostomy care? a. Wear clean gloves b. Insert the catheter without suction c. Suction for 1 minute before removing the catheter d. Place the used catheter in a plastic shield for later use

fecal impaction

a collection of feces in the rectum in the form of a mass that becomes so large or hard that the patient is unable to pass it voluntarily

Continuous Passive Motion (CPM)

_________________________________machines flex and extend joints to mobilize them passively without the strain of active exercises.

The biological theory of aging uses a genetic perspective and suggests that aging is a programmed process in which a. each person will age exactly like those in the previous generation b. a "biological clock" ticks off a predetermined number of cell divisions c. genetic traits can overcome environmental influences d. age-related physical changes are controlled only by genetic factors

a "biological clock" ticks off a predetermined number of cell divisions

What is emphysema?

a condition in which the air sacs of the lungs are damaged and enlarged, causing breathlessness.

patency

a condition of being opened and unblocked

remission

a decrease inseverity of a disease or any of its symptoms

What is a cause of pernicious anemia? a. An iron deficiency b. A deficiency of vitamin B12 c. Low serum potassium level d. Blood loss

a deficiency of vitamin B12

nasal cannula

a device for delivery of oxygen by way of two small tubes that are inserted into the nares

irrigations

a gentle washing of an area with a stream of solution delivered through an irrigating syringe

An older adult is taking diazepam several times a day. What does the nurse specifically monitor for?

a hangover effect

tympany

a high pitched like drumlike sound produced by performing percussion over a hollow organ such as the stomach

urinary catheter

a hollow, flexible tube that can be inserted into the bladder through the meatus and urethra to withdraw urine or instill medication

chevron

a method of applying tape to secure an intravenous line a narrow piece (1/2 inch wide) of sterile tape is placed under hub of catheter with adhesive side up and then crisscrossed over the hub to form an inverted V no longer used

nasogastric tube (NG tube)

a pliable tube that is inserted through the patients nasopharynx and into the stomach

The nurse is aware that an older adult may become socially isolated related to the loss of __________. (Select all that apply.) a. a spouse b. friends to death or relocation c. health d. home e. culture

a spouse friends to death or relocation health home

Adenosine Triphosphate (ATP)

a substance produces in the mitochondria from nutrients and is capable of releasing energy that turn in enables the cell to work

tracheostomy

a surgical opening into the trachea through which an indwelling tube may be inserted; created by a tracheotomy

palpation

a technique used in physical examination in which the examiner feels the texture, size, consistency and location of certain parts of the body with the hands

Which of the following is (are) produced by the rapid and incomplete breakdown of fatty acids? a. Acetone and ketoacids b. Lactic acid c. Amino acids d. Urea

acetone and ketoacids

A child has been having an asthma attack for the last 8 hours. Because of the child's inability to exhale effectively, the nurse assesses for respiratory __________.

acidosis

edema

abnormal accumulation of fluids interstitial spaces of tissue

tachycardia

abnormal condition in which the myocardium contracts regularly but at a rate greater than 100bpm

cheyne-strokes respirations

abnormal pattern of respiration characterized by alternating periods of apnea and deep rapid breathing

bruits

abnormal swishing sound heard over organs, glands, and arteries.

contracture

abnormal usually permanent condition of a joint characterized by edema, discoloration and pain

as in drug absorption the process whereby a drug moves from the muscle, digestive tract, or other site of entry into the body toward the circulatory system

absorption

How does the nurse define a role? a. Positive standard of behavior b. Accepted behavior standard c. Sexually linked standard d. Unchangeable standard

accepted behvior standard

How can a nurse most effectively promote social contact and interaction in the long-term care facility? a. Make a telephone available for family contact. b. Post a list of activities on the bulletin board. c. Tell the patient about social rooms in the facility. d. Accompany the patient to a craft activity.

accompany the patient to a craft activity

Which of the following processes forms the most ATP? a. Glycolysis b. Conversion of glucose to glycogen c. Aerobic catabolism of glucose d. Catabolism of glucose to lactic acid

aerobic catabolism of glucose

prejudice or discrimination against a particular age group especially older adults

ageism

The home health nurse periodically interviews patients relative to their use of _________ because it is the most commonly consumed and abused nonprescription drug used by adults.

alcohol

What would the home health nurse expect the depressed patient to use in excessive amounts as the depression increases? a. Alcohol b. Corticosteroids c. Caffeine drinks d. Comfort foods

alcohol

In the acute care setting, the nurses role in assisting client's to meet nutritional needs include: A. removing the tray cover before placing the tray on the overbed table. B. attractively setting up the food on the table C. Checking to make certain the correct diet has been given. D. all of the above

all of the above

What is the other name for Vitamin E ?

alpha-tocopherol

The nurse assesses that although the college professor has retired, he still perceives himself as an educator and retains his academic title and professional association membership. How could the change in employment status be reflected in the professor? a. Altered self-image b. Neglect of his family c. Identity crisis d. Habitual professional behavior

altered self image

The nurse directs an 80-year-old recovering from a fractured pelvis to participate in several isometric exercises to maintain muscle strength, such as a. alternately tightening and relaxing the abdominal muscles. b. lifting the body up off the bed using an overhead trapeze. c. pushing against the bed to lift the buttocks off the bed a few inches. d. pressing the sole of the foot against a footboard.

alternately tightening and relaxing that abdominal muscles

An -NH2 group and a -COOH group are most characteristic of a(n) a. fatty acid. b. amino acid. c. monosaccharide. d. ketoacid.

amino acid

What are the building blocks of protein?

amino acids

Which of the following are joined together by peptide bonds? a. Amino acids b. Monosaccharides c. Lipids d. Steroids

amino acids

Glomerular Filtration Rate (GFR)

amount of filtrate formed in all the renal corpuscles of both kidneys each minute and leads to output of urine

lack or loss of appetite resulting in the inability to eat

anorexia

An older adult is coming to your clinic for treatment for a gastric ulcer. Which treatment would the nurse expect to be ordered?

antibiotics

chemicals or other agents that retard or inhibits oxidation of a substance to which they are added. Examples are vitamins, carotenoids, selenium, and phytochemicals

antioxidants

A vague uneasy feeling, the source of which is often nonspecific or unknown to the individual

anxiety

disease

any disturbance of a structure or function of the body.

dysrhythmias

any disturbance or abnormality in a normal rhythmic pattern, specifically the heart.

joint

any one of the connections between bones

Sentinel Event

any unexpected occurence involving death or serious physical or psychological injury

Safety Reminder Device

anyone of numerous devices used to immobilize the patient

abnormal neurologic condition in which language function is disordered or absent because of an injury to certain areas of the cerebral cortex.

aphasia

The Krebs cycle and electron transport chain enzymes a. are located within the mitochondria. b. function under anaerobic conditions. c. generate lactic acid. d. are responsible for glycolysis.

are located within the mitochondria

base of support

area of which an object rests; a stance with feet slightly apart

circumorbital

around the orbit of the eye

Under which condition will the renal reabsorption of bicarbonate increase? a. As a compensatory mechanism for respiratory acidosis b. Whenever blood pH exceeds 7.45 c. As a compensatory mechanism for metabolic alkalosis d. Whenever a person hyperventilates

as a compensatory mechanism for respiratory acidosis

An 85-year-old man accompanied by his son is unable to recall the medications that he is presently taking. What would be the most appropriate response by the nurse? a. Ask the question again. b. Rephrase the question. c. Ask the son for the information. d. Leave that part of the health history blank.

ask the son for the information

An older adult in a long-term care facility has been diagnosed with depression. What would indicate a need for further assessment of suicide risk? a. Asking for assistance in writing a will b. Voicing the intention to visit his brother c. Donating excess clothing to charity d. Asking the young widow next door for a dinner date

asking for assistance in writing a will

In what respect do Baby Boombers differ from young adults? (Select all that apply.) a. Experience with technology b. Attitude about lifestyles c. Value of money d. Methods of communication e. Perceptions of gender roles

attitude about lifestyles value of money methods of communication perceptions of gender roles

A technique of assessment that uses the sense of hearing to detect sounds produce within the body such as heart, lung, and bowel sounds

auscultation

muskoskeletal disorders

back injuries

Adenine, thymine, cytosine, and guanine are a. bases of DNA. b. sugars used to synthesize nucleotides. c. intermediates of the Krebs cycle. d. amino acids.

bases of DNA

Adenine, uracil, cytosine, and guanine are a. sugars used to synthesize nucleotides. b. bases of RNA. c. nonessential amino acids. d. ketone bodies.

bases of RNA

What is the major risk of polypharmacy for the older adult?

being treated by more than one physician

dorsiflexion

bending of flexing backward as in upward bending of the fingers, wrist, foot, or toes

Three body systems work at different speeds to keep the pH in the narrow range of normal. What is the order of effectiveness for these three systems? a. Blood buffers, kidneys, and lungs b. Kidneys, lungs, and blood buffers c. Blood buffers, lungs, and kidneys d. Lungs, kidneys, and blood buffers

blood buffers, lungs, and kidneys

Gluconeogenesis converts a. glucose to amino acids for protein synthesis. b. glucose to fatty acids for storage of excess sugar as fat. c. body protein to glucose in an insulin-deficient state. d. ammonia to urea for excretion into the urine.

body protein to glucose in an insulin-deficient state.

Why does the nurse frequently assess an older adult who is on a psychotropic drug?

brain receptors have become hypersensitive

An older adult is performing isometric and isotonic exercises. What should the nurse suggest in order to prevent an accidental Valsalva maneuver? a. Hold the breath during an exercise cycle. b. Breathe through the mouth. c. Breathe deeply and rhythmically during an exercise cycle. d. Breathe in through the nose and out through the mouth.

breathe through the mouth

What is a TIA?

brief episodes of cerebrovascular insufficiency that are usually the result of cerebral blood vessel obstruction (TRANSIENT ISCHEMIC ATTACK)

The fiercely independent 90-year-old woman who is recovering from a stroke frequently ambulates without the use of her walker because she says it is ugly and cumbersome. What would be the most effective intervention of the nurse? a. Allow her to ambulate independently. b. Place a gait belt around her and ambulate when she does. c. Bring her walker to her and remind her that the walker is for her safety. d. Instruct her to use a wheelchair for mobility.

bring her walker to her and remind her that the walker is for her safety

The patient asks how the continuous positive airway pressure (CPAP) machine decreases the incidence of sleep apnea. What is the correct response by the nurse? a. By stimulating inspiration to be deeper b. By taking over respiratory activity when the patient ceases to breathe c. By sounding an alarm if respirations have ceased d. By preventing relaxation of the tissues

by preventing relaxation of the tissues

general ill health and malnutrition marked by weakness and emaciation, usually associated with severe disease such as tuberculosis or cancer

cachexia

Which of the following is descriptive of cholesterol? a. Protein b. Functions as an enzyme c. Can be synthesized by the liver d. Primary sequence and peptide bonds

can be synthesized by the liver

Third spacing a. is caused by aggressive diuretic therapy. b. describes a severe state of hypovolemia and dehydration. c. involves only the interstitial space. d. can result in an abnormal accumulation of large amounts of water.

can result in an abnormal accumulation of large amounts of water

Dietary cellulose a. is absorbed from the digestive tract and converted to glucose and fuels the cells. b. is converted to protein by the process of gluconeogenesis. c. is catabolized by the liver to ketone bodies. d. cannot be metabolized but is beneficial because it provides fiber and bulk.

cannot be metabolized but is beneficial because it provides fiber and bulk

Which of the following is most descriptive of glucose? a. Disaccharide b. Peptide c. Glycerol d. Carbohydrate

carbohydrate

Starches, amylases, and disaccharidases are related to a. carbohydrates. b. steroids. c. enzymes. d. peptides.

carbohydrates

excessively emotional responses

catastrophic reactions

What are Heberden nodes?

caused by abnormal cartilage or bony enlargement, may be seen in the DISTAL finger joints

infection

caused by an invasion of microorganism such as bacteria, viruses, fungi, or parasites that produce tissue damage

chronologic age the number of years a person has lvied

chronologic age

blood buffers

circulate throughout the body in pairs, neutralizing excess acids or bases by contributing or accepting hydrogen ions weak acid and its base salt or weak base and its acid salt

A patient who has hypertension is complaining about the lack of taste with the low- sodium diet that has been prescribed. What should the nurse emphasize that sodium may do?

contribute to hypertension

pertaining to the mental process of comprehension judgement memory and reasoning as contrasted with emotional process

cognition

term used by demographics to describe a group of people born within a specific time period

cohort

What are the symptoms of hypothyroidism?

cold intolerance, dry skin, dry and thin body hair, constipation, depression, and lack of energy.

dumping syndrome

combination of profused perspiration, nausea, vertigo, and weakness experienced by patients who have had a subtotal gastrectomy; symptoms felt soon after eating when the contents of the stomach empty too rapidklly into the duodenum

focused assessment

concentration of attention on the part of the body where signs and symptoms are localized or most active in order to determine their significance

hypothermia

condition of abnormally low body temp

Poison

condition or physical state produced by the ingestion, injection, inhalation, or exposure

pulse deficit

condition that exist when the radial pulse rate is less than the ventrical rate as ausculated at the apex of the heart

The person who is unable to identify time, place, or person to the point that he or she is unable to make appropriate decisions is described as __________.

confused

a mental state characterized by disorientation regarding time, place, or person that leads to bewilderment perplexity, lack of orderly thought, and the inability to choose or act decisively and to perform activities of daily living

confusion

The nurse emphasizes that the relatively new theory that correlates restricted caloric intake to slowing of the aging process would probably extend the life span of the person, provided that the person __________. (Select all that apply.) a. consistently eats high-nutrient, low-calorie foods b. maintains a regular exercise program c. consumes 2000-3000 mL of fluid a day d. supports the diet with adequate fat-soluble vitamins e. eats only organically grown foods

consistently eat high nutrient, low calorie foods maintain a regular exercise program

What is one of the biggest reason why a client would stop taking an iron supplement without his doctors consent?

constipation

The nurse in a long-term care facility notes signs of depression in a resident who is ordinarily positive. What newly prescribed medication could contribute to the depression? a. Erythropoietin b. Corticosteroids c. Calcium replacement d. Broad-spectrum antibiotics

corticosteroids

proposes that cell molecules from DNA and connective tissue interact with free radicals to cause bonds that decrease the ability of tissue to replace itself.

crosslink or connective tissue theory

What are symptoms of Hypernatremia

decreased myocardial control, low bp, dry and flaky skin

What benefit is there for an older adult to participate in balance training? a. Increase peripheral circulation b. Increase strength c. Decrease the incidence of falls d. Eliminate the need for ambulatory assistive devices

decrease the incidence of falls

Know the respiratory changes associated with aging

decreased body fluids decreased number of cilia decreased number of macrophages decreased tissue elasticity in the alveoli and lower lung lobes decreased number of capillaries increased calcification of cartilage

What is a cause for increased rate of drug absorption in an older patient?

decreased gastric motility

a mental disorder characterized by disturbances in cognition attention memory and perception. Symptoms include confusion disorientation restlessness, clouding of consciousness, incoherence fear and anxiety and excitement. it is visual origin and at time delusions

delirium

a general term for a permanent or progressive organic material disorder, characterized by personality changes, confusion, disorientation, and deterioration of intellectual functioning and by impaired control of memory, judgement and impulses

dementia

a general term for a permanent or progressive organic mental disorder, characterized by personality changes confusion disorientation and deterioration of intellectual functioning and by impaired control of memory, judgement, and impulses

dementia

a mental state of depressed mood characterized by feelings of sadness, despair, and discouragement ranging from normal feelings of the blues all the way to major clinical depression. Resembling the grief and mourning that follow bereavement, the symptoms of depression include feelings of low self-esteem, guilt an d self-reproach; withdrawal from interpersonal contact and somatic symptoms, such as eating and sleep disorders

depression

Sucrose, lactose, and maltose are classified as a. enzymes. b. disaccharides. c. peptides. d. nucleotides.

disaccharides

A patient is ventilating at a rate of 8 to 10 respirations/min, having been heavily medicated for pain. Which of the following is true? The patient is (may) a. develop a respiratory acidosis. b. experiencing Kussmaul respirations. c. experiencing diuresis. d. likely to develop a metabolic alkalosis.

develop a respiratory acidosis

chronic

developing slowly and persisting for a long period often for the remaining of an individuals life

bladder training

development of the use of the muscles of the perineum to improve voluntary control over voiding

Lactic acidosis a. is caused only by rapid and incomplete catabolism of fatty acids. b. is accompanied by the formation of large amounts of ATP. c. develops in response to prolonged anaerobic catabolism. d. is caused only by extreme exercise characterized by muscle soreness.

develops in response to prolonged anaerobic catabolism

bedpan

device for receiving feces or urine from patients confined to bed

Spygomomameter

device used for measuring arterial BP

How is essential hypertension diagnosed?

diagnosed based on two elevate blood pressure determinations on three seperate days

pulse pressure

difference between systolic and diastolic BP usually 30-40 mmhg

When the patient complains of dry mouth, what should the nurse assess for? a. Difficulty in chewing and swallowing b. Mouth ulcerations c. Adequate intake of vitamin B d. Inflammation of the tongue

difficulty in chewing and swallowing

Which of the following conditions is most apt to cause a dilutional hyponatremia? a. Excess ingestion of water b. Dehydration c. Hypotension d. Diaphoresis

dilutional hyponatremia

What is the best way to assess psychological status in an older adult? a. Direct observation b. Input from family members c. Subjective findings d. Review of nursing notes

direct observation

When the home health nurse suggests a grief resolution group to the older widow, the widow furiously states, "I don't want or need any help! I want my husband back!" What stage of grief would this be characteristic of? a. Shock b. Searching c. Disorientation d. Reorganization

disorentation

Which stage of grief usually resolves after approximately 4-7 months? a. Disorientation b. Shock and numbness c. Reorganization d. Searching and yearning

disorientation

as in drug distribution, the pattern of distribution of drug molecules by various tissues after the chemical enters the circulatory system

distribution

the presence of punchlike herniations (diverticula) throughout the muscular layer of the colon, which develop because of weaknesses in the intestinal mucosa

diverticulosis

Which of the following assessments are the cardinal signs and symptoms of congestive heart failure? a. Dyspnea and edema b. Myocardial pain and hypotension c. Ventricular arrhythmias and cyanosis d. Atrial arrhythmias and polycythemia

dyspnea and edema

Which of the following is most likely to occur when output is less than intake? a. Dehydration b. Edema c. Hypovolemia d. Polyuria

edema

defecation

elimination of bowel waste

What often worsens the tremors in Parkinson's ?

emotional stress or fatigue

the willingness to attempt to understand the unique world of another person; the ability to put oneself in another person's place and to understand what he or she is feeling and thinking in various situations

empathy

What would be the best method for the nurse to help a grieving widow begin the grief resolution process? a. Distract her with social conversation. b. Encourage her to verbalize her grief. c. Point out behaviors that are not helpful to grief resolution. d. Focus on her loneliness and loss.

encourage her to verbalize her grief

The newly admitted 80-year-old female resident sits in her room and stares at the TV all day. What action by the nurse would best aid the patient in increasing self-esteem? a. Encouraging her to participate in self-care activities b. Suggesting that she change the channel to an intellectually stimulating program c. Giving her privacy until she becomes accustomed to the long-term care facility d. Arranging for a meal tray rather than having her eat in the dining room

encouraging her to participate in self-care activities

The theory that proposes that defects in ribonucleic acid (RNA) protein production cause a progressive decline in the function of all cells is the __________ theory.

error

as in drug excretion the process eliminating shedding or getting rid of a drug by body organs or tissues as part of natural metabolic activity

excretion

endemic

expected or normal incidence native to or occuring naturally to a specific area or environment

Sodium is the most abundant electrolyte in the body. The location of electrolytes is important for maintaining homeostasis. Sodium is the major electrolyte in which fluid compartment? a. Intracellular b. Intravascular c. Extracellular d. Interstitial

extracellular

The nurse uses a diagram to show that fluids in the interstitial and intravascular compartments are combined. What do they combine to form? a. Intercellular compartment b. Circulating compartment c. Vertical compartment d. Extracellular compartment

extracellular compartment

What is paroxysmal nocturnal dyspnea and how would you alleviate it?

extreme orthopnea during sleep; decrease their activity

The nurse takes into consideration that the most common injuries to the older adult are the result of __________.

falls

a response to a perceived threat that is consciously recognized as danger.

fear

Occupation Safety Health Act

federal agency responsible for monitoring safety and health standards in a workplace

centers for disease control and prevention

federal agency that provides facilities and services for investigation, identification, prevention, and control of science

information produced by a receiver and perceived by a sender that informs the sender of the receivers reaction to the message

feedback

During the health interview, the older adult complains of vague physical gastrointestinal complaints, the inability to fall asleep, frequent periods of wakefulness during the night, and a decrease in appetite. What should the nurse inquire about? a. Feelings of depression b. Time of last bowel movement c. Environmental stimuli that disturb sleep d. Frequency and size of meals

feelings of depression

the phase of drug absorption when oral medications take a first pass through the liver before entering the systemic circulation

first pass effect

What is the purpose of the Beers criteria?

identifies medications best avoided by the older adult

intracellular fluid

fluid inside cell of the body

intravascular fluid

fluid or plasma within vessels of the body

exudate

fluids cells, or other substances that have been slowly exuded or discharged from body cells or blood vessels through small pores or breaks in cell membrane

What is the term for a mottled tooth enamel related to having consumed too much fluoride called?

fluorosis

What is a correct method of administering a transdermal medication patch?

fold and dispose of the used patch in the sharps container

What is the name for vitamin B9?

folic acid

The physician states that the patient with Alzheimer disease has progressed from the preclinical stage of the disease to the mild cognitive impairment stage. What would the nurse expect to find upon assessment of the patient? a. Inability to communicate b. Incontinent episodes c. Total dependency d. Forgetfulness

forgetfulness

At approximately 4 to 6 months of age, solid food is introduced to a baby. What foods with high iron content should be recommended by the nurse?

fortified cereals

drainage

free flow or withdrawal of fluids from a wound or cavity by some sort of sytem

The theory that identifies an unstable molecule as the causative factor in aging is the _____ theory. a. free radical b. molecular c. neuroendocrine d. crosslink

free radical

unstable molecule produced by the body during the normal processes of respiration and metabolism or following exposure to radiation and pollution. suspected of causing damage to the cells, DNA, and immune system

free radical

On the admission assessment of an 80-year-old to a long-term care facility, the nurse notes that the resident's toenails are dark, thick, and brittle; extremely misshapen; and growing at an angle from the toe. What does the nurse suspect has caused the changes in the nails? a. Fungal infection of the nails b. Ram's horn nails c. Ingrown nails d. Expected age-related changes in the nails

fungal infection of the nails

proposes the existence of one or more harmful genes that activate over time, resulting in the typical changes seen with aging and limiting the life span of the individual

gene theory

Hyperkalemia a. refers to an elevated serum calcium level. b. generally develops in response to declining renal function. c. is often caused by a kaliuretic diuretic. d. is caused by excess aldosterone.

generally develops in response to declining renal function

the branch of medicine dealing with the physiologic characteristics of aging and the diagnosis and treatment of diseases affecting older adults

geriatrics

The nurse clarifies that the term that refers to the promotion of high-level functioning and supportive care to older adults is __________.

gerontics

the study of all aspects of the aging process, including the clinical, psychologic, economic, and sociologic

gerontology

the fear of aging and the refusal to accept older adults into the mainstream of society.

gerontophobia

the study of how older adults respond to medication

geropharmocology

The 80-year-old man complains that when he goes to bed and cannot fall asleep, he becomes very upset. What would be an appropriate suggestion for when he has not fallen asleep within 30 minutes? a. Take two tablets of a sedative medication. b. Get up and do a mild stretching exercise for 15 minutes. c. Remain in bed with his eyes closed. d. Get up and read until he feels sleepy and then return to bed.

get up and read until he feels sleepy and then return to bed

disease characterized by increased fluids pressure (intraocular pressure) within the eye may result in damage to the retina

glaucoma

Gluconeogenesis is a process that makes a. glucose from glycogen. b. glucose from protein. c. protein from glucose. d. protein from fat.

glucose from protein

An osmotic diuresis is most commonly caused by a. ketonuria. b. albuminuria. c. glucosuria. d. hypoglycemia.

glucosuria

Which of the following is not a carbohydrate term? a. Glycogen b. Monosaccharide c. Glycerol d. Sucrose, maltose, and lactose

glycerol

Which of the following describes the breakdown of glucose to lactic acid? a. Aerobic b. Krebs cycle c. Glycolysis d. Gluconeogenesis

glycolysis

The hospice nurse explains that dying in peace without pain and with dignity in a supportive care environment is defined as a(n) _________ death

good

a combination of sorrow loss and confusion that comes when someone or something of value is lost.

grief

medical asepsis

group of techniques that inhibit the growth and spread of pathogenic microorganism

the amount of time required to reduce a drug level to its half initial volume

half-life

induration

hardening of a tissue, particularly the skin, because of edema, inflammation, or infiltration by neoplasm.

acute

having a short and relatively severe cause

Tredelenburgs

head low and the body and legs are on incline plane

a systemic program planned to prevent illness to maintain maximal function and to promote health

health maintenance

lifestyle and health care practices that improve overall health and quality of life

health promotion

The 70-year-old woman who had a stroke 3 months ago has a body image disturbance related to her spastic right arm and contracted fingers of the right hand. How can the nurse assist the patient in improving her body image? a. Teach her to write with her left hand. b. Place articles within easy reach of her left hand. c. Help her select colorful scarves or accessories to cover her right arm. d. Show her massage techniques to increase circulation in her right arm.

help her select colorful scarves or accessories to cover her right arm

a feeling of loss of control or ability usually after repeated failures, or being immobilized or frozen by circumstances beyond one's control with the result that one is unable to make autonomous choices.

helplessness

vision loss in half of the visual field of one or both eyes

hemianospia

What are the common indicatiors of prostatic hyperplasia?

hesistancy, frequency, the inability to maintain a steady stream of urine, and urinary retention are common indicators

A society that has numerous subcultures with different or conflicting expectations is said to be a(n) __________ society

heterogeneous

Which ion determines pH? a. Sodium b. Hydrogen c. Bicarbonate d. Potassium

hydrogen

What is the toxicity of sodium?

hypertension (high blood pressure) and increased calcium excretion

Which of the following is most apt to cause a hypocalcemic tetany? a. Hyperventilation b. Rapid and excess fat catabolism c. Metabolic acidosis d. pH < 7.35

hyperventilation

Which of the following conditions is most apt to decrease glomerular filtration rate (GFR)? a. Overhydration b. Hypotension c. Administration of a diuretic for the treatment of edema d. Water intoxication

hypotension

reduced function of the thyroid gland.

hypothyroidism

nursing health history

identification by nurse of the needs, preferences and abilities of a patient assessment provides the scientific basis for a complete nursing care plan

immobility

inability to move around freely, caused by any condition in which movement is impaired or therapeutically restricted

Which of the following may be a thermoregulation risk in the older adult? a. Inactivity b. Eating highly spiced foods c. Being overweight d. Mental illness

inactivity

lithotomy

incision of a duct or organ especially the bladder for removal of a stone

Monosaccharides a. include glucose, fructose, and galactose. b. include sucrose, lactose, and maltose. c. are classified as saturated and unsaturated. d. are carbohydrate-splitting enzymes.

include glucose, fructose, and galactose

What is the medical treatment for thrombophlebitis?

includes rest, elevation of the affected leg, application of elastic stockings or wraps, administration of analgesics, anticoagulant therapy, and sometimes application of heat

What happens in older adults that do not perspire as much?

increased dry skin; decreased prespiration means decreased sebaceous and sweat gland function (xerosis)

The physician has written an order to convert an enteric-coated medication from the pill form to the liquid form. What would be the most appropriate response of the nurse?

inquire if the physician wants the dose to be the same as the pill

intravenous (IV)

inside of the vein as of a thrombus or an injection infusion or cathether

the most commonly used method of physical assessment in which the senses of vision smell and hearing are used to collect data

inspection

stethoscope

instrument placed against a patients body to hear heart, lungs, or bowel sounds

potential ability and capacity to acquire retain and apply experience understanding knowledge reasoning and judgement in coping with new experiences and in solving problems

intelligence

cramplike pains in the calves caused by poor circulation of the blood to the leg muscles commonly associated with atherosclerosis (the tissues of lower extremities are deprived of oxygen); manifested only at certain times usually after walking; and relieved by rest

intermittent claudication

A man has roles of father, husband, professional businessman, son, and community leader. What may he be at risk of experiencing? a. Role confusion b. Isolation c. Internal role conflict d. Diminished self-esteem

internal role conflict

What does perception refer to in the environment? a. Intellect b. Memory c. Judgment d. Interpretation

interpretation

A 59-year-old construction worker living with his divorced son and 9-year-old grandson was admitted to the hospital 3 days ago with a massive heart attack. His son is out of town on a business trip and has been unable to visit him. His grandson had to go stay with a friend. The social worker tells the nurse that the son stated he will not be able to care for his father when he gets out of the hospital and does not know how he will afford the skilled nursing facility. What is the most appropriate nursing diagnosis? a. Impaired social interaction b. Dysfunctional grieving c. Social isolation d. Interrupted family process

interrupted family process

What is the correct terminology for the area within 18 inches of a person?

intimate space

catherization

introduction of a rubber or plastic tube through the urethra and into the bladder

What can a nurse in a long-term care facility do to best promote normal circadian rhythm in all patient rooms? a. Keep bright lights on during the daytime. b. Dim lights to promote relaxation. c. Maintain appropriate environmental temperature. d. Pull curtains for privacy.

keep bright lights on during the daytime

What communication technique would be most helpful during the health interview? a. Use medical terminology. b. Keep questions simple. c. Help patients by finishing their sentences. d. Allow patients to ramble as they respond.

keep questions simple

Supination

kind of rotation that allows palm of hand to turn up

The Nursing Competence in Aging initiative advocates enhancing nurses' __________. (Select all that apply.) a. knowledge in gerontics b. skills in geriatrics c. opportunities for employment d. political sensitivity for the older adult e. attitudes related to the older adult

knowledge in gerontics skills in geriatrics attitudes related to the older adult

Which of the following is least related to lactic acid? a. Glycolysis b. Krebs cycle c. Anaerobic d. Cytoplasm

krebs cycle

The white female nurse is concerned that the 80 year old African American male patient is not being truthful with her because of his

lack of eye contact

What exactly happens to cause ischemia?

lack of oxygen

In the absence of oxygen, glucose is catabolized to a. lactic acid. b. carbon dioxide and water. c. urea. d. fatty acid.

lactic acid

The 75-year-old resident in a long-term care facility complains of muscle pain while riding a stationary bicycle. The nurse explains that the discomfort is related to the buildup of __________ __________ in the muscle

lactic acid

Which of the following is most related to glycolysis? a. Krebs cycle b. Mitochondria c. Lactic acid d. Aerobic

lactic acid

auscultate

listen to sounds within the body to elevation the condition of the heart, lungs, pleura, intestines, and other organs or to detect fetal heart sounds

The primary organ of drug metabolism is the __________.

liver

Where is glycogen stored in the body?

liver and skeletal muscle

Which of the following is (are) necessary for the synthesis of urea? a. Ketone bodies b. Insulin c. Glycogen d. Liver enzymes

liver enzymes

What is presbycusis and how does it affect the aging person?

loses the ability to perceive high frequency tones the aging person may only hear parts of spoken words

borborygmi

loud, gurgling sounds that accompany increased motility of the bowel

The nurse reminds a group of prospective caregivers that elder abuse may take the form of __________. (Select all that apply.) a. caring for physical needs b. misappropriation of finances c. psychological intimidation d. emotional depersonalization e. abandonment

misappropriation of finances psychological intimidation emotional depersonalization abandoment

This anion is an important part of a major buffer pair and therefore plays an important role in acid-base regulation. a. Na+ b. Bicarbonate c. HCO3- d. More than one of the above are true.

more than one of the above are true

Transcription a. occurs within the nucleus. b. involves DNA and mRNA. c. involves tRNA and mRNA. d. More than one of the above are true.

more than one of the above are true

a unit of the hospital with facilities for the storage and autopsy of the dead

morgue

Sodium

most abundant extracellular cation and in the body

venipuncture

most common method of drawing a blood sample, involving inserting a hollow bore needle into the lumen of a large vein

extension

movement of allowed by certain joints of the skeleton that increases the angle between two adjoining bones

flexion

movement of certain joints that decreases the angle between two adjoining bones

Range of Motion (ROM)

movement of the body that involves the muscles and joints in natural directional movements

Generally speaking the movement of Na+ determines the a. blood pH. b. blood glucose. c. secretion of PTH. d. movement of water.

movement of water

What action by the 75-year-old would indicate that an appropriate action has been taken to combat reduced stamina? a. Moving his home office to a downstairs location b. Using public transportation rather than driving his own car c. Tilling the garden plot with a motor-driven tiller d. Visiting the senior center twice in 1 week to play dominoes

moving his home office to a downstairs location

Chloride

negatively charge extracellular anion present alone bond to potassium or sodium; a salt of hydrochloric acid

Anions

negatively charged ions that when in solution is attracted to the positive electrode

passive form of abuse in which caregivers fail to provide for the needs of the person under their care

neglect

focuses on the complicated chemical interactions set off by the hypothalamus of the brain

neuroendocrine theory

Pellagra is a disease in which the mucous membranes of the mouth and digestive tract become red and inflamed and lesions appear on the skin. What vitamin problem is this disease linked to

niacin deficiency

What is the deficiency of Vitamin A ?

night blindness, xerophthalmia, increased susceptibility to infections, follicular hyperkeeatosis

Which of the following is most descriptive of ammonia? a. Nitrogen containing and brain toxic b. A catalyst c. An essential amino acid d. Derived from glycogen catabolism

nitrogen containing and brain toxic

Which of the following is characteristic of urea? a. Formed in the kidneys and excreted by the liver into bile b. Nitrogen-containing waste formed in the liver c. Characterized as an essential amino acid d. Classified as a disaccharide

nitrogen-containing waste formed in the liver

Which group is correct? a. Lipids: triglycerides and urea b. Polysaccharides: glycogen and glucose c. Amino acids: ammonia and glycerol d. Nitrogen-containing waste: urea and creatinine

nitrogen-containing waste: urea and creatinine

Range of motion

normal movement that any given joint is capable of making

milliequivalent (mEq)

number of grams of soluble substances dissolved in 1 ml of normal solution

Which of the following foods is good source for magnesium?

nuts

rapid; involuntary eye movement

nystagmus

A newly admitted older adult asks for a sedative every night and states that she cannot fall asleep. What type of sleep disorder is the patient most likely experiencing? a. Onset related to anxiety of relocation b. Maintenance related to unfamiliar environment c. Initiation related to depression associated with relocation d. Maintenance related to episodes of nocturnal movement disorders

onset related to anxiety of relocation

The nurse charts, "Patient stated pain is at a level of 8 on a scale of 1 to 10 and that he is still nauseated. Patient complains of feeling cold and has an oral temperature of 97.8°." What information would be considered objective findings? a. Pain measurement b. Presence of nausea c. Sense of cold d. Oral temperature

oral temperature

A patient has just begun taking an iron supplement as prescribed by their doctor. The patient should be encouraged to take the iron supplement at the same time as: A. coffee B. orange juice C. milk D. fiber cereal

orange juice

condition that occurs because the circulation does not respond quickly to postural changes

orthostatic hypotension

condition that occurs because of circulation does not respond quickly to postural changes

orthrostatic hypotension

a hereditary condition of the bony labrinyth of the ear in which there is formation of spongy bone resulting in hearing loss

otosclerosis

peripheral

outside, surface, or surrounding area of the organ, other structure or fluid of vision

genupectoral

patient kneels so weight of body is supported by knees and chest

Sims

patient lies on side with knee and thigh drawn upward toward chest

orthopneic

patient sitting up in bed at 90 degree angle or sometimes resting forward tilt while supported by pillow overbed

What does the following describe: Alanine-leucine-cysteine-phenylalanine-proline-arginine? a. Fatty acid b. Peptide c. Disaccharide d. Nucleotide

peptide

the conscious recognition and interpretation of sensory stimuli that sensory stimuli that serve as a basis for understanding learning and knowing or for motivating a particular action or reaction

perception

a technique of physical assessment in which the size position and density of structures under the skin are assessed by tapping the area and listening to the resonance of the sound. Depending on the amount of vibration(sound) heard the presence of masses fluid or air can be determined

percussion

Diastolic

period of time between contractions of the atria or ventures, during which blood enters the relaxed chambers from the systemic circulation and the lungs

What relationship does adipose tissue have with medication administration?

person ages, muscle mass decreases, and the proportion of body weight resulting from fatty or adipose tissue increases

What is the correct term for the area between 18 inches and 4 feet of a person?

personal space

the study of the action drugs within the body including the mechanisms of absorption distribution metabolism and excretion onset of action, duration of effect biotransformation and effects and routes of excretion of the metabolites of the drug

pharmokinetics

How does the concept of ageism portray older adults? a. An inactive population of self-indulgent persons b. A group that has opted to isolate themselves c. Physically inept and nonproductive d. An antisocial but active group

physically inept and nonproductive

body mechanics

physiologic study of the muscular actions and the functions of muscles in maintaining the posture of the body

What oral medication can be safely crushed?

plain antihypertensive medication tablet

What would be included in a plan of care that is custodial in its focus? a. Attention to high-level wellness b. Plans for physiologic and safety concerns c. Aggressive rehabilitation goals d. Patient participation in his or her own care

plans for physiologic and safety concerns

the prescription administration or use of more medications than are clinically indicated a common problem among older adults

polypharmacy

Starch is classified as a(n) a. lipid. b. fatty acid. c. amino acid. d. polysaccharide.

polysaccharide

Glycogen is a(n) a. alcohol, an end product of fat digestion. b. indigestible polysaccharide also called cellulose. c. polysaccharide that is the storage form of glucose. d. plant starch.

polysaccharide that is the storage form of glucose

A deficiency of ADH, called diabetes insipidus, is characterized by a. glucosuria and an osmotic diuresis. b. polyuria and polydipsia. c. Kussmaul respirations and hyperventilation. d. edema formation and sodium retention.

polyuria and polydipsia

Ions

positively and negatively charged atoms; electronically charge particle resulting from the breakdown of an electrolyte

Cations

positively charged ions that when in solution is attached to negatively charged electrode

hyperextension

postion of maximum extension; extreme or abnormal stretching

What special precautions should the nurse take during the physical exam of an 86-year-old woman? a. Weigh the patient carefully without clothing. b. Prevent the patient from becoming chilled during the examination. c. Give fluids before the examination. d. Elevate the patient's head while she is lying in a supine position.

prevent the patient from becoming chilled during the examination

The caregiver who provides daily meals and attends to the daily needs of the older adult is classified as the _______________caregiver

primary

Seventy-seven percent of dietary sodium comes from and intake of: A. meat B. apples C. processed foods D. milk

processed foods

The area 12 feet from a person and beyond is known as _____ space

public

the absence of palpable pulse beats when simultaneously compared with an apical pulse measurement

pulse deficit

Ketosis develops in response to a. glycolysis. b. hyperglycemia. c. activation of the urea cycle. d. rapid and incomplete catabolism of fatty acids.

rapid and incomplete catabolism of fatty acids

What are symptoms of hyponatremia

rapid pulse, shallow breathing, confusion, headache, seizures, increased urine output, hyperactive bowel.

proposes that individuals have infinite number of breaths or heartbeats that are used up over time.

rate of living theory

erythema

redness or inflammation of the skin or mucous membrane resulting from dilation and congestion of superficial capillaries

proposes that every person has a limited amount of genetic material that will run out eventually

run out of program theory

Which of the following foods has the highest amount of calcium?

sardines

What information related to drug administration should be included in the nursing care plan? (Select all that apply.) a. Schedule for drawing blood values b. Patient's need for crushing medication c. Patient's preference as to the use of medium in which to give crushed medicines d. Schedule of medication and dose times e. Parameters of pulse or blood pressure, if significant to administration

schedule for drawing blood values patient's need for crushing medication patient's preference as to the use of medium in which to give crushed medicines parameters of pulse or blood pressure, if significant to administration

Which care plan modification would be the most beneficial for a 62-year-old woman who is suffering from a flare in her rheumatoid arthritis? a. Increase fluid intake b. Schedule several rest periods to balance activity c. Reduce salt in the diet d. Assist with rigorous finger extension exercises

schedule several rest periods to balance activity

Which stage of grief usually resolves after approximately 2 weeks to 4 months? a. Disorientation b. Shock and numbness c. Reorganization d. Searching and yearning

searching and yearning

Which of the following is not a function of the lungs? a. Oxygenation of blood b. Excretion of carbon dioxide c. Secretion of aldosterone and the regulation of Na+ d. Regulation of blood pH

secretion of aldosterone and the regulation of Na+

Define metabolism

series of chemical reactions necessary for the use of raw material

Which stage of grief usually resolves after approximately 2 weeks? a. Disorientation b. Shock and numbness c. Reorganization d. Searching and yearning

shock and numbness

An 80 year old woman injured her foot on a piece of rusty wire. She tells the nurse she had a tetanus booster when she was 75. The nurse's response will be based on the knowledge that tetanus boosters

should be repeated with every injury, regardless of the previous booster.

What measures may the older adult take to reduce the high cost of prescription drugs? (Select all that apply.) a. Simply not fill a new prescription b. Take less than prescribed to preserve their supply c. Fill all prescriptions at once d. Save old prescription drugs for later use e. Share medications

simply not fill a new prescription take less than prescribed to preserve their supply save old prescription drugs for later use share medications

Seeing a patient with his head in his arms resting on the over-the-bed table, the nurse steps into the room and asks if the patient feels ill. The patient, without raising his head, says, "I'm fine." The nurse should:

sit down next to the bed and say, "You dont act fine"

In order to decrease fall risk due to orthostatic hypotension, what advice should be given to an older adult who is taking medication for hypertension? a. Ambulate with a walker. b. Avoid hot baths. c. Avoid climbing stairs. d. Sit on the side of the bed for a moment before ambulation.

sit on the side of the bed for a moment before ambulation

Why is an arthritic patient less likely to exercise? a. Fragility of the bones puts the patient at risk for fractures. b. Numbness in the feet and legs puts the patient at risk for a fall. c. Stiffened ligaments and tendons put the patient at risk for reduced flexibility. d. Moving heavy edematous limbs puts the patient at risk for fatigue.

stiffened ligaments and tendons put the patient at risk for reduced flexibility

Which of the following is true of base sequencing? a. Occurs within the mitochondrion b. Occurs along the ribosome c. Stores the genetic code d. Is a function of the Krebs cycle

stores the genetic code

What would be the most appropriate exercise suggestion for the older adult who wants to build muscle mass? a. Aerobic class b. Stretching exercises c. Strength training d. Tai chi

strength training

etiology

study of all factors that may be involved in the development of a disease

What are the signs and symptoms of intermittent claudication with peripheral vascular disease?

sudden pain pallor pulselessness loss of sensation (pain in legs during or after walking)

a condition in which persons with cognitive impairment and older people tend to become confused or disorientated at the end of the day exhibiting such as behaviors as wandering combativeness suspiciousness hallucinations and delusions

sundowning

dorsal recumbent

supine position with patient lying on the back, with head, shoulders, and extremities, moderately flexed and legs extended.

colostomy

surgical creation of a stoma on the abdominal wall to where the colon is normally attached

Which of the following carries the individual amino acids from the cytoplasm to the ribosome for assembly into a protein? a. DNA b. tRNA c. mRNA d. ATP

tRNA

When the 65-year-old patient who is a type 1 diabetic informs the home health nurse that he now exercises for 1 hour a day at a club, the nurse cautions him to be sure to a. drink plenty of fluids. b. wear clothing that allows ventilation. c. take hard candy to the gym when he exercises. d. give himself less insulin than is prescribed.

take hard candy to the gym when he exercises

The Gene Theory of aging suggests which of the following ? a. The presence of a "master gene" prolongs youth. b. Genes interact with each other to resist aging. c. Specific genes target specific body systems to initiate system deterioration. d. The activation of harmful genes initiates the aging process.

the activation of harmful genes initiates the aging process

urostomy

the diversion of urine away from a disease or defective bladder through a surgically created opening or stoma in the skin

incontinence

the inability to control urination of defecation

enema

the instillation of a solution into the rectum of the sigmoid colon for the purpose of evaluating bowels or introducing a medication

The long-term care facility resident who has not worn his dentures for several months complains that the dentures no longer fit. Why do the dentures no longer fit? a. The gums have hypertrophied. b. The gums have receded. c. The jaw shape has altered. d. The dentures have warped from disuse.

the jaw shape has altered.

passive transport

the movement of small molecules across a cell membrane by diffusion; no cellular energy is required

What is successful communication dependent on? (Select all that apply.) a. The need to share information with someone else b. Empathetic listening c. Assessing or correcting communication barriers d. Using perfect grammar e. Use of a variety of communication skills

the need to share information with someone else empathetic listening assessing or correcting communication barriers use of a variety of communication skills

hydrostatic pressure

the pressure within a blood vessel that tends to push water out of the vessel

ergonomics

the science of matching workplace conditions and job demands to the capability of workers, especially in regards to MSDS and their prevention

pruritus

the symptoms of itching and uncomfortable sensation leading to the urge to scratch; scratching often leads to secondary infection. Some causes of pruritus are allergy, infection, elevated serum urea, jaundice, and skin irritation

Why is drug testing done by pharmaceutical companies not always appropriate for the older adult?

the testing is usually conducted on healthy young persons

Which of the following is descriptive of monosaccharides? a. They are fatty acids. b. They are stored as glycogen. c. They are joined together by peptide bonds. d. The most important is trypsin

they area stored as glycogen

What is true of the psychosocial theories of aging? a. They focus on methods to delay the aging process. b. They are directed at decreasing depression in the older adult. c. They are organized to enhance the perception of aging. d. They attempt to explain responses to the aging process.

they attempt to explain responses to the aging process

The 93 year old woman with chronic back pain is found crying. When the nurse approaches, the patient says, "I know can't do anything more, but I hurt so bad." What nursing intervention would be most effective?

touch the patient's shoulder and sit quietly without speaking

syncope

transient loss of consciousness due to inadequate blood flow to the brain

What does the following describe: Three long-chain fatty acids that are attached to a glycerol molecule? a. Steroid b. Polysaccharide c. Pyrimidine d. Triglyceride

triglyceride

labia majora

two large folds of tissue extending from the mons pubis to the perineal flavor

How do you treat Type 1 and Type 2 diabetes?

type 1 involves careful balance of diet and insulin therapy, exercise and stress management. Type 2 weigh lost encouraged

Terrorism

use or threats of violence by individuals or organized groups using biological chemical or nuclear weapons against society or government for political gain

Cholesterol is a. an essential amino acid. b. stored as glycogen. c. degraded to sucrose, lactose, and maltose. d. used to synthesize steroids.

used to synthesize steroids

percussion

using fingertip to tap the bodys surface to produce vibration and sound

The nurse crushes a pill and disguises the dose in the mashed potatoes of a resident in a long-term care facility who previously refused the drug. It is then fed to the patient by the nursing assistant. This should be considered an error because it __________. (Select all that apply.) a. violates the patient's right to refuse medication b. involves delegation of medication administration to the nursing assistant c. increases the amount of time for the drug administration pass d. becomes impossible to confirm the patient received the entire dose e. alters the food

violates the patient's right to refuse medication involves delegation of medication administration to the nursing assistant becomes impossible to confirm the patient received the entire dose

The nurse is sensitive to the use of nonverbal communication from patients, which includes the interpretation of __________. (Select all that apply.) a. choice of words b. voice tone c. body language d. gestures e. facial expressions

voice tone body language gestures facial expressions

Which of the following is true about these percentages: 60% and 28%? They represent the amount of a. water eliminated as urine and through the skin and lungs. b. Na+ and K+ reabsorbed by the kidneys in response to aldosterone. c. water ingested through moist food and metabolism. d. Na+ and K+ reabsorbed in response to ADH secretion.

water eliminated as urine and through the skin and lungs

presumes that the body is similar to a machine, which loses function when its parts wear out.

wear and tear theory

A patient uses good health maintenance practices. What aging theory most accurately relates to the patient's practices? a. Wear-and-tear b. Free radical c. Neuroendocrine d. Molecular

wear-and-tear


Conjuntos de estudio relacionados

US Hybrid Trimester 2 Final Review

View Set

Medical conditions caused by frequent and prolonged hyperglycemia related to Diabetes type 2

View Set

Flashcards for Assignment 3: An "Unfinished Revolution?" America's Civil War & Reconstruction

View Set

Базовый словарь для специалистов по digital-рекламе ADCONSULT

View Set

BIBL 104-Quiz: The Old Testament Books of Prophecy

View Set

California Driver's Ed Permit Test

View Set

biotechnology chapter 3- quiz 3 and 4

View Set